General Chemistry NYA by Yann Brouillette Revised05 F2016

You might also like

Download as pdf or txt
Download as pdf or txt
You are on page 1of 229

EXTRA CLASS NOTES FOR

GENERAL CHEMISTRY
202-NYA-05

INSTRUCTOR: YANN BROUILLETTE


Dear Students,

Thank you for choosing this class. I hope you will find answers to some of the scientific questions
you may ask yourself.

Be aware that these class notes were put together to ease the comprehension and facilitate the
studying of the College chemistry class entitled: General Chemistry 202-NYA-05.

You are still required to read the text book Chemistry by Zumdahl/Zumdahl, and do the suggested
problems.

Have a good session!

Your Instructor

Yann Brouillette, Ph.


Ph. D.
Chapter 1:
REVIEW
Significant Figures, Scientific Notation and Units
Observations are sometimes qualitative and/or quantitative. For example, “This substance is a
green solid (qualitative) that weighs 5.4 grams (quantitative).

A quantitative observation is called a measurement. A measurement always consists of two


parts: a number and a unit that tells us the scale being used.

1.0 Scientific Notation


Scientific work most often deals with very large numbers or very small numbers, either through
measurements or through calculations. For example, in a sample of 12.0000 g of carbon atoms,
there are:
602 300 000 000 000 000 000 000 atoms.

That is the number 6023 followed by 20 zeroes. Clearly it is unreasonable to write out 20 zeroes
each time this number (known as Avogadro’s Number) is used. Scientific notation has been
devised to write out numbers (such as Avogadro’s Number) in a more compact form which
conveys the same information more quickly.

To see how to do this, it is necessary to note the following:


1
10 = 10
2
100 = 10 x 10 = 10
3
1000 = 10 x 10 x 10 = 10
…..
6
1 000 000 = 10 and so on.

Thus, for example, the number one million, as above is one followed by 6 zeroes. Scientific
notation makes use of this relationship. The following example illustrates how numbers can be
represented in scientific notation.

The number 1234 can be written as the following product:


3
1.234 x 1000 = 1.234 x 10 x 10 x 10 = 1.234 x 10
3
The number, 1.234 x 10 , represents the number 1234 in standard scientific notation.

Scientific Notation simply expresses a number as a product of a number between 1 and 10 and
the appropriate power of 10.

The fast way to do this is simply by counting potential decimal places. For example, if you want to
express the number 5715 in scientific notation, it would be:

5.715 times some power of 10


Locating the decimal after the first
digit
Essentially 5715 = 5715. = 5 7 1 5. The decimal will be moved to the left by 3 places
+3
therefore the factor is 10 .
Thus 5715 = 5.715 x 10
+3 Effective location of the decimal

Yann Brouillette - General Chemistry 202-NYA-05: Chapter 1 Review 1


Applying the same principle to Avogadro’s Number:
+23
602 300 000 000 000 000 000 000 = 6.023⋅10

23 places to the left


For small numbers, note that

1
0.1 = = 10 −1
10
1 1
0.01 = x = 10 −2
10 10
1 1 1
0.001 = x x = 10 −3
10 10 10
.....
0.000001 = 10 −6

For small numbers, movement of the decimal place is to the right.

Scientific notation is very helpful as it makes an excellent shorthand way to write numbers.

• Any number can be represented as the product of a number between 1 and 10 and a power
of 10 (either positive or negative).
• The power of 10 depends on the number of places the decimal point is moved and in which
direction. The number of places the decimal point is moved determines the power of 10. The
direction of the move determines whether the power of 10 is positive or negative. If the
decimal point is moved to the left, the power of 10 is positive; if the decimal point is moved to
the right, the power of 10 is negative.

A number that is greater than 1 will always have a positive exponent when written in scientific
notation. A number that is less than 1 will always have a negative exponent when written in
scientific notation.

Example 1: Write the following numbers in standard scientific notation.


–8
0.0000000721 = 7.21 x 10
–2 1
6319 x 10 = 6.319 x 10
–3
0.005219 = 5.219 x 10
–9
0.000000007251 = 7.251 x 10
3
5219 = 5.219 x 10
–2 +1 –2 +1–2 –1
72.41 x 10 = (7.241 x 10 ) x 10 = 7.241 x 10 = 7.241 x 10
–2
6.319 x 10 = This is already in standard scientific notation
0
1.4325 = 1.4325 x 10
+3
4207 = 4.207 x 10
+2
347.215 = 3.47215 x 10
+7
77000000 = 7.7 x 10
–3
0.0043050 = 4.3050 x 10

Yann Brouillette - General Chemistry 202-NYA-05: Chapter 1 Review 2


Example 2: Write the following numbers as ordinary decimal numbers.

4
8.621 x 10 = 86210

–2
6.235 x 10 = 0.06235

3
7.229 x 10 = 7229

–6
5.001 x 10 = 0.000 005 001

Rules for exponentiation:


x0 = 1
1
= x−a
xa
x a ⋅ x b = x a +b
xa 1
= x a ⋅ b = x a ⋅ x − b = x a−b
xb x
[ xy ]a = x a y a
a
x xa
  = a
 y y
b
(x )
a
= x ab

1.3 Units
The unit part of a measurement tells us what scale or standard is being used to represent the
results of the measurement.

The two most widely used systems are the English (British) system, used in the United States,
and the metric system, used in most of the rest of the industrialized world.

The most comprehensive system of units is called the International System (le système
international in French) and is abbreviated SI. The SI units are based on the metric system and
units derived from the metric system.

Base Units in the SI and British Systems of Units

SI System British System


Physical Quantity Name of Unit Abbreviation Name of Unit Abbreviation
Length metre m foot ft
Mass kilogram kg Pound mass lbm
Time second s Second S
Temperature Kelvin K Rankine °R
Current Ampere A Ampere A

All SI units may be expressed as combinations of the basic units of length, mass, time,
temperature and electrical current. For the SI units Conversion to the English system units,
please refer to the back cover of your text book.

Yann Brouillette - General Chemistry 202-NYA-05: Chapter 1 Review 3


Often the standard SI unit does not provide a convenient scale for representing a measurement.
In such cases, a prefix is attached to the basic SI unit to indicate the order of magnitude of the
particular measurement. The standard prefixes are provided below.

Prefixes for the Powers of Ten

Power Prefix Abbreviation Power Prefix Abbreviation


1024 Yotta Y 10–1 deci d
1021 Zetta Z 10–2 centi c
1018 exa E 10–3 milli m
1015 peta P 10–6 micro µ
1012 tera T 10–9 nano n
109 giga G 10–12 pico p
106 mega M 10–15 femto f
103 kilo k 10–18 atto a
101 deka da 10–21 zepto z
10–24 yocto y
The above list is provided for completeness. It is necessary to memorize only those prefixes (and
12 -15
their abbreviations) that are highlighted (i.e., those between tera, 10 , and femto, 10 ).

For example, a correct use of prefixes, abbreviations, symbols and powers of ten:
–2
One centimetre is one hundredth of a metre ⇒ 1 cm = 10 m,
6
One megawatt is a million watts ⇒ 1 MW = 10 W,
3
One kilohertz is a thousand hertz ⇒ 1 kHz = 10 Hz.

Converting From One Unit to Another


Note that units are treated exactly the same as if they are number. This means that you can
multiply them, divide them, cancel them etc.

Example 1: How many kilograms are there in one gram?


Step 1: Write the given datum on the left hand side of a proposed inequality. On the
right hand side, write the required unit.
1g → ? kg

Step 2: A conversion factor (CF) is needed to relate the two sets of units. First examine
the units of the CF – first examine the given unit. This unit must be cancelled
and replaced with the required unit. The units of the appropriate CF, relating kg
to g, appear within the parentheses.

 kg 
1 g   → ? kg
 g 
Step 3: The numerical portion of the CF must now be determined. In this case, the
definition of “kilo” serves as the numerical portion of the CF.
 kg 
1 g  3  = 10 kg
−3

 10 g 

Yann Brouillette - General Chemistry 202-NYA-05: Chapter 1 Review 4


Example 2: How many nanometres are there in one micrometre?
Step 1: Write the given datum on the left hand side of a proposed inequality. On the
right hand side, write the required unit.
1µ m → ? nm

Step 2: A conversion factor (CF) is needed to relate the two sets of units. First examine
the units of the CF – first examine the given unit. This unit must be cancelled
and replaced with the required unit.
 nm 
1 µ m   → ? nm
µm 
The units of the appropriate CF, relating µm to nm, appear within the
parentheses.
Step 3: The numerical portion of the CF must now be determined. In this case, the
definitions of “micro” and “nano” serve as the numerical portion of the CF. The
solution is actually obtained by breaking the CF into two CF’s. The first related
µm to m and the second relates m to nm.
 nm   m   nm   10 −6 m   nm  10
−6

1 µ m   = 1 µ m     = 1 µ m 
 µm   −9  = −9
nm = 10 −6 ⋅10 +9 nm = 10 −6+9 nm = 10 +3 nm
µm  µm  m     10 m  10

1.4 Uncertainty in Measurement


All measurements involve an element of uncertainty. In experiments, therefore, it will be important
to determine quantitatively how these uncertainties affect the values that are computed from the
data. For example, if five people measure a certain length and obtain the following results:

Person Result of measurement


1 3.15 cm
2 3.14 cm
3 3.16 cm
4 3.15 cm
5 3.16 cm
The first two digits in the example are the same regardless of who made the measurement; these
are called the certain numbers of the measurement. However, the third digit is estimated and can
vary; it is called an uncertain number.

The uncertainty in the last number (the estimated number) is usually assumed to be ±1 unless
otherwise indicated. For example, the measurement 3.21 grams can be interpreted as 3.21 ±
0.01 grams, where the symbol ± means plus or minus. That is, it could be 3.21 − 0.01 g = 3.20 g
or 3.21 + 0.01 g = 3.22 g.

The numbers recorded in a measurement (all the certain numbers plus the first uncertain number)
are called significant figures.

Percentage Error and Percentage Difference


If a generally accepted value of a quantity, Y, is known to exist (referred to as the literature value
of Y) and one (or more) experiments are performed to measure Y, then it is usual to compare the
experimentally measured value of Y with the literature value according to:

%error = Experimental Value − Literature Value x 100%


Literature Value

Yann Brouillette - General Chemistry 202-NYA-05: Chapter 1 Review 5


This %error represents the accuracy to which the experimental value has been measured. If a
literature value is unavailable but the results of two different measurements are, then it is useful
to compute the percentage between the results. One of the methods for computing the
percentage difference is:
The % difference between Y1 and Y2 relative to Y1:
Y2 − Y1
%difference = ⋅ 100%
Y1

1.5 Significant Figures

In science there are two methods of acquiring numbers. One method is by counting; the other is
by measurement. Counting is by its very nature exact. Measurement on the other hand is done
by comparison with a calibrated instrument. Since there are limitations on the scales of all
calibrated instruments, there is a limit to the accuracy to which any measurement may be made.
The use of significant figures expresses the extent of this accuracy. The use of significant figures
is really only an approximate method for handling uncertainty in measurement and its
propagation.

Generally the number of significant figures to which a measurement should be reported includes
all of those digits which are certain and only one digit, the last, which is uncertain. Thus, for
example, if one were to measure the width of a page of paper using a 30-cm ruler graduated to
the nearest 0.1 cm, one might find that it measures somewhere between 21.6 cm and 21.7 cm.
Furthermore, one may estimate that this width lies approximately halfway between the two and
therefore record the width as 21.65 cm. In this instance, the last 0.05 cm is uncertain.

Significant Figures in Calculations


When numbers are multiplied (or divided), the error in the accuracy of these numbers is also
multiplied. The product of a multiplication typically has more numbers in it than are really
significant. The following rules are used to determine the number of significant digits following
various mathematical operations. When “truncating” large numbers to their proper number of
significant figures, the rules for rounding off (next section) should be observed.

Multiplication and Division:


Under multiplication and division, the final answer must have the fewest number of significant
figures among the numbers involved in the calculation.
e.g. 24.015 x 6.50 = 156
5 sig. figs. 3 sig. figs. 3 sig. figs.

Addition and Subtraction


Under addition and subtraction, the final answer must have the fewest number of decimal places
among the numbers involved in the calculation.
e.g. 2.4015 ⋅ 10-6 + 6.50 ⋅ 10-4 = ( 0.024015 + 6.50 ) ⋅ 10
-4

6 decimal places 2 decimal places


-4
= 6.524015 ⋅ 10
-4
= 6.52 ⋅ 10
Final answer has only 2 decimal places
Note: Conversion factors are regarded as infinitely precise, therefore should never be considered
when the number of significant figures in the final answer is determined.

Yann Brouillette - General Chemistry 202-NYA-05: Chapter 1 Review 6


The following set of rules is helpful in determining which of the digits in a number are significant.

Determining the number of significant figures

1. Non-zero Digits
All non-zero digits (integers) in a number are significant. For example, 4321 has four non-zero
integers, all of which count as significant figures.

2. Zeros
There are three classes of zeroes to be considered
a) Leading Zeros (zeros that precede all of non-zero digits) – never significant
e.g. 0001234 – 4 significant figures, one for each non-zero digit.
0.001234 – 4 significant figures, one for each non-zero digit.

b) Trapped Zeros (captive zeros between non-zero digits) – always significant


e.g. 1001 – 4 significant figures; the zeroes are trapped and are therefore significant
e.g. 4.005 – 4 significant figures; the zeroes are trapped and are therefore significant

c) Trailing Zeros (zeros at the right end of the number) – may or may not be significant
If a decimal point exists somewhere in the number, the trailing zeroes are significant.
e.g. 100 – 1 significant figures; no decimal points exists.
e.g. 100. – 3 significant figures; a decimal points exists.
e.g. 10.0 – 3 significant figures; a decimal points exists.
0.010200 – 5 significant figures; the leading zeroes are not significant, the trapped
zero is and since there is a decimal point in the number, the trailing
zeroes are significant.
If there is no decimal point anywhere in the number, the situation is ambiguous.
e.g. 100 – Ambiguous. The only way to properly express the correct number of
significant figures in this situation is to express the number in scientific
notation
2
1⋅10 – 1 significant figure
2
1.0⋅10 – 2 significant figures
2
1.00⋅10 – 3 significant figures
100. – 3 significant figures

Yann Brouillette - General Chemistry 202-NYA-05: Chapter 1 Review 7


Rounding Off
In calculations, the number obtained usually has many more digits than would correctly express
the precision to which the original measurements were made. Thus a calculated number must be
reduced to the proper number of significant figures; this process is known as rounding off. The
following set of rules represent one convention for rounding off numbers to the proper number of
significant figures.

First determine the correct number of significant figures to which the calculated number is to be
rounded off. Inspect the digit immediately to the right of the last significant digit.
If the digit to be removed is less than 5, the preceding digit stays the same
Ignore the digit.
e.g. 0.0257539 → 0.02575

If this digit is greater than 5


Increase the last significant digit by one.
e.g. 0.02999793 → 0.03000

If this digit is equal to 5


If the last significant digit is even, ignore it, otherwise increase it by one.
e.g. 0.0299459 → 0.02994
0.02997592 → 0.02998

Yann Brouillette - General Chemistry 202-NYA-05: Chapter 1 Review 8


Exercises:
1. How many significant digits are present in the following numbers?
102 3 significant figures
000 000 034 89 4 significant figures
2500 2 significant figures
0.000 3004 4 significant figures
0.00 520700 6 significant figures

2. Round off the following numbers to 3 significant figures.


435 435
4
76145 76100 = 7.61 x 10
4
22752 22800 = 2.28 x 10
4
9997 1.00 x 10
3
2500 2.50 x 10
4
46459 4.64 x 10

3. Perform the following calculations (In the examples that follow, the last appropriate
significant digit is underlined.)
149.2 + 0.034 + 2000.34 = 2149.574 = 2149.6
3 3 3 3
1.0322 x 10 + 4.34 x 10 = 5.3722 x 10 = 5.37 x 10
–2 –3 –2 –2
4.03 x 10 – 2.44 x 10 = 3.786 x 10 = 3.79 x 10
5 6 6 5
2.094 x 10 – 1.073 x 10 = –8.636 x 10 = –8.636 x 10
8 7 7
(0.0432)(2.909)(4.43 x 10 ) = 5.56712784 x 10 = 5.57 x 10
4 2 1 4 2
(2.9932 x 10 )(2.4443 x 10 + 1.0032 x 10 ) = (2.9932 x 10 )(2.54462 x 10 )
4 2
= (2.9932 x 10 )(2.5446 x 10 )
6
= 7.6166 x 10
2 –1 2 3
(2.34 x 10 + 2.443 x 10 ) / 0.0323 = (2.34 x 10 ) / 0.0323 = 7.24 x 10
–3 2 –5 –5
(4.38 x 10 ) = 1.91844 x 10 = 1.92 x 10
–6 ½ –3 –3
(5.9938 x 10 ) = 2.448223846 x 10 = 2.4482 x 10

Suggested Textbook Questions and Problems, Zumdahl/ Zumdahl 8th edition:


Chapter 1
30, 32, 36, 40, 42, 48, 60.

Yann Brouillette - General Chemistry 202-NYA-05: Chapter 1 Review 9


Extra Problems :
1. The number 0.005823 expressed in scientific notation is
A) 5.82 × 103 D) 5.823 × 10–3
B) 5.823 × 103 E) 5823 × 10–6
–3
C) 5.82 × 10
2. Express the number 0.00342 in scientific notation.
A) 3.42 × 10–3 D) 342 × 10–5
3
B) 3.42 × 10 E) none of these
C) 0.342 × 10–3
3. 6.8 kilograms contain this many grams.
A) 6.8 × 103 B) 6.8 × 102 C) 6.8 × 101 D) 6.8 × 10–2 E) 6.8 × 10–3
4. How many milliliters are in 0.070 L?
A) 0.70 mL D) 7.0 × 101 mL
B) 7.0 mL E) 7.0 × 103 mL
2
C) 7.0 × 10 mL
5. The SI prefix that corresponds to a factor of 10-3 is
A) kilo B) deci C) centi D) milli E) none of these
6. Which metric prefix is used to designate 1000?
A) m B) M C) k D) c E) d
7. The number of cubic centimeters (cm3) in 27.8 mL is
A) 0.0278 cm3 B) 2.78 cm3 C) 27.8 cm3 D) none of these
8. Using the rules of significant figures, calculate the following:
6.167 + 70 =
A) 76 B) 80 C) 76.167 D) 77 E) 76.17
9. Using the rules of significant figures, calculate the following:
14.8903 – 2.14 =
A) 12.7503 B) 12.75 C) 12.750 D) 12 E) 13
10. How many significant figures are in the number 34.00500?
A) 3 B) 4 C) 5 D) 6 E) 7
11. How many significant figures are in the number 1.20 × 103?
A) 1 B) 2 C) 3 D) 4 E) 5
12. In the sum of 54.34 + 45.66, the number of significant figures is
A) 2 B) 3 C) 4 D) 5 E) 6
13. A student finds that the weight of an empty beaker is 12.024 g. She places a solid in the
beaker to give a combined mass of 12.108 g. To how many significant figures is the
mass of the solid known?
A) 1 B) 2 C) 3 D) 4 E) 5
14. What is the result of the following multiplication expressed in scientific notation to the
correct number of significant figures?
(2.08 × 105)(7.0 × 10-5) =
A) 1.4 × 10-1 B) 1.456 × 101 C) 1.4 × 101 D) 1 × 101 E) 1.46 × 101
15. Convert 745.6 qt to milliliters (1 L = 1.060 qt).
A) 7.456 × 105 mL D) 703.4 mL
5
B) 7.034 × 10 mL E) none of these
C) 7.903 × 105 mL
Ans: 1)D 2)A 3)A 4)D 5)D 6)C 7)C 8)B 9)B 10)E 11)C 12)D 13)B 14)C 15)B

Yann Brouillette - General Chemistry 202-NYA-05: Chapter 1 Review 10


Chapter 2:

Atoms, Molecules and Ions

th
Suggested problems in chapter 2 of Zumdahl 8 Edition:

47, 49, 51, 53, 55, 57, 61, 63, 65, 67, 71, 73, 75, 77, 86, 87, 91.

Yann Brouillette - General Chemistry 202-NYA-05: Chapter 2 11


2.1 The Early History of Chemistry
• The Greeks were the first to try to explain why chemical changes occur. By about 400 B.C. they
proposed that all matter was composed of 4 fundamental substances: fire, water, earth and air. They
also considered the question of wheter matter is continuous, and thus infinitely divisible into smaller
pieces, or composed of small, indivisible particles. Supporters of the latter position were Democritus,
who used the term atomos.

• The next 2000 years were dominated by a pseudoscience called alchemy. They were mainly trying to
turn cheap metals into gold.

• Robert Boyle (1627-1691) was the first “chemist” to perform truly quantitative experiments, and
carefully measured the relationship between the pressure and volume of air. When he published his
book The Skeptical Chymist in 1661, the quantitative sciences of physics and chemistry were born.
As Boyle’s experimental definition of an element became generally accepted, the list of known
elements began to grow, and the Greek system of four elements finally died.

2.2 Fundemental Chemical Laws


• Antoine Lavoisier (1743-1794) finally explained the true nature of combustion. He stated the Law of
conservation of mass: In a chemical reaction, mass is neither created nor destroyed.

• Joseph Proust (1754-1826) stated the Law of definite proportion: A given compound always
contains exactly the same proportion of elements by mass.

• John Dalton (1766-1844) stated the Law of multiple proportions: When two elements form a series
of compounds, the ratios of the masses of the second element that combine with 1 gram of the first
element can always be reduced to small whole numbers.

2.3 Dalton’s Atomic Theory (1808)


• Each element is made up of tiny particles called atoms.

• The atoms of a given element are identical; the atoms of different elements are different in some
fundamental way or ways.

• Chemical compounds are formed when atoms of different elements combine with each other. A given
compound always has the same relative numbers and types of atoms.

• Chemical reactions involve reorganization of the atoms—changes in the way they are bound together.
The atoms themselves are not changed in a chemical reaction.

Using similar reasoning for other compounds, Dalton prepared the first table of atomic masses.

Yann Brouillette - General Chemistry 202-NYA-05: Chapter 2 12


In 1809, Joseph Gay-Lussac measured (under same conditions of Temperature and Pressure) the
volumes of gases that reacted with each other.

In 1811, Amadeo Avogadro’s proposed that at the same Temperature and Pressure, equal volumes of
different gases contain the same number of particles (the Avogadro’s hypothesis).

At the molecular level, the volume of gases, whether it be H2O or HCl, is the same. The spheres
represent atoms in the molecules.

2.4 Early Experiments to Characterize the Atom


• J.J. Thomson (1856-1940) postulated that an atom consisted of a diffuse cloud of positive charge
with the negative electrons embedded randomly in it (often called the plum pudding model).

• Ernest Rutherford (1871-1937) described the nuclear atom: An atom with a dense center of positive
charge (the nucleus) with electrons moving around the nucleus at a distance that is large relative to
the nuclear radius.

Yann Brouillette - General Chemistry 202-NYA-05: Chapter 2 13


2.5 The Modern View of Atomic Structure: An Introduction
• The simplest view of the atom is that it consists of a tiny nucleus and electrons that move about the
nucleus.
• The nucleus is assumed to contain protons, which have a positive charge equal in magnitude to the
electron’s negative charge.
• The nucleus is also assumed to contain neutrons, which have virtually the same mass as a proton,
but no charge.

A nuclear atom viewed in cross section.

It is worth noting here that the electrons are relatively far from the nucleus. If, for example, the nucleus is
blown up to the size of a ping-pong ball (40 mm), then the electron in the hydrogen atom is on the
average located approximately at Decarie Blvd.

The tiny nucleus accounts for almost all the atom’s mass.

In other words:
Chemistry is the study of matter and its transformations.
Matter is the material of which the universe is composed. For the purposes of chemistry, matter has two
generic properties:
1. It has mass
2. It occupies space (i.e., it has volume).
Matter can be found in two forms.
1. mixtures
2. pure substances
These can be further subdivided as follows:
1. Mixtures • Homogeneous Mixtures (which include solutions) which are uniform in
appearance and composition
• Heterogeneous Mixtures which are not uniform in appearance and
composition

Yann Brouillette - General Chemistry 202-NYA-05: Chapter 2 14


2. Pure Substances • Elements
There are 117 known elements of which 92 are naturally occurring
(the other 25 elements are man-made)
Each element is made up of smaller virtually identical particles
called atoms
• Compounds (molecules) are chemical combinations of atoms of different
elements

Ultimately then, matter is built up of atoms linked up in different ways. However, even atoms can be
subdivided into simpler particles:

1. Electrons (symbol: e ) are the fundamental unit of negative charge, electron charge = –1 (in the
Système International (SI) system of units, e = −1.6022 × 10 −19 C ).


2. Protons (symbol: p ) are the fundamental unit of positive charge, proton charge = +1, (in SI units,
e = +1.6022 × 10 −19 C ).

0
3. Neutrons (symbol: n ) are the fundamental particle having no charge.
These fundamental particles can be further reduced to even more fundamental particles, but this is
beyond the scope of normal chemical applications.
Schematically, the relationship between these forms of matter is depicted in the figure below.

Fundamental
Particles Pure
Atoms Mixtures
e–, p+, n0 Substances

Heterogeneous Homogeneous
Elements Compounds mixtures mixtures

The question is now: why is there so much variation in the properties of the different forms of matter when
all matter, from the perspective of chemistry, can be reduced to the same set of particles (the three types:
– – 0
e , p and n )?
th
The answer to this question is based on electrostatics and on advances made in the 20 century on the
structure of atoms (and molecules):
• atoms of the same element:
always have exactly the same number of protons and electrons. This number of protons
defines the element uniquely and is known as the atomic number (symbol: Z = the number
of protons in the atom).
may have different numbers of neutrons. Such atoms are known as isotopes. Each element
is made up of different isotopes in a fixed relative abundance.
• atoms of different elements always have a different number of protons (hence different values of
Z). In a free atom the number protons equals the number of electrons.

Yann Brouillette - General Chemistry 202-NYA-05: Chapter 2 15


Isotopes:
Since atoms have no net charge, the number of electrons must equal the number of protons. If we take
sodium for example, a sodium atom has 11 electrons moving around its nucleus. It is always true that a
sodium atom has 11 protons and 11 electrons. However, each sodium atom also has neutrons in its
nucleus, and different types of sodium atoms exist that have different numbers of neutrons.
The two atoms of sodium in Figure 2.15 below are isotopes, or atoms with the same number of protons
but different numbers of neutrons.

Figure 2.15: Two isotopes of sodium.


Note that the symbol for one particular type of sodium atom is written so the atomic number Z (number of
protons) is written as a subscript, and the mass number A (the total number of protons and neutrons) is
written as a superscript. Because the chemistry of an atom is due to its electrons, isotopes show almost
identical chemical properties.

2.6 Molecules and Ions

Atoms have electrons and these electrons participate in bonding one atom to another. The forces that
hold atoms together in compounds are called chemical bonds.

Covalent Bonding:
One way that atoms can form bonds is by sharing electrons. These bonds are called covalent bonds, and
the resulting collection of atoms is called a molecule.
In covalent bonds, the electrons are shared between two atoms. Covalent bonds are formed between two
nonmetal atoms

Molecule Representation
Chemical Formula:
Symbols for the elements are used to indicate the types of atoms present and subscripts are used to
indicate the relative numbers of atoms.
Example: CO2, H2O, CH4, NH3

Yann Brouillette - General Chemistry 202-NYA-05: Chapter 2 16


Structural Formula:
Individual bonds are shown between the atoms.
Example:
H
O N
O C O H H C H H
H H H
H
Ionic Bonding:
A second type of chemical bond results from attractions among ions. An ion is an atom or group of atoms
that has a net positive or negative charge.
A positive ion is called a cation. For example, sodium:
+
Na → Na + e−
An ion with a negative charge is called an anion. For example, chloride:
Cl + e− → Cl−
+
The ionic compound sodium chloride (Na Cl−) is commonly known as table salt.

Atoms acquire a charge only by gaining or losing electrons, NOT by gaining or losing protons. Note that
the total number of electrons is conserved. The electrons are simply jumping from one nucleus to
another.
Substances may lose or gain more than one electron.
e.g. Fe → Fe3+ + 3 e–
O + 2 e– → O2–
This being the case, it is necessary to be able to determine the particular charge that an atom may
acquire.

Example
The free atom 3Li, which is a Group IA atom, has 3 e–. To achieve stability, it must arrive at the number of
electrons of one of the inert gases (i.e., 2, 10, 18, 36, 54). The nearest number of electrons is either 2 or
10. Thus, the Li atom can either lose one e– or it can gain seven e–. Clearly the minimum fuss will be for Li
to lose the one e–.

Li – e → Li+
or, writing this more conventionally:

Li → Li+ + e

Example
The free atom 20Ca, which is a Group IIA atom, has 10 e–. To achieve stability, it must arrive at the number
of electrons of a one of the inert gases (i.e., 2, 10, 18, 36, 54). The nearest number of electrons is either 18
or 36. Thus, the Ca atom can either lose two e– or it can gain 16 e–. Clearly the minimum fuss will be for Ca
to lose the two e–.

Ca – 2 e → Ca2+
or, writing this more conventionally:

Ca → Ca2+ + 2 e

Yann Brouillette - General Chemistry 202-NYA-05: Chapter 2 17


Example
The free atom 17Cl, which is a Group VIIA atom, has 17 e–. To achieve stability, it must arrive at the
number of electrons of a one of the inert gases (i.e., 2, 10, 18, 36, 54). The nearest number of electrons is
either 10 or 18. Thus, the Cl atom can either gain one e– or it can lose seven e–. Clearly the minimum fuss
will be for Cl to gain the one e–.

Cl + e → Cl–

Typically:
Metals will lose electrons thereby forming cations.
Non-metals tend to gain electrons thereby forming anions.

The reason that metals tend to lose electrons is that they are to the left of the Periodic Table – it is
simpler for them to lose electrons thereby acquiring the electron count to the noble gas of the preceding
row. Thus, for example, it is easier for a metal such as Li to lose one electron to attain the electron count

of He (2) than to gain 7 e to acquire the electron count of Ne (10). Likewise a nonmetal such as 9F will
– –
prefer to gain one e to attain the Ne electron count (10) than to lose 7 e to get to He (2).

In general, substances will not give up electrons unless there are other atoms ready to receive those
electrons. Likewise, a substance cannot accept electrons without other substances ready to supply those
electrons. Thus, for example, sodium can lose an electron:
Na → Na+ + e–
but will not do so unless it is in the presence of another atom willing to receive that electron, such as
chlorine:
Cl + e– → Cl–
Na → Na+ + e–
Cl + e– → Cl–
Na + Cl → Na+ + Cl–

Note that there are no net free electrons remaining at the end. All of the electrons lost by Na atoms have
been absorbed by the Cl atoms. At this point the system consists of a cation (Na+) and an anion (Cl–).
These two oppositely charged ions now attract each other to form an ionic bond.
Na+ + Cl– → Na+ Cl–

The attraction of positive to negative results in the formation of an ionic bond. The resulting neutrally
charged compound is known as an ionic compound. Ionic compounds are crystalline solids.
Ionic compounds can always be recognised because they have metal atoms.

Yann Brouillette - General Chemistry 202-NYA-05: Chapter 2 18


FORMULAS OF IONIC COMPOUNDS
A balance of charges between the cation and anion results in a neutral compound. This fact can be used
to predict the simplest formula of an ionic compound.
20Ca → Ca2+ + 2e– (Ca loses two e– to acquire the same number of electrons as Ar)
2 × {Cl + e– → Cl–} (Cl loses only one e– to acquire the same number of electrons as Ar)
therefore two atoms of Cl are needed to absorb the two electrons that Ca needs to give up.
Ca2+ + 2 Cl– → CaCl2

In general, once the charges of the cation and the anion are known, it is a simple matter to predict the
formula of the simplest ionic compound that can be formed between them. If M is a metal (which forms a
cation having a charge n+) and X is a non-metal (which forms an anion having a charge m–) then the
ionic compound is: n+ m-
M X MmXn

Charge of X Charge of M

m × (Mn+) + n × (Xm+) = 0

Note that the charges of M and X in the formula are reduced to their simplest proportions (e.g. for the ion
pair M6+ and X4– the formula is M2X3 not M4X6).

Examples: Determine the formulas of the compounds from the following ion pairs.
+ –
1. K and Cl : KCl

2. Co2+ and F–: CoF2

3. Co2+ and F–: CoF2

4. Fe2+ and S2– FeS (N.B., the ionic compounds are usually represented in their simplest proportions)

5. Fe3+ and O2– Fe2O3

2.7 An Introduction to the Periodic Table

The names and symbols of many elements must be memorized. These elements are highlighted in grey
on the Periodic Table provided below.

Yann Brouillette - General Chemistry 202-NYA-05: Chapter 2 19


PERIODIC TABLE OF THE ELEMENTS
1 2
1.0079 4.003
H He
hydrogen helium
hydrogène hélium
3 4 atomic number 5 6 7 8 9 10
6.941 9.012 atomic mass 10.811 12.011 14.007 15.9994 18.998 20.18
Li Be Symbol B C N O F Ne
lithium beryllium English name boron carbon nitrogen oxygen fluorine neon
lithium béryllium French name* bore carbone azote oxygène fluor néon
11 12 *all are masculine 13 14 15 16 17 18
22.99 24.31 26.98 28.086 30.974 32.07 35.453 39.95
Na Mg Al Si P S Cl Ar
sodium magnesium aluminum silicon phosphorus sulfur chlorine argon
sodium magnésium aluminium silicium phosphore soufre chlore argon
19 20 21 22 23 24 25 26 27 28 29 30 31 32 33 34 35 36
39.098 40.08 44.96 47.87 50.94 51.996 54.94 55.85 58.93 58.69 63.546 65.39 69.72 72.61 74.922 78.96 79.904 83.80
K Ca Sc Ti V Cr Mn Fe Co Ni Cu Zn Ga Ge As Se Br Kr
potassium calcium scandium titanium vanadium chromium manganese iron cobalt nickel copper zinc gallium germanium arsenic selenium bromine krypton
potassium calcium scandium titane vanadium chrome manganèse fer cobalt nickel cuivre zinc gallium germanium arsenic sélénium brome krypton
37 38 39 40 41 42 43 44 45 46 47 48 49 50 51 52 53 54
85.468 87.62 88.91 91.22 92.91 95.94 98 101.07 102.91 106.42 107.87 112.411 114.82 118.71 121.76 127.60 126.904 131.29
Tc
Rb Sr Y Zr Nb Mo technetium Ru Rh Pd Ag Cd In Sn Sb Te I Xe
rubidium strontium yttrium zirconium niobium molybdenum technétium ruthenium rhodium palladium silver cadmium indium tin antimony tellurium iodine xenon
rubidium strontium yttrium zirconium niobium molybdène ruthénium rhodium palladium argent cadmium indium étain antimoine tellure iode xénon
55 56 57 72 73 74 75 76 77 78 79 80 81 82 83 84 85 86
132.91 137.33 138.91 178.49 180.95 183.84 186.21 190.21 192.22 195.08 196.97 200.59 204.38 207.2 208.98 209 210 222

Cs Ba La Hf Ta W Re Os Ir Pt Au Hg Tl Pb Bi Po At Rn
cesium barium lanthanum hafnium tantalum tungsten rhenium osmium iridium platinum gold mercury thallium lead bismuth polonium astatine radon
césium baryum lanthane hafnium tantale tungstène rhénium osmium iridium platine or mercure thallium plomb bismuth polonium astate radon
87 88 89 104 105 106 107 108 109 110 111 112
223 226.03 227.03 261 262 266 264 269 268 269 272 277

Fr Ra Ac Rf Unp Unh Uns Uno Une Uun Uuu Uub


francium radium actinium rutherfordium unnilpentium unnilhexium unnilseptium unnilloctium unnilennium ununnilium unununium ununbium
francium radium actinium rutherfordium unnilpentium unnilhexium unnilseptium unnilennium unnilennium ununnilium unununium ununbium

58 59 60 61 62 63 64 65 66 67 68 69 70 71
140.12 140.91 144.24 145 150.36 151.96 157.25 158.93 162.50 164.93 167.26 168.93 173.04 174.97

Ce Pr Nd Pm Sm Eu Gd Tb Dy Ho Er Tm Yb Lu
cerium praseodym- neodymium promethium samarium europium gadolinium terbium dysprosium holmium erbium thulium ytterbium lutetium
cérium ium néodyme prométhium samarium europium gadolinium terbium dysprosium holmium erbium thulium ytterbium lutécium
praséodyme
90 91 92 93 94 95 96 97 98 99 100 101 102 103
232.038 231.036 238.029 237.048 244 243 247 247 251 252 257 258 259 262

Th Pa U Np Pu Am Cm Bk Cf Es Fm Md No Lr
thorium protactinium uranium neptunium plutonium americium curium berkelium californium einsteinium fermium mendelevium nobelium lawrencium
thorium protactinium uranium neptunium plutonium américium curium berkélium californium einsteinium fermium mendélévium nobélium lawrencium

Yann Brouillette - General Chemistry 202-NYA-05: Chapter 2 20


Classification of the Elements
Atoms of the elements are organized in the Periodic Table according to property in the sequence of their
atomic numbers, Z (the number of protons in the atom).
The letters in the boxes are the symbols for the elements.
The numbers shown above each symbol is the atomic number Z.
There are several ways in which the elements are classified and it is necessary to know them.

METALS AND NONMETALS


The vast majority of elements are metals. Chemically, metals tend to lose electrons to form positive ions.
Metals have the following properties:
• Generally metals are crystalline solids at room temperature and pressure (mercury, Hg, is
the exception)
• Lustrous
• Malleable
• Ductile
• Good conductors of electricity and heat
Alkaline-Earth metals
Alkali metals

Chalcogens
Pnictogens

Halogens
B

Transition metals
Al Si

Ge As

Metals Sb Te

Po At

Non-metals are to the right of the stepladder structure (bold lines). Chemically, they tend to gain
electrons in reactions with metals to form negative ions. They often bond to each other by forming
covalent bonds.
Metalloids (semi-metals) are at the boundary of the stepladder and are indicated on the Periodic Table
above by the lighter shade of grey. They have many properties typical of the metals but are generally
semi-conductors rather than the excellent conductors of electricity and heat that metals are.

Yann Brouillette - General Chemistry 202-NYA-05: Chapter 2 21


GROUP NAMES
Some of the groups of elements (i.e., the columns of elements in the Periodic Table) have specialized
names. These have to be learned.
Group IA (Li, Na, K, Rb, Cs): Alkali Metals
Group IIA (Be, Mg, Ca, Sr, Ba, Ra): Alkaline-Earth Metals
Group B Transition Metals
Group VA (N, P, As, Sb, Bi): sometimes (rarely) known as the Pnictogens
Group VIA (O, S, Se, Te, Po): sometimes referred to as the Chalcogens
Group VIIA (F, Cl, Br, I, At): Halogens
Group VIIIA (He, Ne, Ar, Kr, Xe, Rn): Noble Gases (or Inert Gases)

Yann Brouillette - General Chemistry 202-NYA-05: Chapter 2 22


2.8 Naming Simple Compounds
The system for naming inorganic binary compounds is divided in different types for easier
recognition.
• Binary Ionic Compounds (Type I)
• Binary Ionic Compounds (Type II)
• Ionic Compounds with Polyatomic Ions
• Binary Covalent Compounds (Type III)
• Acids

Binary Ionic Compounds (Type I)


Binary ionic compounds contain a positive ion (cation) always written first in the formula and a
negative ion (anion). In naming Type I Binary Compounds, the following rules apply:

1. The cation is always named first and the anion second.


2. A monoatomic (meaning “one-atom”) cation takes its name from the name of the
+
element. For example, Na is called sodium in the names of compounds containing this
ion.
3. A monoatomic anion is named by taking the root of the element name and adding –ide.
Thus the Cl− ion is called chloride.

Example:
LiCl: Lithium chloride KBr: Potassium bromide MgCl2: Magnesium chloride

Binary Ionic Compounds (Type II)


While the metal present in binary ionic compounds of type I only formed a single type of cation
+ 2+
(sodium is only Na , calcium is only Ca , etc.) the metal present in binary ionic compound of type
2+ 4+
II form more than one positive ion (lead can be Pb and Pb ).
2+ 3+
For example, the compound FeCl2 contains Fe ions, and the compound FeCl3 contains Fe
ions. In that case, the charge on the metal ion must be specified. The systematic names for these
two iron compounds are iron(II) chloride and iron (III) chloride, respectively, where the Roman
numeral indicates the charge of the cation.

General Chemistry 202-NYA-05: Chapter 2 23


* Note that mercury(I) ions always occur bound
2+
together to form Hg ions.

Although these are transition metals, they form
only one type of ion, and a Roman numeral is not
used.

Note that the use of a Roman numeral in a systematic name is required only in cases where more
than one ionic compound forms between a given pair of elements. Elements that form only one
cation do not need to be identified by a Roman numeral.

Common metals that do not require Roman numerals are the Group 1A elements, which form
only 1+ ions; and the Group 2A elements, which form only 2+ ions. Aluminium, silver and zinc do
3+ + 2+
not require a Roman number, because they most commonly form only the Al , Ag and Zn ions
respectively.

Example:
CoBr2: Cobalt(II) bromide Al2O3: Aluminium oxide CrCl3: Chromium(III) chloride

Common cations and anions:

General Chemistry 202-NYA-05: Chapter 2 24


Ionic Compounds with Polyatomic Ions
Ionic compounds formed of a group of atomic ions. (For example, ammonium nitrate, NH4NO3
+
contains the polyatomic ions NH4 and NO3−). There names must be memorized. The commonly
used ones are summarized in this table:

Several series of anions contain an atom of a given element and different numbers of oxygen
atoms. These anions are called oxyanions. When there are two members in such a series, the
name of the one with the smaller number of oxygen atoms ends in –ite and the name of the one
with the larger number ends in –ate. For example, nitrite (NO2−) and nitrate (NO3−).

When more than two oxyanions make up a series, hypo- (less than) and per- (more than) are
used as prefixes to name the members of the series with the fewest and the most oxygen atoms,
respectively. The best example involves the oxyanions containing bromine: BrO− (Hypobromite),
BrO2− (Bromite), BrO3− (Bromate) and BrO4− (Perbromate).

Example:
Na2SO4: Sodium sulfate Na2SO3: Sodium sulfite Fe(NO3)3: Iron(III) nitrate

Binary Covalent Compounds (Type III)


Binary covalent compounds are formed between two nonmetals, and do not contain ions. They
are named in a similar manner than the ionic compounds, according to the following rules:

The first element in the formula is named first, using the full element name.
The second element is named as if it were an anion.
Prefixes are used to denote the numbers of atoms present.
The prefix mono- is never used for naming the first element. For example, CO is called carbon
monoxide, not monocarbon monoxide.

To avoid awkward pronunciations, the final o or a of the prefix is dropped when the element
begins with a vowel. For example, N2O4 is called Dinitrogen tetrOxide.

General Chemistry 202-NYA-05: Chapter 2 25


Example

PCl3 : Phosphorus trichloride

PCl5 : Phosphorus pentachloride

P4O10 : Tetraphosphorus decaoxide

Acids (an introduction)


When dissolved in water, acids produce a solution containing free H+ ions. The rules for naming
acids depend on whether the anion contains oxygen.

When the anion does not contain oxygen:


If the name of the anion ends in –ide, the acid is named with the prefix hydro- and the suffix –ic.
For example, HCl is Hydrochloric acid.

When the anion does contain oxygen:


The acidic name is formed from the root name of the anion with a suffix of –ic or –ous, depending
on the name of the anion.
If the anion name ends in –ate, the suffix –ic is added to the root name. For example, H2SO4
2−
contains the sulfate anion (SO4 ) and is called sulphuric acid.
If the anion name ends in –ite, the suffix –ous is added to the root name. For example, H2SO3
2−
contains the sulfite anion (SO3 ) and is called sulfurous acid.

General Chemistry 202-NYA-05: Chapter 2 26


Simple Nomenclature In Other Words
Nomenclature is a system of naming compounds so that:
• given the name of the compound, write the formula of the compound; or
• given the formula of the compound, write the name.

Unique
correspondence
Formula Name

Before giving the system for naming compounds, it is necessary to know something about the
nature of the compound.

1.1.1 BINARY IONIC COMPOUNDS


The term binary compound means that the compound consists only of two different types of
atoms.
Exercise: State whether the following compounds are binary.
1. CO
2. CO2
3. C2H2Cl2
4. CH3CH3
5. HCN

Binary ionic compounds are formed from single atom (metal) cations and single atom (non-metal)
anions.

Binary ionic compound = metal + non-


metal

Whenever one of the atoms is a metal the compound is an ionic compound.


Review:
You must be able to
recognize a metal from its
location in the Periodic Table
e.g. Which of the following compounds are binary ionic compounds?
CO2
CaO
CaCO3
FeCl3
NaNO2
CCl4

General Chemistry 202-NYA-05: Chapter 2 27


NAMING SIMPLE BINARY IONIC COMPOUNDS
First the compound must be identified as:
Ionic
binary

metal + non-metal + ide

The suffix –ide is very important. It is


the signature of binary compounds.

Root name of the nommetal

Care must be taken to pay attention to the suffix, with only two exceptions, only binary
compounds (and more particularly single atom anions) may use the suffix –ide.
Example:
NaCl sodium chloride (binary and ionic)
NaClO2 sodium chlorite (not binary but ionic)

Examples

1. NaBr binary and Ionic: sodium bromide

2. CaCl2 binary and Ionic: calcium chloride

3. CCl4 binary but not ionic (no metal atom) rule does not apply

4. BaO binary and Ionic: barium oxide

5. Li3N binary and Ionic: lithium nitride

6. NaNO3 Not binary (more than two atoms of different elements): rule does not apply

The rules for naming simple binary ionic compounds are straightforward. It is only necessary to
establish that the compound is binary and ionic.
The problem is that most binary ionic compounds cannot be named in this fashion uniquely
because most metal atoms can show several charges in different compounds. For example, the
combination of iron and oxygen atoms can give two possible compounds:
FeO black coloured powder Fe2O3 rust
According to the rules learned in the previous section, the name of both of these compounds
would be iron oxide. This is not adequate however, because the name would not give us a unique
chemical formula.

General Chemistry 202-NYA-05: Chapter 2 28


The nomenclature rule for these types of compounds is as follows:

GENERAL RULE FOR THE NOMENCLATURE OF ALL BINARY IONIC


COMPOUNDS
 Charge of 
Metal   anion −ide

 metal 
 

• Expressed as Roman numerals (in brackets).


• Do not put this in if the metal can have only
one charge (oxidation state).

Thus in the examples cited above:


FeO would be named iron (II) oxide.
Fe2O3 would be named iron (III) oxide.
The difficulty with applying this system of nomenclature is that you have to know or calculate the
charge of the metal atom.
The charges carried by atoms in a compound are known as oxidation numbers (or oxidation states).

RULES FOR ASSIGNING OXIDATION STATES TO ATOMS


These are to be memorised in the sequence provided.
1. The sum of the oxidation numbers of all atoms in the chemical must add up to the overall
charge.
2. Group IA metals: Li, Na, K, Rb, Cs will always form cations having (+1) charge.
3. Group IIA elements: Be, Mg, Ca, Sr, Ba will always have (+2) charge.
4. Group IIIA elements: Al, Ga, In will always have (+3) charge.
5. Ag usually has (+1)
Al3+
Zn always has (+2)
Cd always has (+2) Zn2+ Ga3+
Ag(1+) Cd2+ In3+

6. H will usually have (+1) charge. The exception is when H is bonded to a metal; in this case
it will have an oxidation number of (–1).
7. Oxygen in compounds have (–2) except for peroxides (–1) or superoxides (–½).
8. When necessary, the main group nonmetals (i.e., Groups IIIA – VIIA) will have a charge
associated with the group number. Thus:
• Group VA elements (Nitrogen group) will have (–3).
• Group VIA elements (Oxygen group) will have (–2).
• Group VIIA elements (Halogens) will have (–1).
These assignments of charge are to be made only when the main group nonmetals are
bonded to atoms of the elements not listed above.
9. The oxidation number of all other atoms must be calculated.

Once the oxidation states of all atoms in the compound have been calculated, you can use them
to name the compound as for the nomenclature rule.

General Chemistry 202-NYA-05: Chapter 2 29


SUMMARY

+1

H +2 +3 (–3) (–2) (–1)

B N O F

Al P S Cl

Zn2+ Ga As Se Br

Ag(+) Cd2+ In Sb Te I

Po At

H (in the shaded box) can have (–1) but only when it is bonded to a metal.
The charges placd in brackets indicates the most common oxidation states but these values
should be used only if necessary. Attempt to calculate the charges of these atoms first if possible.

e.g., Write the name of each of the following compounds.


1. Cr2O3
a. Identify the compound as binary/ionic/covalent: it is a binary, ionic compound.
b. Determine the oxidation state of each element in the compound
• Use Rule 1 to set up an equation
2(Cr) + 3 (O) = 0
• This is an equation with two unknowns. You must choose a value of oxidation
number for one of the atoms. The priority is given to Group A atoms. Choose
oxygen according to its group number.
(O) = –2
• Substitute this into the equation above.
2(Cr) + 3 (–2) = 0
2(Cr) = +6
(Cr) = +3
c. Write the name of the compound. It is a binary, ionic compound and Cr may have
multiple oxidation states so its charge must be included in the name.

Cr2O3: chromium (III) oxide

General Chemistry 202-NYA-05: Chapter 2 30


2. CoBr6
a. Identify the compound as binary/ionic/covalent: it is a binary, ionic compound
b. Determine the oxidation state of each element in the compound
• Use Rule 1 to set up an equation
2(Co) + 6 (Br) = 0
• This is an equation with two unknowns. You must choose a value of oxidation
number for one of the atoms. The priority is given to Group A atoms. Choose
bromine according to its group number.
(Br) = –1
• Substitute this into the equation above.
(Co) + 6 (–1) = 0
(Co) = +6
c. Write the name of the compound. It is a binary, ionic compound and Co may have
multiple oxidation states so its charge must be included in the name.
CoBr6: cobalt (VI) bromide

3. SnS2
a. Identify the compound as binary/ionic/covalent: it is a binary, ionic compound
b. Determine the oxidation state of each element in the compound
• Use Rule 1 to set up an equation
(Sn) + 2 (S) = 0
• This is an equation with two unknowns. You must choose a value of oxidation
number for one of the atoms. The priority is given to Group A atoms. Choose
sulphur according to its group number.
(S) = –2
• Substitute this into the equation above.
(Sn) + 2 (–2) = 0
(Sn) = +4
c. Write the name of the compound. It is a binary, ionic compound. Sn may have multiple
oxidation states so its charge must be included in the name.

SnS2: tin (IV) sulphide

FORMULAS FROM THE NAMES


A functional system for naming compounds requires that the reverse also be true. That is, given
the name of the compound, we must be able to unambiguously write the chemical formula.
Name Formula
There are two things to look out for:
1. Is it binary? look for the suffix –ide.
2. Is it ionic? see if there is a metal.
To put together the formula of a binary ionic compound you must:
1. Identify the charge of the cation and the anion.
2. Multiply each cation and each anion by small integers so that the total charge supplied by
all the cations equal the total charge supplied by all the anions.

General Chemistry 202-NYA-05: Chapter 2 31


e.g., Write the formula for each of the following compounds
1. Iron (III) chloride
a. Identify the type of compound
This is ionic – iron is a metal.
It is binary – it has the suffix –ide.
b. Identify the charges of the ions:
Iron (III) means: (Fe) = +3
Assign (Cl) = –1 on the basis that it is a main group non-metal.
c. Combine the two types of atoms so there is a net charge of zero
Fe3+ Cl1–
d. Write the formula: FeCl3

2. Manganese (V) oxide


a. Identify the type of compound
This is ionic – manganese is a metal.
It is binary – it has the suffix –ide.
b. Identify the charges of the ions:
Manganese (V) means: (Mn) = +5
Assign (O) = –2 on the basis that it is a main group non-metal.
c. Combine the two types of atoms so there is a net charge of zero
Mn5+ O2–
d. Write the formula: Mn2O5

3. Mercury (II) nitride


a. Identify the type of compound
This is ionic – mercury is a metal.
It is binary – it has the suffix –ide.
b. Identify the charges of the ions:
Mercury (II) means: (Hg) = +2
Assign (N) = –3 on the basis that it is a main group non-metal.
c. Combine the two types of atoms so there is a net charge of zero
Hg2+ N3–
d. Write the formula: Hg3N2

4. Potassium iodide
a. Identify the type of compound
This is ionic – potassium is a metal.
It is binary – it has the suffix –ide.
b. Identify the charges of the ions:
K belongs to Group IA so it has a charge of +1
Assign (I) = –1 on the basis that it is a main group non-metal.
c. Combine the two types of atoms so there is a net charge of zero
K+ I–
d. Write the formula: KI

General Chemistry 202-NYA-05: Chapter 2 32


5. Calcium chloride
a. Identify the type of compound
This is ionic – calcium is a metal.
It is binary – it has the suffix –ide.
b. Identify the charges of the ions:
Ca belongs to Group IIA so it has a charge of +2
Assign (Cl) = –1 on the basis that it is a main group non-metal.
c. Combine the two types of atoms so there is a net charge of zero
Ca2+ Cl–
d. Write the formula: CaCl2

Exercise: Write the formula for each of the following compounds.


magnesium chloride
barium sulphide
aluminium selenide
rubidium oxide
strontium fluoride
calcium bromide
indium oxide

Exercise: Write the names of each of the following compounds


K2S
W2O5
CdO
ZnCl2
AgCl

General Chemistry 202-NYA-05: Chapter 2 33


1.1.2 BINARY COVALENT COMPOUNDS

Binary covalent can be recognised because they contain no metal atoms. For our purposes, only
non-metal atoms can form covalent bonds.
Binary covalent compounds consist of nonmetal atoms bonded to other nonmetal atoms

nonmetal + nonmetal

Binary covalent compounds are named quite differently from binary ionic compounds. In the case
of binary ionic compounds, once the oxidation numbers of all species are known, there is only
one way to put the compound together. In the case of binary covalent compounds, the situation is
completely different. The same pair of atoms may form several covalent compounds. The case of
the reactions of nitrogen (N2) with oxygen (O2) demonstrates this. Possible combinations of these
two elements include:
NO
N2O
NO2
N2O3
N2O4
N2O5
Since the products are not predictable when nonmetals react, we must use a different system of
nomenclature. We use numerical prefixes to state explicitly the number of each type of atom
present in the molecule.
Memorize the following list of numerical prefixes
Prefix Number
mono- 1
di- 2
tri- 3
tetra- 4
penta- 5
hexa- 6
hepta- 7
octa- 8
nona- 9
deca- 10

Nomenclature of binary covalent compounds: (prefix) nonmetal (prefix)nonmetal -ide

First nonmetal Signature of


is named. It is binary
the one compounds
Number of closest to the
first nonmetal metal side
atom (omit if it
is mono)

General Chemistry 202-NYA-05: Chapter 2 34


1. NO
a. Is it ionic or covalent? Is it binary? Yes to both.
b. Number of N-atoms = 1 → prefix = mono
Number of O-atoms = 1 → prefix = mono
c. Name the compound
Nitrogen is named first because it is closer than Oxygen to the metals
mononitrogen monoxide
nitrogen monoxide
2. N2O
a. Is it ionic or covalent? Is it binary? Yes to both.
b. Number of N-atoms = 2 → prefix = di
Number of O-atoms = 1 → prefix = mono
c. Name the compound
dinitrogen monoxide
3. NO2
a. Is it ionic or covalent? Is it binary? Yes to both.
b. Number of N-atoms = 1 → prefix = mono
Number of O-atoms = 2 → prefix = di
c. Name the compound
mononitrogen dioxide
nitrogen dioxide
4. N2O3
a. Is it ionic or covalent? Is it binary? Yes to both.
b. Number of N-atoms = 2 → prefix = di
Number of O-atoms = 3 → prefix = tri
c. Name the compound
dinitrogen trioxide
5. N2O4
a. Is it ionic or covalent? Is it binary? Yes to both.
b. Number of N-atoms = 2 → prefix = di
Number of O-atoms = 4 → prefix = tetra
c. Name the compound
dinitrogen tetroxide
6. N2O5
a. Is it ionic or covalent? Is it binary? Yes to both.
b. Number of N-atoms = 1 → prefix = mono
Number of O-atoms = 1 → prefix = mono
c. Name the compound
dinitrogen pentoxide

General Chemistry 202-NYA-05: Chapter 2 35


7. P4O6
a. Is it ionic or covalent? Is it binary? Yes to both.
b. Number of P-atoms = 4 → prefix = tetra
Number of O-atoms = 6 → prefix = hexa
c. Name the compound
Phosphorous is named first because it is closer than Oxygen to the metals
Tetraphosphorous hexoxide

1.1.3 POLYATOMIC IONS

NH 4+ Ammonium C2 H 3O2− Acetate

Hg 22+ Mercury (I) C2 O42− Oxalate

MnO4− Permanganate OH − Hydroxide

Cr2 O72− Dichromate CN − Cyanide

CrO42− Chromate SCN − Thiocyanate

SO42− Sulfate ClO4− Perchlorate

SO32− Sulfite ClO3− Chlorate

PO43− Phosphate ClO2− Chlorite

PO33− Phosphite ClO − Hypochlorite

NO3− Nitrate CO32− Carbonate

NO2− Nitrite

The formulas and names are put together using these ions precisely the same way as for the
binary ionic compounds.

Thus, to predict the formula of the ionic compound formed from:

1. Na+ and ClO3−


NaClO3 sodium chlorate

2. NH 4+ and Cl–
NH4Cl ammonium chloride

3. Mn5+ and CO32−


Mn2(CO3)5 manganese (V) carbonate

General Chemistry 202-NYA-05: Chapter 2 36


4. NH 4+ and Cr2 O72−
(NH4)2Cr2O7 ammonium dichromate

One procedure for memorizing Polyatomic Oxyanions of the nonmetals:

Mnemonic for Learning Oxyanions of the Nonmetals

Basic formula: XOnm−

Needed information: 1. Number of oxygens (n) – to get the formula of the anion
1. Charge of the anion (m–) – to get the formula of a complete ionic compound

–3 –2 –1 –1
B C N O F

P S
Cl

As Se Br

The oxyanions of these nonmetals having three


oxygens, XO3m−
has the suffix –ate
Borate BO3m−
The –ate anions Carbonate CO3m−
are PO43−and SO42−
respectively. The Nitrate NO3m−
other members ClO3m−
Chlorate
of the P and S
groups will follow
the trends for P
and for S.

One way to remember the value of the negative charge of the anions, m, is to count how far the
atom X’s column is from the O-columns. Thus, for example, since C is two (2) columns away from
O, the formulaCO
of32carbonate

is . Similiarly, since I is one (1) column away from O, the IO3−
formula of iodate is .

General Chemistry 202-NYA-05: Chapter 2 37


Different number
of oxygens, Suffixes
same charge

XOnm+−1 per–…–ate

XOnm− …–ate

XOnm−−1 …–ite

XOnm−−2 hypo–…–ite

e.g.
ClO4− perchlorate

ClO3− chlorate

ClO2− chlorite

ClO − hypochlorite

1.1.4 NAMING ACIDS


Acids (Arhennius definition) are substances which, when dissolved in water, produce H+ ions.
By implication then, acids are ionic compounds in which the cations are all protons, H+.
Nomenclature of acids is based on the nomenclature of the anion as there is no need to
reference the cation, which is always H+.

Corresponding Example
Anion
Acid Anion Corresponding Acid

ide hydroic acid Cl , chloride HCl, hydrochloric acid
ate ic acid ClO3− , chlorate HClO3 , chloric acid

ite ous acid ClO − , hypochlorite HClO, hypochlorous acid

Examples:
HI hydroiodic acid
HCN hydrocyanic acid
HIO4 periodic acid
HBrO2 bromous acid

General Chemistry 202-NYA-05: Chapter 2 38


Acid anions
For anions having multiple charges, there is no reason for the cations to be identical. It is
perfectly legitimate to have an ionic compound with a formula that combines several different
cations. For example, the combination of lithium and ammonium cations with phosphate can
have:
(NH4)Li2PO4 Ammonium dilithium phosphate
(NH4)2LiPO4 diammonium lithium phosphate
Note that the sequence in which the cations are named is alphabetical.
+
As you would expect, ionic compounds having one or more H cations, but not all, are special
cases. Thus for example, compounds such as:
Na2HPO4
NaH2PO4
and
Ba(HCO2)2
are possible. The nomenclature is based on the anion root.
Cation (Cation Charge) + (prefix for the number of hydrogen – omit mono) hydrogen + root
anion.
Thus
Na2HPO4 sodium hydrogen phosphate
NaH2PO4 sodium dihydrogen phosphate
Ba(HCO2)2 barium hydrogen carbonite
Note that although the name is based on the anion root, the anion in these hydrogenated anions
includes the hydrogens. Thus the following substances decompose as follow:
Na2 HPO4 → 2 Na + + HPO42− Anion = HPO42− not PO43−
NaH 2 PO4 → Na + + H 2 PO4− Anion = H 2 PO4− not PO43−

General Chemistry 202-NYA-05: Chapter 2 39


General Chemistry 202-NYA-05: Chapter 2 40
Chapter 3:

Stoichiometry

th
Suggested problems in chapter 3 of Zumdahl 8 Edition:

33, 41, 43, 46, 47, 49, 51, 57, 59, 61, 64, 65, 67, 71, 73, 75, 77, 79, 81, 83, 87, 93, 99, 101, 103,
105, 109, 111, 114.

General Chemistry 202-NYA-05: Chapter 3 41


Chemical stoichiometry: Quantities of material consumed and produced in chemical reactions.

3.1 Counting by Weighing


Objects do not need to have identical masses to be counted by weighing. We simply need to
know the average mass of the objects. For purpose of counting, the objects behave as though
they were all identical, as though they each actually had the average mass.

3.2 Atomic Masses


12
The modern system of atomic masses, instituted in 1961, is based on C (“carbon twelve”) as the
12
standard. In this system, C is assigned a mass of exactly 12 atomic mass units (amu), and the
masses of all other atoms are given relative to this standard.

The mass for each element is given in the periodic table of the elements. This value, even though
it is actually a mass, is (for historical reasons) sometimes called the atomic weight for each
element. The average mass of an element is called the average atomic mass (or simply the
atomic mass).

The atomic mass for carbon is not 12 but 12.01 because the carbon found on earth (natural
12 13 14
carbon) is a mixture of the isotopes C, C, C. Since natural carbon is composed of 98.89%
12 13
C atoms and 1.11% C atoms, the average atomic mass for carbon is calculated as follows:

98.89% of 12 amu + 1.11% of 13 amu = (0.9889)(12 amu) + (0.0111)(13 amu) = 12.01 amu

Even thought natural carbon does not contain a single atom with mass 12.01, for stoichiometric
purposes, we can consider carbon to be composed of only one type of atom with a mass of
12.01. This enables us to count atoms of natural carbon by weighing a sample of carbon.

3.3 The mole


12
Mole (abbreviated mol) is the number of carbon atoms in exactly 12 grams of pure C.
23
One mole of something consists of 6.022 x 10 units of that substance.
23
Avogadro’s number (in honor of Avogadro’s contribution to chemistry) is 6.022 x 10 .
12 23
That means that 12 g of C contains 6.022 x 10 atoms, and 12.01 g of natural carbon contains
23 12 13 14
6.022 x 10 atoms (a mixture of C, C, and C atoms, with an average atomic mass of 12.01)

A sample of 12.01 g of natural carbon contains the same number of atoms as 20.18 g of natural
23
Neon. Both samples contain 1 mole of atoms (6.022 x 10 atoms).

Example to determine the moles of atoms:

What is the number of moles of atoms, and the number of atoms in a 10.00 g sample of
Magnesium (Mg)?

Solution:
23
The mass of 1 mole (6.022 x 10 atoms) of magnesium is 24.31 g (according to the periodic
table of the element). The sample we are considering has a mass of 10.00 g. Since the mass is
less than 24.31 g, this sample contains less than 1 mole of magnesium atoms. We can calculate
the number of moles of magnesium atoms in 10.00 g as follows:

General Chemistry 202-NYA-05: Chapter 3 42


10 g Mg x 1 mol Mg = 0.411 mol Mg atoms
24.31 g Mg

The number of atoms in 10.00 g (0.411 mol) of magnesium is


23 23
0.411 mol Mg x 6.022 x 10 atoms = 2.477 x 10 atoms
1 mol Mg

Advice: Always write the units beside your quantities, to make sure you are calculating with the
correct notions.

Enrichment:

Experimental Determination of Avogadro’s Number

Some attribute the discovery of the Avogadro Number to Sir Michael Faraday, but its
importance and significance was realized much later by Avogadro while dealing with
industrial synthesis and chemical reactions.

Faraday passed 96480 C (coulombs) of electricity through hydrogen cations and found
that 1gram hydrogen was formed. Then he analysed that when 1 electron (with the charge
of 1.6 X 10-19 C) passed it gave 1 hydrogen atom. Therefore 96480 C must give 6.023 X
1023 atoms of hydrogen.

96480 C / 1.6 X 10-19 C = 6.03 x 1023

By this research scientists started calculating relative atomic masses of other atoms with
respect to hydrogen. Later hydrogen became difficult for experiment, so C-12 was chosen
for the determination of relative atomic masses.

Another approach to determining Avogadro¿s number starts with careful measurements


of the density of an ultrapure sample of a material on the macroscopic scale. The density
of this material on the atomic scale is then measured by using x-ray diffraction techniques
to determine the number of atoms per unit cell in the crystal and the distance between the
equivalent points that define the unit cell (see Physical Review Letters, 1974, 33, 464).

General Chemistry 202-NYA-05: Chapter 3 43


3.4 Molar Mass

A chemical compound is, ultimately, a collection of atoms. For example, ammonia consists of
molecules that each contain one nitrogen and three hydrogen atoms (NH3). How can we calculate
23
the mass of 1 mole of ammonia (in other words, what is the mass of 6.022 x 10 NH3
molecules)? Since each NH3 molecule contains one nitrogen atom and three hydrogen atoms, 1
mole of NH3 molecules contains 1 mole of nitrogen atoms and 3 moles of hydrogen atoms. The
mass of 1 mole of ammonia can be found by summing the masses of nitrogen and hydrogen
present

Mass of 1 mol of N = 14.01 g


Mass of 3 mol of H = 3 x 1.008 g
Mass of 1 mol of NH3 = 15.03 g

Because 15.03 g represents the mass of 1 mole of ammonia molecules, it makes sense to call it
the molar mass for ammonia.

The molar mass of a substance is the mass in grams of one mole of the compound. (The term
molecular weight has also been used for this quantity).

Example to calculate Molar mass:

1) Calculate the molar mass of potassium carbonate (K2CO3).


2) A certain sample of potassium carbonate contains 4.25 moles. What is the mass in
2−
grams of this sample? What is the mass of the CO3 ions present?

Solution:
+ 2−
1) Potassium carbonate is an ionic compound composed of K and CO3 ions. In 1 mole of
+ 2−
potassium carbonate there are 2 moles of K ions and 1 mole of CO3 ions. The molar mass is
calculated by summing the masses of the components:

2 K: 2 x 39.10 g = 78.20 g
1 C: 1 x 12.01 g = 12.01 g
3 O: 3 x 16.00 g = 48.00 g
Mass of 1 mol K2CO3 = 138.21 g
Thus the mass of 1 mole of K2CO3 (eg the molar mass) is 138.21 g
2) The mass of 1 mole of K2CO3 is 138.21 g. The sample contains 4.25 moles of K2CO3.

4.25 mol K2CO3 x 138.21 g K2CO3 = 587 g K2CO3


1 mol K2CO3
2−
To find the mass of carbonate ions (CO3 ) present in the sample, we must realize that 4.25
2− +
moles of K2CO3 contains 4.25 moles of CO3 ions and 8.50 moles of K ions. The mass of 1
2−
mole of CO3 ions is

1 C: 1 x 12.01 g = 12.01 g
3 O: 3 x 16.00 g = 48.00 g
2−
Mass of 1 mol CO3 = 60.01 g
2−
Thus the mass of 4.25 moles of CO3 ions is

General Chemistry 202-NYA-05: Chapter 3 44


2− 2− 2−
4.25 mol CO3 x 60.01 g CO3 = 255 g CO3
2−
1 mol CO3

To get better with the calculation regarding molar mass, please do the recommended problems in
the text book.

3.6 Percent Composition of Compounds


The percentages (by mass) of the elements of a compound can also be used to describe the
composition of a compound.

We can obtain the mass percents of the elements from the formula of the compound by
comparing the mass of each element present in 1 mole of the compound to the total mass of 1
mole of the compound.

Example:

For propanol, which has the formula C3H7OH, the mass of each element present and the molar
mass are obtained as follows:

Mass of C = 3 mol x 12.01 g/mol = 36.03 g


Mass of H = 8 mol x 1.008 g/mol = 8.064 g
Mass of O = 1 mol x 16.00 g/mol = 16.00 g
Mass of 1 mol C3H7OH = 60.09 g

The mass percent (often called the weight percent) of carbon in propanol can be calculated by
comparing the mass of carbon in 1 mole of propanol to the total mass of 1 mole of propanol, and
multiplying the result by 100:

Mass percent of C = mass of C in 1 mol C3H7OH x 100%


Mass of 1 mol C3H7OH

= 36.03 g x 100% = 59.96


60.09 g

The mass percents of hydrogen and oxygen in propanol are obtained in a similar manner:

The percentages must add up to 100% (that provides a check that the calculations are correct).

General Chemistry 202-NYA-05: Chapter 3 45


3.7 Determining the Formula of a Compound
The empirical formula of a chemical compound is a simple expression of the relative numbers of
each type of atom in it, (or the simplest whole number ratio of atoms of each element present in a
compound).

The molecular formula identifies each constituent element by its chemical symbol and indicates
the number of atoms of each element found in each discrete molecule of that compound. (It is
only one whole number possibility of the empirical formula, since it is for a definite molecule).

Empirical Formula Determination

• Since the mass percentage gives the number of grams of a particular element per 100 grams
of compound, base the calculation on 100 grams of compound. Each percent will then
represent the mass in grams of that element.
• Determine the number of moles of each element present in 100 grams of compounds using
the atomic masses of the elements present.
• Divide each value of the number of moles by the smallest of the values. If each resulting
number is a whole number (after appropriate roundings), these numbers represent the
subscripts of the elements in the empirical formula.
• If the numbers obtained in the previous step are not whole numbers, multiply each number by
an integer so that the results are whole numbers.

Determining Molecular Formula from Empirical Formula

• Obtain the empirical formula.


• Compute the mass corresponding to the empirical formula.
• Calculate the ratio: Molar mass / Empirical formula mass.
• The integer from the previous step represents the number of empirical formula units in one
molecule. When the empirical formula subscripts are multiplied by this integer, the molecular
formula results. This procedure is summarized by the equation :
• Molecular formula = (empirical formula) x (molar mass / empirical formula mass)

Determining Molecular Formula from Mass Percent and Molar Mass

• Using the mass percentages and the molar mass, determine the mass of each element
present in one mole of compound.
• Determine the number of moles of each element present in one mole of compound.
• The integers from the previous step represent the subscripts in the molecular formula.

To properly understand this concept, you need to practice solving different problems.

General Chemistry 202-NYA-05: Chapter 3 46


• Example 1:
The composition of adipic acid is 49.3% C, 6.9% H, and 43.8% O (by mass). The molar mass of
the compound is about 146 g/mol. What is the empirical formula? What is the molecular formula?

Solution 1:
What is the mass of each element in 100.00 g of compound?

C 49.3 g H 6.90 g O 43.8 g

What are the moles of each element in 100.00 g of compound?

49.3 g C ÷ 12.01 g mol−1 = 4.105 mol C


6.90 g H ÷ 1.008 g mol−1 = 6.845 mol H
43.8 g O ÷ 16.01 g mol−1 = 2.735 mol O

What is the empirical formula of the compound? (Divide each mole value by the smaller one)

4.105 / 2.735 = 1.5 C 6.845 / 2.735 = 2.5 H 2.735 / 2.735 = 1 O

This yields the formula C1.5H2.5O. Since the compounds must contain whole numbers of atoms,
the empirical formula should contain only whole numbers. To obtain the simplest set of whole
numbers, we multiply all numbers by 2 to give the empirical formula: C3H5O2

What is the molecular formula for the compound? (Compare the empirical formula mass to the
molar mass)

Empirical formula mass = (12.01 x 3) + (1.008 x 5) + (16.01 x 2) = 73.09 g/mol


Molar mass is given = 146 g/mol.
Molar mass / Empirical formula mass = (146 / 73.09) = 2

The molecular formula is (C3H5O2)2, which means: C6H10O4

• Example 2:
During physical activity, lactic acid forms in muscle tissue and is responsible for muscle soreness.
Elemental analysis shows that lactic acid has the following composition in % by mass: 40.0%C,
6.71%H and 53.3%O. Mass spectrometry data shows that lactic acid has a molecular mass of
90.08. Determine the empirical and molecular formulas of lactic acid.
Solution 2: Assume a sample of 100.00 g lactic acid.
Atomic Mass (g/mol): C = 12.011 H = 1.008 O = 16.00

Simplest
Atom Mass Moles Scaled Ratio Integer Ratio
Ratio
C 40.0 g 3.33 mol 1.00 (1.00) × 1 = 1.00 1
H 6.71 g 6.66 mol 2.00 (2.00) × 1 = 2.00 2
O 53.3 g 3.33 mol 1.00 (1.00) × 1 = 1.00 1

Empirical Formula: CH2O


Empirical Formula Mass (FM) = 1× (12) + 2 × (1) + 1 × (16) = 30 g/mol & Molecular Mass(MM) = 90.08
Scale factor = MM / FM = 90.08 / 30 ≈ 3
Molecular Formula: C3H6O3

General Chemistry 202-NYA-05: Chapter 3 47


Empirical Formulas by combustion Analysis
Combustion analysis involves burning the compound in oxygen and analyzing the combustion
products. The combustion products are always oxides. Thus, if the original sample contains a
metal atom, the combustion product is the metal oxide. For organic compounds which generally
consist of at least carbon (C) and hydrogen (H) atoms, the combustion products are carbon
dioxide, CO2 , and water, H2O.
The steps in obtaining the empirical formula from combustion analysis data are identical to those
in elemental analysis. The only additional complication is in the first step, namely obtaining the
mass of each atom in the compound.
This is done as follows. Combustion analysis data always includes the mass of the products (i.e.,
the oxides). From the mass of the oxide product, the mass of the atom is computed using the %
mass of the atom in the oxide. Note, however, that is not true if the original compound contains O,
since there will be two sources of O in the combustion process.

Example 1:
Cumene is a compound containing only carbon and hydrogen atoms. Combustion of a sample of
cumene yields 156.8 mg carbon dioxide and 42.8 mg water. The molar mass of cumene is between
115 and 125 (g per mole). Determine the empirical and molecular formulas.
From the mass of carbon dioxide, the mass of C must be extracted. Likewise, the mass of H must
be obtained from water. To do this the %C and %H in CO2 and H2O respectively must be
calculated.
Molar Mass:
CO2 = 1 atom C × (Atomic Mass C) + 2 atoms O × (Atomic Mass O) = 1 atom C × (12.011) + 2 atom O ×
(16.00)
= 12.011 amu C + 32.00 amu = 44.01 amu
12.011
%C = × 100% = 27.29%
44.01

H2O = 2 atoms H × (Atomic Mass H) + 1 atom O × (Atomic Mass O) = 2 atoms H × (1.008) + 1 atom O ×
(16.00)
= 2.016 amu H + 16.00 amu = 18.02 amu
2.016
%H = × 100% = 11.19%
18.02

In the sample of the unknown compound:


 27.29 g C 
mC = 0.1568 g CO2   = 0.04279 g C
 100 g CO2 
 11.19 g H 
mH = 0.0428 g H 2O   = 0.00479 g H
 100 g H 2O 
Atomic Mass: C =12.011 H = 1.008

Simplest Integer
Atom Mass Moles Scaled Ratio
Ratio Ratio
C 0.04279 g 0.003562 mol 1.00 (1.00) × 3 = 3.00 3
H 0.00479 g 0.00475 mol 1.33 (1.33) × 3 = 4.00 4

General Chemistry 202-NYA-05: Chapter 3 48


Empirical Formula: C3H4
Empirical Formula Mass: FM = 3× (12) + 4 × (1) = 40
Molecular Mass: MM = 115 and 125
FM 115 125
Scale factor = = ; = 2.9; 3.1
MM 40 40

The nearest integer between 2.9 and 3.1 is 3


Molecular Formula: C9H12

Example 2:
Propionic acid is an organic acid containing C, H and O. A 0.236 g sample of propionic acid is
completely burned in oxygen, yielding 0.421 g carbon dioxide and 0.172 g water. Calculate the
empirical formula of propionic acid.
The % compositions of C and H in CO2 and H2O respectively have calculated in the previous problem.
In the sample of the propionic acid:
 27.29 g C 
mC = 0.421 g CO2   = 0.115 g C
 100 g CO2 
 11.19 g H 
mH = 0.172 g H 2O   = 0.0192 g H
 100 g H 2O 
mO = m propionic acid − mC − mH = 0.236 g − 0.115 g C − 0.0192 g H = 0.102 g O

Simplest Integer
Atom Mass Moles Scaled Ratio
Ratio Ratio
C 0.115 g 0.00957 mol 1.50 (1.50) × 2 = 3.00 3
H 0.0192 g 0.0190 mol 2.98 (2.98) × 2 = 5.96 6
O 0.102 g 0.00638 mol 1.00 (1.00) × 2 = 2.00 2

Empirical Formula: C3H6O2

Extra Problems:

General Chemistry 202-NYA-05: Chapter 3 49


1. The atomic mass of rhenium is 186.2. Given that 37.1% of natural rhenium is rhenium-185, what is
the other stable isotope?
183
a) 75 Re
187
b) 75 Re
189
c) 75 Re
181
d) 75 Re
190
e) 75 Re

2. Consider the element indium, atomic number 49, atomic mass 114.8 g. The nucleus of an atom of
indium-112 contains
a) 49 protons, 63 neutrons, 49 electrons
b) 49 protons, 49 neutrons
c) 49 protons, 49 alpha particles
d) 49 protons, 63 neutrons
e) 49 protons, 112 neutrons
3. A hypothetical element consists of two isotopes of masses 86.95 amu and 88.95 amu with
abundances of 35.5% and 64.5%, respectively. What is the average atomic mass of this element?
a) 87.95 amu
b) 86.95 amu
c) 87.7 amu
d) 88.2 amu
e) 88.95 amu
4. Naturally occurring copper exists in two isotopic forms: 63Cu and 65Cu. The atomic mass of copper is
63.55 amu. What is the approximate natural abundance of 63Cu?
a) 63%
b) 90%
c) 70%
d) 50%
e) 30%
5. Naturally occurring element X exists in three isotopic forms: X-28 (27.977 amu, 92.23% abundance),
X-29 (28.976 amu, 4.67% abundance), and X-30 (29.974 amu, 3.10% abundance). Calculate the
atomic weight of X.
a) 28.09 amu
b) 48.63 amu
c) 27.16 amu
d) 28.97 amu
e) 86.93 amu
6. Naturally occurring element X exists in three isotopic forms: X-28 (27.977 amu, 92.23% abundance),
X-29 (28.976 amu, 4.67% abundance), and X-30 (29.974 amu, 3.10% abundance). What is the
identity of element X?
a) Cu
b) Al
c) Ni
d) Si
e) Sr
7. The average mass of a carbon atom is 12.011. Assuming you were able to pick up only one carbon
unit, the chances that you would randomly get one with a mass of 12.011 is
a) 0%
b) 0.011%
c) about 12%
d) 12.011%
e) greater than 50%

General Chemistry 202-NYA-05: Chapter 3 50


8. What is the mass of 4 atom(s) of copper in grams?
a) 254.2 g
b) 2.37 × 1021 g
c) 9.57 × 10–24 g
d) 6.022 × 1023 g
e) 4.22 × 10–22g
9. Iron is biologically important in the transport of oxygen by red blood cells from the lungs to the
various organs of the body. In the blood of an adult human, there are approximately 2.69 × 1013 red
blood cells with a total of 2.90 g of iron. On the average, how many iron atoms are present in each
red blood cell? (molar mass Fe = 55.85 g/mol)
a) 8.60 × 10–10
b) 1.16 × 109
c) 3.13 × 1022
d) 2.69 × 1013
e) 6.02 × 10–2
10. You have a sample of zinc (Zn) and a sample of aluminum (Al). You have an equal number of atoms
in each sample. Which of the following statements concerning the masses of the samples is true?
a) The mass of the zinc sample is more than twice as great as the mass of the aluminum sample.
b) The mass of the zinc sample is more than the mass of the aluminum sample, but it is not twice
as great.
c) The mass of the aluminum sample is more than twice as great as the mass of the zinc sample.
d) The mass of the aluminum sample is more than the mass of the zinc sample, but it is not twice
as great.
e) The masses of each sample are equal.
11. A sample of ammonia has a mass of 45.5 g. How many molecules are in this sample?
a) 2.67 molecules
b) 2.74 × 1025 molecules
c) 2.25 × 1023 molecules
d) 1.61 × 1024 molecules
e) 9.02 × 10–16 molecules
12. How many moles of hydrogen chloride are contained in a 52.6-g sample of this gas?
a) 0.693 mol
b) 1.44 mol
c) 89.1 mol
d) 52.2 mol
e) 2.89 mol
13. What is the molar mass of propanol (C3H7OH)?
a) 59.09 g/mol
b) 36.07 g/mol
c) 60.09 g/mol
d) 30.03 g/mol
e) 149.04 g/mol
14. For which compound does 0.256 mole weigh 12.8 g?
a) C2H4O
b) CO2
c) CH3Cl
d) C2H6
e) none of these

Ans: 1) b 2) d 3) d 4) c 5) a 6) d 7) a 8) e 9) b 10) a 11) d 12) b 13) c 14) c

General Chemistry 202-NYA-05: Chapter 3 51


3.8 Chemical Equations
Chemical Reactions
A chemical change involves a reorganization of the atoms in one or more substances.

This is represented by a chemical equation with the reactants on the left side of an arrow and
the products on the right side.

Example: Combustion of methane (CH4)

CH4 + O2 → CO2 + H2O


Reactants Products

All atoms present in the reactants must be accounted for among the products. There must be the
same number of each type of atom on the product side and on the reactant side of the arrow.
Making sure this rule is obeyed for a reaction is called balancing a chemical equation.

The reaction above is not balanced. It should have the same number of each type of atom
represented among the reactants and the products:

CH4 + 2 O2 → CO2 + 2 H2O


Reactants (1C, 4H, 4O) Products (1C, 4H, 4O)

Meaning of a Chemical Equation


The nature of the reactants and products, and the relative numbers of each are represented in a
chemical equation.

Also, the equation often gives the physical states of the reactants and products.

State Symbol
Solid (s)
Liquid (l)
Gas (g)
Dissolved in water (aqueous solution) (aq)

Note that the mass for both reactants and products are the same, in all cases.

Information obtained by the balanced equation for the combustion of methane (CH4)
Reactants Products
CH4(g) + 2O2(g) → CO2(g) + 2H2O(g)
1 molecule + 2 molecules → 1 molecule + 2 molecules
1 mole + 2 moles → 1 mole + 2 moles
6.022 x 1023 molecules + 2 (6.022 x 1023 molecules) → 6.022 x 1023 molecules + 2 (6.022 x 1023 molecules)
16 g + 2 (32) g → 44 g + 2 (18 g)
80 g of reactants → 80 g of products

General Chemistry 202-NYA-05: Chapter 3 51


3.9 Balancing Chemical Equations
The principle is that atoms are conserved in a chemical reaction.
For example, when liquid ethanol is burned in the presence of sufficient oxygen gas, the products
are always carbon dioxide and water (there is no other products that are created)

C2H5OH(l) + 3O2(g) → 2CO2(g) + 3H2O(g)

The formulas of the compounds must never be changed in balancing a chemical equation. That
means the subscripts in a formula cannot be changed, nor can atoms be added or subtracted
from a formula. (Do not confuse atoms and molecules).

Writing and Balancing the Equation for a Chemical Reaction


1. Determine what reaction is occurring. What are the reactants, the products, and the
physical states involved?
2. Write the unbalanced equation that summarizes the reaction described in step 1.
3. Balance the equation (add the coefficients in front of the molecules) by inspection,
starting with the most complicated molecule(s). The same number of each type of atom
needs to appear on both reactant and product sides.

Example:

Unbalanced equation: CaO(s) + C(s) → CaC2(s) + CO2(g)

Balanced equation 2CaO(s) + 5C(s) → 2CaC2(s) + CO2(g)

• The number of atoms of each type of element must be the same on both sides of a
balanced equation.
• Subscripts must not be changed to balance an equation.
• A balanced equation tells us the ratio of the number of molecules which react and are
produced in a chemical reaction.
• Coefficients can be fractions, although they are usually given as lowest integer multiples.

3.10 Stoichiometric Calculations: Amounts of Reactants and


Products
Chemical equations can be used to relate the masses of reacting chemicals.

Coefficients in chemical equations represent numbers of molecules, not masses of molecules.


However, when a reaction is run in a lab, the amounts of substances needed cannot be
determined by counting molecules directly. Counting is always done by weighing. This section will
show how chemical equations can be used to determine masses of reacting chemicals.

Using stoichiometric calculations, you must be able to determine the amounts (mol and grams) of
reactants or products from the chemical equation.

General Chemistry 202-NYA-05: Chapter 3 52


Calculating Masses of Reactants and Products in Reactions
1. Balance the equation for the reaction.
2. Convert the known mass of the reactant or product to moles of that substance.
3. Use the balanced equation to set up the appropriate mole ratios.
4. Use the appropriate mole ratios to calculate the number of moles of desired reactant or
product.
5. Convert from moles back to grams if required by the problem.

Example:
P4(s) + O2(g) → P2O5(s)

If 6.25 g of phosphorus is burned, what mass of oxygen does it combine with?

Solution:

1. P4(s) + 5O2(g) → 2P2O5(s)


2. P4(s) has a molar mass of (4 x 30.97 g mol−1 of P) 123.9 g mol−1
6.25 g / 123.9 g mol−1 = 0.0504 mol
So 6.25 g of P4(s) is equal to 0.05 mol of P4(s)
3-4. Therefore, in this equation, there is (5 x 0.05 mol of O2(g)) 0.25 mol of O2(g)
And (2 x 0.05 mol of P2O5(s)) 0.10 mol of P2O5(s)
5. We can determine that if the reaction is complete, it will need
0.25 mol of O2(g) x 32.02 g mol−1 of O2(g) = 8.07 g of O2(g)
0.10 mol of P2O5(s) x 142.0 g mol−1 of P2O5(s) = 14.2 g of P2O5(s)

Since in this reaction, all the phosphorous combines with all the oxygen, 6.25 g of phosphorus
combine with a mass of oxygen of 8.07 g.

Read carefully the examples 3.15 and 3.16 of your text book.

General Chemistry 202-NYA-05: Chapter 3 53


Extra problems:

1. Which of the following equations is not balanced?


a) 4Al + 3O2 → 2Al2O3
b) C2H6 + 7 2 O2 → 2CO2 + 3H2O
c) 2KClO3 → 2KCl + 32 O2
d) 4P4 + 5S8 → 4P4S10
e) P4 + 5O2 → P4O10

2. Use the following to answer the two following questions:

Consider a specific chemical reaction represented by the equation aA + bB cC + dD. In this


equation the letters A, B, C, and D represent chemicals, and the letters a, b, c, and d represent
coefficients in the balanced equation.
How many possible values are there for the quantity “c/d”?
a) 1
b) 2
c) 3
d) 4
e) infinite
How many possible values are there for the quantity “c”?
a) 1
b) 2
c) 3
d) 4
e) infinite

3. What is the coefficient for oxygen when the following equation is balanced?
NH3(g) + O2(g) → NO2(g) + H2O(g)
a) 3
b) 6
c) 7
d) 12
e) 14

4. Determine the coefficient for O2 when the following equation is balanced in standard form (smallest
whole numbers).
C8H18(g) + O2(g) → CO2(g) + H2O(g)
a) 8
b) 17
c) 18
d) 25
e) 16

General Chemistry 202-NYA-05: Chapter 3 54


5. wPCl5 + xH2O → yPOCl3 + zHCl
The above equation is properly balanced when:
a) w = 1, x = 2, y = 2, z = 4
b) w = 2, x = 2, y = 2, z = 2
c) w = 2, x = 2, y = 2, z = 1
d) w = 1, x = 1, y = 1, z = 2
e) none of these

6. Give (in order) the correct coefficients to balance the following reaction:
H2SnCl6 + H2S → SnS2 + HCl
a) 1, 2, 1, 6
b) 1, 2, 2, 2
c) 1, 1, 1, 6
d) 6, 2, 1, 1
e) 2, 4, 2, 6

7. In the balanced equation for the reaction:


xP4O6(s) + yH2O(l) → zH3PO3(aq)

if x equals 2, the coefficient z equals:


a) 2
b) 4
c) 6
d) 10
e) none of these

8. A 6.82-g sample of potassium chlorate was decomposed according to the following equation:
2KClO3 → 2KCl + 3O2
How many moles of oxygen are formed?
a) 2.67 g
b) 0.0557 mol
c) 0.0371 mol
d) 0.0835 mol
e) none of these

9. How many grams of Ca(NO3)2 can be produced by reacting excess HNO3 with 6.55 g of Ca(OH)2?
a) 7.25 g
b) 14.5 g
c) 29.0 g
d) 13.1 g
e) 6.55 g

Ans: 1) c 2) a then e 3) c 4)d 5)d 6)a 7)e 8)d 9)b

General Chemistry 202-NYA-05: Chapter 3 55


3.11 The Concept of Limiting Reagent
When a chemical reaction occurs, the reagents involved are not always in the perfect proportion
so that they will all be consumed.

Balanced reaction: O2 + 2 H2 → 2 H2 O
Proportions used "in a lab": 32 g 10 g ?
So what quantity (in g) is produced?
Beginning End
O―O H―H H―O―H
H―H H―O―H
H―H H―H
H―H H―H
H―H H―H
O2 is limiting the reaction. Two moles of H2O will be produced, so 36 g.

The reactant (also called reagent) that runs out first, and thus limits the amounts of products that
can form, is called the limiting reactant (limiting reagent). In other words, the limiting reagent is
the chemical that determines how far the reaction will go before the chemical in question gets
"used up", causing the reaction to stop. The chemical of which there are fewer moles than the
proportion requires is the limiting reagent. The other reagents are considered to be in surplus,
and therefore are called excess reagents.

To determine how much product can be formed from a given mixture of reactants, we have to
look at the reactant that is limiting.

Example 1 :

If 20.0 g of Fe2O3 are reacted with 8.00 g Al(s) in the thermite reaction, Which reactant is
limiting?.

Solution:

First, determine how many moles of Fe(l) can be produced from either reactant.

Moles produced of Fe from reactant Fe2O3

Moles produced of Fe from reactant Al

Because the moles Fe produced from Fe2O3(0.254mol) is less than the moles Fe produced from
Al(0.297mol), Fe2O3 is the limiting reagent.

General Chemistry 202-NYA-05: Chapter 3 56


Problem Solving Strategy:
1. Write and balance the equation for the reaction
2. Convert the known masses of substances to moles
3. Determine which reactant is limiting
4. Using the amount of the limiting reactant and the appropriate mole ratios, compute
the number of moles of the desired product.
5. Convert from moles to grams, using the molar mass
6. Calculated the yields according to the information you have.

Example 2:
When 36.127 g of benzene (C6H6) is burnt in 115.723 g of oxygen, how much product is formed?

Solution:

Atomic Mass: H = 1.00797 C = 12.011 O = 15.9994

The steps in the solution are identical to the stoichiometry problems solved previously. There is
only one additional step – to determine the limiting reactant through an extra calculation.

Step 1: Write the balanced chemical equation


2 C6H6 (l) + 15 O2 (g) → 12 CO2 (g) + 6 H2O (l)
Step 2: Calculate the number of moles of the given compounds

Molar Masses: C6H6 = 6 × (12.011) + 6 × (1.00797) = 78.114 g mol−1


O2 = 2 × (15.9994) = 31.9988 g mol−1
CO2 = (12.011) + 2 × (15.9994) = 44.010 g mol−1
H2O = 2 × (1.00797) + (15.9994) = 18.0153 g mol−1
 mol 
nC6H6 = 36.127 g C6 H 6   = 0.46249 mol
 78.114 g 
 mol 
nO = 115.723 g O2   = 3.61648 mol
 31.9988 g 
2

Step 3: Calculate the number of moles of the other compounds in the chemical equation.
To do this we follow the schematic outlined in the following schematic diagram.

2 C6H6 (l) + 15 O2 (g) → 12 CO2 (g) + 6 H2O (l)

Moles of C6H6 Moles of O2 Use the moles of the Limiting Reactant


to determine the quantities of produce

Use mole ratios to


determine the
Limiting Reactant

Mole of the Limiting Reactant

General Chemistry 202-NYA-05: Chapter 3 57


Determine the Limiting Reactant. Assume that one of the reactants is limting. On the basis
of the amount of this reactant, calculate the quantity of the other reactant needed.
For no real reason, let us choose C6H6 . We now ask: On the basis of 0.46249 mol C6H6 , how
much oxygen are required?

 15 mol O2 
nO = 0.46249 mol C6 H 6   = 3.4687 mol O2 are required
 2 mol C6 H 6
2

Now we compare the value of O2 required (3.4687 mol), with the amount of O2 that we
actually have (i.e., 3.61648 mol).
In this problem, we actually have more oxygen than we need. Thus, we have a basis for judging
whether we have established the limiting reactant.

Moles of O2 Moles of O2
available required
3.61648 mol 3.4687 mol

Since we have more O2 than is actually needed, the C6H6 will be used up before the oxygen.
Therefore, C6H6 is the limiting reactant. The amount of C6H6 initially present will determine the
amount of product obtained.

Step 4: Based on the Limiting Reactant, calculate the number of moles of the other compounds
at the end of the reaction.
nO = 3.61648 − 3.4687 = 0.1478 mol
2

 12 mol CO2 
nCO = 0.46249 mol C6 H 6   = 2.7749 mol CO2
 2 mol C6 H 6 
2

 66 mol 2O 2 
mol HCO
nH O = 0.46249 mol C6 H 6   = 1.3875 mol H 2 O
 2 mol C6 H 6 
2

Step 5: Calculate the masses of the other compounds at the end of the reaction.
 31.9988 g O2 
mO = 0.1478 mol O2   = 4.733 g O2
 mol O2
2

 44.010 g CO2 
mCO = 2.7749 mol CO2   = 122.13 g CO2
 mol CO2 
2

 18.0153 g H 2 O 
mH O = 1.3875 mol H 2 O 
2  = 24.996 g H 2 O
 mol H 2 O 
Thus, at the end of the reaction, there will be no C6H6, there will remain 4.733 g O2 , and 122.13 g
CO2 and 24.996 g H2O.

General Chemistry 202-NYA-05: Chapter 3 58


Example 3 :

Calculate the amount of each compound remaining after 5.00 g each of hydrogen and oxygen are
reacted to form water?

Solution:

Atomic Mass: H = 1.00797 O = 15.9994

Step 1: Write the balanced chemical equation


2 H2 (l) + O2 (g) → 2 H2O (l)

Step 2: Calculate the number of moles of the given compounds


Molar Masses: H2 = 2 × (1.008) = 2.016
O2 = 2 × (16.00) = 32.00
H2O = 2 × (1.008) + (16.00) = 18.02
 mol 
nH 2 = 5.00 g H 2   = 2.48 gmol
H 2H2
 2.016 g H 2 

 mol 
nO = 5.00 g O2   = 0.156 gmol
O2O2
2
 32.00 g O2 

Step 3: Determine the Limiting Reactant.

Assume H2 is the Limiting Reactant. Then the number of moles of O2 required to react completely
with all the H2 is:
 1 mol O2 
nO2 = 2.48 mol H 2   = 1.24 mol O2
 2 mol H 2 
Actual moles of O2 = 0.156 mol < 1.24 mol O2 needed.
There is not enough O2 to react with all the H2 . Therefore O2 must be the Limiting Reactant. The
Limiting Reactant will determine how much product remains.

Step 4: Calculate the number of moles of the other compounds in the chemical equation at the
end of the reaction based on the Limiting Reactant.
 2 mol H 2 
nH 2 = moles H 2 initially present − moles H 2 used up = 2.48 − 0.156 mol O2   = 2.17 mol H 2 remains
 1 mol O2 
 2 mol H 2 O 
nH O = 0.156 mol O2   = 0.312 mol H 2 O
 1 mol O2 
2

Step 5: Based on the Limiting Reactant, calculate the number of moles of the other compounds
at the end of the reaction.
 2.016 g 
mH 2 = 2.17 mol   = 4.37 g H 2
 mol 
 18.02 g 
mH 2O = 0.312 mol   = 5.62 g H 2 O
 mol 

General Chemistry 202-NYA-05: Chapter 3 59


Example 4:

If 20.0 g aluminum is reacted with 15.0 g oxygen, how much aluminum oxide will be obtained?

Solution:

Atomic Mass: Al = 26.98 O = 16.00

Step 1: Write the balanced chemical equation


4 Al (s) + 3 O2 (g) → 2 Al2O3 (s)

Step 2: Calculate the number of moles of the given compounds


Molar Mass O2 = 2 × (16.00) = 32.00
Al2O3 = 2 × (26.98) + 3 × (16.00) = 101.96
 mol 
nAl = 20.0 g Al   = 0.741 mol Al
 26.98 g Al 
 mol 
nO2 = 15.0 g O2   = 0.469 mol O2
 32.00 g O2 

Step 3: Determine the Limiting Reactant.

Assume Al is the Limiting Reactant. Then the number of moles of O2 required to completely react
with all the Al is:
 3 mol O2 
nO2 = 0.741 mol Al   = 0.556 mol O2 are required
 4 mol Al 
Compare the number of moles O2 that are actually present with the number of moles O2 that are
required”
0.467 mol O2 are given < 0.556 mol O2 are required.
There are not enough moles O2 initially present so O2 must be the Limiting Reactant.

Step 4: Based on the Limiting Reactant, calculate the number of moles of the other compounds
at the end of the reaction.
 4 mol Al 
nAl = moles Al initially present − moles Al used up = 0.741 − 0.469 mol O2   = 0.116 mol Al remains
 3 mol O2 
 2 mol Al2 O3 
nAl2O3 = 0.469 mol O2   = 0.313 mol Al2 O3
 3 mol O2 

Step 5: Calculate the mass of the other compounds in the chemical equation
 26.98 g Al 
mAl = 0.116 mol Al   =3.13 g Al
 mol Al 
 101.96 g Al2 O3 
nAl2O3 = 0.313 mol Al2 O3   = 31.9 g Al2 O3
 mol Al2 O3 

General Chemistry 202-NYA-05: Chapter 3 60


Limiting Reactant Simpler Determination:
A simpler way to determine which reactant is limiting is to compare the mole ratio of the
substances required by the balanced equation with the mole ratio of reactants actually present.

Example:

N2(g) + 3H2(g) → 2NH3(g)

In this case, the mole ratio of H2 to N2 required by the balanced equation is:

3 mol H2 / 1 mol N2

That is,

mol H2 / mol N2 (required) = 3 / 1 = 3


3 2
If in this experiment, we have 2.48 x 10 mol H2 and 8.93 x 10 mol N2, the ratio
3 2
mol H2 / mol N2 (actual) = 2.48 x 10 / 8.93 x 10 = 2.78

Since 2.78 is less than 3, the actual mole ratio of H2 to N2 is too small, and H2 must be limiting. If
the actual H2 to N2 mole ratio had been greater than 3, then the H2 would have been in excess
and the N2 would be limiting.

Yields
The amount of a product formed when the limiting reactant is completely consumed is called the
theoretical yield of that product. This is the maximum amount of nitrogen that can be produced
from the quantities of reactants used.

The actual yield of product is often given as a percentage of the theoretical yield. This is called
the percent yield:

Actual yield x 100% = Percent yield


Theoretical yield

So if a reaction gave 34.3 grams of products instead of 61.6 grams of predicted products, the
percent yield of products would be

34.3 g x 100% = 55.7%


61.6 g

This is the ratio of what you got to what you should have gotten.
Thus for Example 4, in the preceding problem where 20.0 g Al are reacted with 15.0 g O2 , we
calculated that 31.9 g Al2O3 would be expected. The 31.9 g Al2O3 are the theoretical yield. If the
experiment was actually carried out, and 31.1 g Al2O3 were obtained then:
31.1 g
% yield = ×100% = 97.5%
31.9 g

General Chemistry 202-NYA-05: Chapter 3 61


Example 5:

In the following (unbalanced) reaction

NH3 (g) + O2 (g) → NO (g) + H2O (l) ,


16.5 g NH3 is allowed to react with 33.0 g O2 to produce 21.3 g nitrogen monoxide. Calculate the
theoretical yield and the percent yield.
Atomic Mass: H = 1.008 N = 14.007 O = 16.00
Step 1: Write the balanced chemical equation
4 NH3 (g) + 5 O2 (g) → 4 NO (g) + 6 H2O (l) ,
Step 2: Calculate the number of moles of the given reactants
Molar Mass O2 = 2 × (16.00) = 32.00
NH3 = (14.007) + 3 × (1.008) = 17.031
NO = (14.007) + (16.00) = 30.01
H2O = 2 × (1.008) + (16.00) = 18.02
 mol 
nNH3 = 16.5 g NH 3   = 0.969 mol
 17.031g 
 mol 
nO2 = 33.0 g O2   = 1.03 mol
 32.00 g 
Step 3: Determine the Limiting Reactant.
Assume NH3 is the Limiting Reactant. Then the number of moles of O2 required to completely
react with all the NH3 is:

 5 mol O2 
nNH3 = 0.969 mol NH 3   = 1.21 mol O2 are required
 4 mol NH 3 
Compare the number of moles O2 that are actually present with the number of moles O2 that are
required”
1.03 mol O2 are given < 1.21 mol O2 are required.
There are not enough moles O2 initially present so O2 must be the Limiting Reactant.
Step 4: Based on the Limiting Reactant, calculate the number of moles of the other compounds
at the end of the reaction.
 4 mol NH 3 
nNH3 = moles NH 3 initially present − moles NH 3 used up = 0.969 − 1.03 mol O2   = 0.145 mol NH 3 remains
 5 mol O2 
 4 mol NO 
nNO = 1.03 mol O2   = 0.824 mol NO
 5 mol O2 
 6 mol H 2 O 
nH 2O = 1.03 mol O2   = 1.24 mol H 2 O
 5 mol O2 

General Chemistry 202-NYA-05: Chapter 3 62


Example 5: Different method (same result) for step 3

Step 3: Determine the Limiting Reactant.

With the given amount of each reagent (in moles), how many moles of product can be obtained?
4 NH3 (g) + 5 O2 (g) → 4 NO (g) + 6 H2O (l)
Choose one product molecule that contains atoms from both reagent molecules (in this case, NO
or H2O can be chosen. For this example, we will continue with NO).
With the starting amount of NH3, how many moles of NO can be made?
0.969 mol NH3 x 4 moles NO = 0.969 moles of NO
4 moles NH3
With the starting amount of O2, how many moles of NO can be made?
1.03 mol O2 x 4 moles NO = 0.824 moles of NO
5 moles O2

Since less product can be obtained with O2 then with NH3 (0.824 < 0.969), O2 is said to be the
limiting reagent.

This method offers the advantage of finding out the amount of moles of final product being
produced. Usually, an academic question also asks for the amount of final product produced after
asking for the identification of the limiting reagent. Since you have already figured it out, you kind
of save a step ☺

General Chemistry 202-NYA-05: Chapter 3 63


Step 5: Calculate the mass of the other compounds in the chemical equation
 17.031 g 
mNH3 = 0.145 mol Al   =2.47 g NH 3
 mol 
 30.01 g 
mNO = 0.824 mol NO   =24.7 g NO
 mol 
 18.02 g 
mH 2O = 1.24 mol H 2 O   = 22.3 g H 2 O
 mol 
Step 6: Calculate the % yield
Theoretical yield = 24.8 g NO
Actual yield = 21.3 g NO
21.3 g
% yield = ⋅100% = 86.2%
24.7 g

The Limiting Reagent: Summary


The limiting reagent (or reactant) is the reactant that is consumed first and therefore limits the
amounts of products that can be formed.
You have to determine the amount of limiting reagent to correctly calculate the theoretical yield of
100% conversion.

Actual yield
× 100% = percent yield
Theoretical yield

• Actual yield: Obtained Experimentally (in g)


• Theoretical yield: Calculated, if ALL moles of the limiting reagent form products (in g)
• Percent yield: The fraction describing how much product is obtained experimentally,
compared to the expected amount of a perfect reaction (in %)

Example:
A 15 g sample of lithium is reacted with 15 g of fluorine to form lithium fluoride. After the reaction
is complete, what will be present?
2Li + F2 → 2LiF
Solution

1) Determine which reactant is the limiting reagent.


2) Calculate how many moles of product will be formed if all the limiting reagent is
consumed.
3) Calculate how many moles of the “excess reagent” will be left after all the limiting reagent
is consumed.
4) Transform the quantities of moles back to grams.

Li: 15g = (15 g / 6.941 g⋅mol−1) = 2.16 molLi


that means it can produce 2.16 molLi x (2molLiF/2molLi) = 2.16 molLiF

F2: 15g = (15 g / 38 g⋅mol−1) = 0.395 molF2


that means it can produce 0.395 molF2 x (2molLiF/1molF2) = 0.789 molLiF

Therefore, F2 is the limiting reagent (it can produce the least amount of final product (LiF)).

So at the end of the reaction, no more F2 is left.

General Chemistry 202-NYA-05: Chapter 3 64


And 0.789 mol of LiF will be produced.
(which means in mass : 0.789 mol x 25.641gmol−1 = 20.47g of LiF )

To produce 0.789 molLiF, you needed 0.789 molLi


At the beginning, you had 2.16 molLiF
At the end, you will be left with (2.16 – 0.789) = 1.371 molLi
(which means in mass : 1.371 mol x 6.941 gmol−1 = 9.52 g of Li )

If you’ve experimentally obtained 18.56 g of LiF, what is the percent yield of the reaction?
Actual yield = 18.56 g = 90.7%
Theoretical yield 20.47 g

Extra Problems:
1. Phosphoric acid can be prepared by reaction of sulfuric acid with “phosphate rock” according to the
equation:
Ca3(PO4)2 + 3H2SO4 → 3CaSO4 + 2H3PO4

Suppose the reaction is carried out starting with 105 g of Ca3(PO4)2 and 93.3 g of H2SO4. Which
substance is the limiting reactant?
a) Ca3(PO4)2
b) H2SO4
c) CaSO4
d) H3PO4
e) none of these
2. Suppose the reaction Ca3(PO4)2 + 3H2SO4 → 3CaSO4 + 2H3PO4 is carried out starting with 127 g of
Ca3(PO4)2
and 71.9 g of H2SO4. How much phosphoric acid will be produced?
a) 71.8 g
b) 47.9 g
c) 198.9 g
d) 107.7 g
e) 80.2 g
3. Phosphorus, P4, can be prepared from calcium phosphate by the reaction
3390 g 1795 g 660 g

2Ca3(PO4)2 + 6SiO2 + 10C → 6CaSiO3 + P4 + 10CO

310 g/mol 60.1 g/mol 12.0 g/mol

The molar mass for each reactant is shown below the reactant, and the mass of each reactant for this
problem is given above. Which reactant is the limiting reagent?
a) C
b) SiO2
c) Ca3(PO4)2
d) P4
e) More information is needed.

General Chemistry 202-NYA-05: Chapter 3 65


4. How many grams of H2O will be formed when 32.0 g H2 is mixed with 27.0 g of O2 and allowed to
react to form water?
a) 30.4 g
b) 286 g
c) 15.2 g
d) 7.6 g
e) 144 g
5. A 15-g sample of lithium is reacted with 15 g of fluorine to form lithium fluoride:
2Li + F2 → 2LiF. After the reaction is complete, what will be present?
a) 2.16 moles lithium fluoride only
b) 0.789 moles lithium fluoride only
c) 2.16 moles lithium fluoride and 0.395 moles fluorine
d) 0.789 moles lithium fluoride and 1.37 moles lithium
e) none of these
6. Consider the fermentation reaction of glucose:
C6 H12 O 6 → 2C2 H 5 OH + 2CO 2

A 1.00-mole sample of C6H12O6 was placed in a vat with 100 g of yeast. If 51.1 grams of C2H5OH
was obtained, what was the percent yield of C2H5OH?
a) 55.5%
b) 27.7%
c) 100%
d) 51.1%
e) none of these
7. A 5.95-g sample of AgNO3 is reacted with BaCl2 according to the equation
2AgNO3 (aq) + BaCl2 (aq) → 2AgCl(s ) + Ba(NO3 ) 2 (aq)

to give 4.19 g of AgCl. What is the percent yield of AgCl?


a) 59.4%
b) 70.4%
c) 41.7%
d) 83.5%
e) 100%
8. The reaction of 11.9 g of CHCl3 with excess chlorine produced 10.6 g of CCl4, carbon tetrachloride:
2CHCl3 + 2Cl 2 → 2CCl4 + 2HCl

What is the percent yield?


a) 100%
b) 34.5%
c) 68.9%
d) 89%
e) 45.9%
9. Given the equation 3A + B → C + D, you react 2 moles of A with 1 mole of B. Which of the
following is true?
a) A is the limiting reactant because of its higher molar mass.
b) A is the limiting reactant because you need 3 moles of A and have 2.
c) B is the limiting reactant because you have fewer moles of B than A.
d) B is the limiting reactant because 3 A molecules react with 1 B molecule.
e) Neither reactant is limiting.

Ans: 1) b 2) b 3) b 4) a 5) d 6) a 7) d 8) c 9) b

General Chemistry 202-NYA-05: Chapter 3 66


Chapter 5:

Gas Stoichiometry

th
Suggested problems in chapter 5 of Zumdahl 8 Edition:

63, 65, 67, 71, 73, 75.

General Chemistry 202-NYA-05: Chapter 5 67


5.4 Gas Stoichiometry
Gases are submitted to different conditions of Pressure, Temperature and Volume.

Pressure

Pressure is the effect that occurs when a force is applied on a surface.


There are many units that measures pressure.
2
The SI units are the Newton/m and the Pascal (Pa).
More commonly, we use the notion of atmosphere (atm):

1 atm = 101325 Pa = 101.325 KPa


= 1.01325 bar
= 760 mm Hg = 760 torr

and 1 bar = 100000 Pa = 100 KPa

Conversion between units of pressure can be carried out using simple multiplications and
divisions. For example, 2.5 atm is equal to:

( 2.5 atm ) × 
101,325 Pa  5
 = 2.5 × 10 Pa
 1 atm 

Temperature

The Kelvin scale is a temperature scale where absolute zero, the theoretical absence of all
thermal energy, is zero kelvin (0 K). The kelvin is not referred to as a "degree", nor is it typeset
with a degree symbol; that is, it is written K and not °K.

Conversion from Kelvin to Celsius:


K = °C + 273.15

General Chemistry 202-NYA-05: Chapter 5 68


Gas stoichiometry:
Gas stoichiometry is the quantitative relationship between reactants and products in a chemical
reaction when it is employed for reactions that produce gases. Gas stoichiometry applies when
the gases produced are assumed to be ideal and the temperature, pressure, and volume of the
gases are all known.

An ideal gas is a theoretical gas composed of a set of randomly-moving particles that interact
only through collisions. The ideal gas concept is useful because it obeys the ideal gas law, a
simplified equation of state.

At normal ambient conditions such as standard temperature and pressure (STP), most real gases
behave qualitatively like an ideal gas.

The ideal gas law is used for these stoichiometric calculations. Often, but not always, the
standard temperature and pressure (STP) are taken as 0°C and 1 bar and used as the conditions
for gas stoichiometric calculations.

We can combine laws together into one comprehensive law for gases:

PV = nRT
where:
• P = absolute gas pressure
• V = gas volume
• n = number of moles
• R = universal ideal gas law constant
• T = absolute gas temperature (in K)

Gas Constant R
–1 –1
= 0.08206 L⋅atm⋅K ⋅mol
–1 –1
= 8.314 L⋅kPa⋅K ⋅mol
–1 –1
= 8.314 J⋅K ⋅mol

The simple conversion in the units gives a different numerical value of R:

0.08206 L⋅atm x 101.325 kPa = 8.314 L⋅kPa


K⋅mol atm K⋅mol

Gas stoichiometry calculations solve for the unknown volume (or mass) of a gaseous product or
reactant. For example, if we wanted to calculate the volume of gaseous NO2 produced from the
combustion of 100 g of NH3, by the reaction:

4NH3 (g) + 7O2 (g) → 4NO2 (g) + 6H2O (l)

we would carry out the following calculations:

100 g of NH3 x 1 mol of NH3 = 5.871 mol of NH3


17.034 g NH3

General Chemistry 202-NYA-05: Chapter 5 69


There is a 1:1 molar ratio of NH3 to NO2 in the above balanced combustion reaction, so 5.871 mol
of NO2 will be formed. We will employ the ideal gas law to solve for the volume at 0 °C (273.15
K) and pressure of 1 atmosphere using the gas law constant of R.
-1 -1
(R = 0.08206 L · atm · K · mol ) :

PV = nRT

Therefore:

V = nRT = 5.871 mol x 0.08206 L · atm · K-1 · mol-1 x 273.15 K = 131.59 L NO2
P 1 atm

Example:
What is the pressure in a 304.0 L tank that contains 5.670 kg of helium at 25°C?

Solution:

Data: n = mass / M
T = 25oC = 298K = 5670 g / 4.003 g⋅mol−1
V = 304.0 L = 1417.5 mol
P = ? atm
R = 0.08206 L·atm/mol·K PV = nRT
n=? P = nRT / V
MHe = 4.003 g/mol = 1417.5 mol ⋅ 0.08206 L·atm·mol−1·K−1 ⋅ 298K
massHe = 5.670 kg = 5670 g 304.0 L

= 114 atm

Gas stoichiometry often involves having to know the molar mass of a gas, given the density of
that gas. (The density of a material is defined as its mass per unit volume). The ideal gas law can
be re-arranged to obtain a relation between the density and the molar mass of an ideal gas:

ρ = m and n = m
V M

and thus:

ρ = MP
RT

where:
ρ = gas density at T and P
m = mass of gas
M = molar mass of gas

Do example in the book.

General Chemistry 202-NYA-05: Chapter 5 70


Closed System

A closed system is a system which is isolated so that it cannot exchange matter with its
surroundings. Therefore, the amount of moles doesn’t change, even if the Pressure, Volume or
Temperature of the system changes.

Initial Conditions (1) Final Conditions (2)


P1V1 = n1RT1 P2V2 = n2RT2
n1 = P1V1 n2 = P2V2
RT1 RT2

In a closed system, since n1 = n2


P1V1 = P2V2
T1 T2

The units of Pressure, Volume and Temperature must be the same for the comparison.
o
The units of Temperature must be in Kelvin (K), NOT Celsius ( C)
Even if the units can be cancelled out, the value of the temperature is wrong if done in Celsius.
o
Because the conversion from K to C is an addition/subtraction, and not a multiplication/division
as in the pressure units.

Example:
At what temperature (in °C) does 121 mL of CO2 at 27°C and 1.05 atm occupy a volume of 293
mL and a pressure of 1.40 atm?

Solution
Data P1V1 = P2V2 so T2 = P2V2T1
V1 = 121 mL T1 T2 P1V1
T1 = 27°C = 300K
P1 = 1.05 atm
T2 = 1.40 atm ⋅ 293 mL ⋅ 300K
V2 = 293 mL
1.05 atm ⋅ 121 mL
P2 = 1.40 atm?
= 969 K
T2 = ?
= (969 – 273)
= 696°C

Molar Mass of a Gas

Since PV = nRT and that Molar Mass is Mass of mole/Volume

V = RT
n P
You multiply this equation by the density and you get the equation for Molar Mass:
(Density is the mass per unit volume of a substance),
 g   L ⋅ atm 
dRT  L   mol ⋅ K 
(K ) g
Molar mass = = =
P ( atm ) mol

Where
d = density of gas R = universal gas constant
T = temperature in Kelvin P = pressure of gas

Problem:
What is the density of F2 at STP (in g/L)?
Answer: 1.70 g/L

General Chemistry 202-NYA-05: Chapter 5 71


Extra Problems:
1. Gaseous chlorine is held in two separate containers at identical temperature and pressure. The volume
of container 1 is 1.30 L, and it contains 6.70 mol of the gas. The volume of container 2 is 2.05 L.
How many moles of the gas are in container 2?
a) 10.6 mol
b) 17.9 mol
c) 0.398 mol
d) 4.25 mol
e) none of these

2. A balloon has a volume of 2.36 liters at 24.0°C. The balloon is heated to 48.0°C. Calculate the new
volume of the balloon.
a) 2.36 L
b) 2.55 L
c) 2.18 L
d) 4.72 L
e) 1.18 L

3. Consider a sample of gas in a container on a comfortable spring day. The Celsius temperature
suddenly doubles, and you transfer the gas to a container with twice the volume of the first container.
If the original pressure was 12 atm, what is a good estimate for the new pressure?
a) 3 atm
b) 5.5 atm
c) 6.4 atm
d) 12 atm
e) 15 atm

4. You are holding four identical balloons each containing 10.0 g of a different gas. The balloon
containing which gas is the largest balloon?
a) H2
b) He
c) Ne
d) O2
e) All have the same volume.

5. A gas sample is heated from -20.0°C to 57.0°C and the volume is increased from 2.00 L to 4.50 L. If
the initial pressure is 0.109 atm, what is the final pressure?
a) 0.0371 atm
b) –0.138 atm
c) 0.320 atm
d) 0.188 atm
e) 0.0632 atm
6. You fill a balloon with 2.50 moles of gas at 24°C at a pressure of 1.78 atm. What is the volume of the
balloon?
a) 17.4 L
b) 108 L
c) 34.2 L
d) 2.77 L
e) 22.4 L

General Chemistry 202-NYA-05: Chapter 5 72


7. A sample of helium gas occupies 19.5 L at 23°C and 0.956 atm. What volume will it occupy at 40°C
and 1.20 atm?
a) 25.9 L
b) 27.0 L
c) 14.7 L
d) 16.4 L
e) 19.7 L
8. A piece of 9.70g of solid CO2 (dry ice) is allowed to sublime in a balloon. The final volume of the
balloon is 1.00 L at 298 K. What is the pressure of the gas?
a) 5.39 atm
b) 2.37 × 102 atm
c) 2.52 atm
d) 0.186 atm
e) none of these

Ans: 1) a 2) b 3) c 4) a 5) e 6) c 7) d 8) a

General Chemistry 202-NYA-05: Chapter 5 73


General Chemistry 202-NYA-05: Chapter 5 74
Chapter 4:

Types of Chemical Reactions and Solution Stoichiometry

th
Suggested problems in chapter 4 of Zumdahl 8 Edition:

7, 9, 10, 12, 13, 15, 21, 23, 25, 27, 29, 31, 33, 35, 37, 41, 43, 45, 47, 51, 55, 65, 67, 71, 73, 77,
79, 81, 83, 87, 88.

th
Suggested problems in chapter 18 of Zumdahl 8 Edition:

15, 17, 29, 30, 31.


th
(or in chapter 4 of Zumdahl 7 Edition: 73, 74, 75)

General Chemistry 202-NYA-05: Chapter 4 75


4.1 Water the Common Solvent
Because of its unequal charge distribution, water is said to be a polar molecule.
It is this polarity that gives water its great ability to dissolve compounds.
Solutions in which water is the dissolving medium, or solvent, are called aqueous solutions.
Therefore, water is a polar solvent that dissolves many ionic and polar substances.

A schematic of an ionic solid dissolving in water is shown in the figure below.


Note that the “positive ends” of the water molecules are attracted to the
negatively charged anions and that the “negative ends” are attracted to the
positively charged cations. This process is called hydration. The hydration of its
ions tends to cause a salt to “fall apart” in the water, or to dissolve. The strong
forces present among the positive and negative ions of the solid are replaced by
strong water-ion interactions.

H2O(l)
NaCl(s) Na+(aq) + Cl (aq)

General Chemistry 202-NYA-05: Chapter 4 76


4.2 The Nature of Aqueous Solutions: Strong and Weak
Electrolytes
Definitions:
Mixture: When two or more different substances are mixed together but not combined chemically
to form only one new compound.

Homogeneous mixture: A mixture that is uniform throughout, and has only one phase (liquid).

Heterogeneous mixture: A mixture that is separated into different regions or phases that have
different compositions or properties, where its constituents are not uniformly dispersed.

Solute: A substance which is dissolved in a solvent, the component(s) in smaller quantity.

Solvent: The dissolving medium. A liquid (or gas) dissolving other solid, liquid or gas substances
(solutes). The component in major quantity. The solvent does not change its state in forming a
solution. (Aqueous solutions are solutions in which the solvent is water).

Electrical conductivity: Ability to conduct an electric current.

Electrolyte: Substance, that when dissolved in water produces a solution that can conduct
electricity. There are three types of electrolytes (strong-, weak-, and nonelectrolytes).

Strong Electrolytes
Strong electrolytes are substances that are completely ionized when they are dissolved in water.
(Example: NaCl, also called “table salt”)

We will consider three classes of strong electrolytes: Soluble salts, Strong acids and Strong
bases.

A salt consists of an array of cations and anions that separate and become hydrated when the
+
salt dissolves. For example, when NaCl dissolves in water, it produces hydrated Na and Cl− ions
in the solution. Virtually no NaCl units are present. Thus, NaCl is a strong electrolyte
+
An acid is a substance that produces H ions when it is dissolved in water.
+
HA(aq) + H2O(l) → H3O (aq) + A−(aq)

General Chemistry 202-NYA-05: Chapter 4 77


A strong acid is one that completely dissociates into its ions. Thus, if 100 molecules of HCl are
+
dissolved in water, 100 H ions and 100 Cl− ions are produced. Virtually no HCl molecules exist in
the aqueous solution.

When HCl, HNO3, and H2SO4 are placed in water, virtually every molecule ionizes. These
substances are strong electrolytes and are thus called strong acids.

Sulfuric acid, nitric acid, and hydrochloric acid are aqueous solutions and should be written in
chemical equations as H2SO4(aq), HNO3(aq), and HCl(aq), respectively, although they often
appear without the (aq) symbol.
+
Sulfuric acid is a special case. The formula H2SO4 indicates that this acid can produce two H
+
ions per molecule when dissolved in water. However, only the first H ion is completely
+
dissociated. The second H ion can be pulled off under certain conditions, which we will discuss
+
later. Thus an aqueous solution of H2SO4 contains mostly H ions and HSO4− ions.

Another important class of strong electrolytes consists of the strong bases, soluble ionic
compounds containing, in this case, the hydroxide ion (OH−). Common examples include NaOH
and KOH.

Weak Electrolytes
Weak electrolytes are substances that exhibit a small degree of ionization in water. They produce
relatively few ions when dissolved in water, and conduct only a small current. The most common
weak electrolytes are weak acids and weak bases.

Acetic acid is very different from the strong acids because only about 1% of its molecules
dissociate in aqueous solutions at typical concentrations. For example, for every 100 molecules
of acetic acid (HC2H3O2) originally dissolved in water, approximately 99 molecules of HC2H3O2
+
remain intact. Only one molecule dissociates into H and C2H3O2−. Since it is a weak electrolyte, it
is called a weak acid.

In a similar manner, weak bases are weak electrolytes, because they form very few ions of OH−.

Nonelectrolytes
Nonelectrolytes are substances that dissolve in water but do not produce any ions. Since the
molecules do not break up into ions, the resulting solution does not conduct an electric current.
Example: sucrose (table sugar, C12H22O11) and ethanol.

General Chemistry 202-NYA-05: Chapter 4 78


4.3 The Composition of Solutions
Concentration can be expressed with molarity (M), which is defined as moles of solute per
volume of solution in liters.

M = molarity = moles of solute


litres of solution

A solution that is 1.0 molar (written as 1.0 M) contains 1.0 mole of solute per litre of solution.

Example 1:
Calculate the molarity of a solution prepared by dissolving 10.8 g of solid KOH in enough water to
make 1.50L of solution.

Solution:
What are the moles of KOH (56.11 g/mol)?

10.8 g KOH x (1 mol KOH / 56.11 g KOH) = 0.192 mol KOH

What is the molarity of the solution?

Molarity = mol solute / L solution = 0.192 mol KOH / 1.50 L = 0.128 M KOH

Example 2:
Give the concentration of each type of ion in a 0.50 M Co(NO3)2 solution.

Solution:
What is the balance equation for dissolving the ions?
2+
Co(NO3)2(s) → Co (aq) + 2NO3− (aq)

What is the molarity for each ion?


2+ 2+
Co : 1 x 0.50 M = 0.50 M Co
NO3−: 2 x 0.50 M = 1.0 M NO3−

Makes sense:
Often, you will be asked to furnish the number of moles of solute present in a given volume of a
solution of known molarity. Since M = moles of solute / liters of solution :

liters of solution x molarity = liters of solution x moles of solute = moles of solute


liters of solution

General Chemistry 202-NYA-05: Chapter 4 79


Dilution
Standard solution: A solution whose concentration is accurately known.

Dilution: Process of adding a solvent (usually water) to achieve the molarity desired for a
particular solution.

Moles of solute after dilution = Moles of solute before dilution

Because only solvent (in many cases water) is added to accomplish the dilution, the number of
moles of the solute doesn’t change during a dilution.

Example:

What volume of 14 M sulphuric acid must be used to prepare 1.5 L of a 0.10 M H2SO4 solution?

Solution:

What are the moles of H2SO4 required?


M x V = mol
0.10 mol H2SO4 x 1.5 L solution = 0.15 mol H2SO4
L solution
What volume of 14 M H2SO4 contains 0.15 mol H2SO4?
V x 14 mol H2SO4 / L solution = 0.15 mol H2SO4
Solving V gives
V = 0.15 mol H2SO4 x 1 L solution = 1.1 x 10−2 L or 11 mL solution.
14 mol H2SO4

Another way to express the process of dilution is the following expression:

M1V1 = M2V2

Where M1 and V1 represent the molarity and volume of the original solution (before dilution) and
M2 and V2 represent the molarity and volume of the diluted solution.

M1 x V1 = mol solute before dilution = mol solute after dilution = M2 x V2

If you repeat the previous example using this equation, you will find:
M1 = 14 M M2 = 0.10 V2 = 1.5 L
By isolating V1, we find a value of 11 mL.

General Chemistry 202-NYA-05: Chapter 4 80


4.4Types of Chemical Reactions
Types of solution reactions viewed in this course:

• Precipitation reactions
• Acid-base reactions
• Oxidation-reduction reactions

4.5Precipitation Reactions
When two solutions are mixed, an insoluble substance sometimes forms; that is, a solid forms
and separates from the solution. Such a reaction is called a precipitation reaction. The solid
that forms is called a precipitate.

The (aq) attached to an ionic substance always means the component ions are in the correct
proportions, and that the ionic substance is soluble.
NaCl (aq) means the same as Na+ (aq) + Cl– (aq)
It is thus very important to recognize ionic compounds and to know what the component ions are.

When two different aqueous solutions are mixed, we have a “soup” consisting of all the ions
floating around in water. For example, a solution of an aqueous solution of sodium chloride mixed
with an aqueous solution of silver nitrate:

NaCl (aq) + AgNO3 (aq) → products


Is the same as:
Na+ (aq) + Cl– (aq) + Ag+ (aq) + NO3− (aq) → products

These atoms are bouncing around the container colliding with each other. If Na+ ions collide with
other Na+ ions, they will repel each other (they have the same charge). Likewise, if Ag+ ions
collide with Na+ ions or other Ag+ ions, they will repel each other. Similarly, any combination of
collisions between anions will be repulsive.

If the Na+ ions collide with Cl–, they will attract each other to form NaCl. However, since NaCl is
soluble, it will immediately break back up into ions. A similar situation arises when Ag+ collides
with NO3− ions.

Thus the only new things that can happen is for Na+ ions to collide with NO3− ions to form NaNO3
and for Ag+ to collide with Cl– to form AgCl.

It is an experimental fact that NaNO3 is soluble; therefore it cannot form a precipitate. As long as
water is present, NaNO3 remains dissolved as separated ions.

It is also known experimentally that AgCl is insoluble, therefore it will form a precipitate. When
ions form a solid compound, the compound must have a zero net charge. Thus the products of
this reaction must contain both anions and cations.

The Overall equation (or the Formula equation or the Complete Molecular equation) of this
reaction can be written as:

NaCl (aq) + AgNO3 (aq) → AgCl(s) + NaNO3(aq)

General Chemistry 202-NYA-05: Chapter 4 81


You have to be able to predict the formation of precipitates, according to the rules of solubility of
salts in water. When an insoluble salt (ionic compound) forms after two or more solutions are
mixed, the substance will precipitate out of solution. If the salt is soluble no precipitate will form.

Solubility Rules for Some Common Salts in Water

Soluble Salts
1. Most nitrate ( NO3− ), acetate ( C2 H 3O2− ), chlorate ( ClO3− ) and perchlorate ( ClO4− ) salts are
soluble.

2. Most salts containing the alkali metal ions (Li+, Na+, K+, Cs+, Rb+) and the ammonium ion
( NH 4+ ) are soluble.

3. Most chloride, bromide and iodide salts are soluble.


Notable exceptions: Salts containing the ions Ag+, Pb2+ , Cu+ and Hg 22+ .

4. Most sulfate salts ( SO42− ) are soluble.


Notable exceptions: Ca2+ , Sr2+ , Ba2+ and Pb2+ and Hg 22+ .

Insoluble Salts
1. All common metal hydroxides (OH–) are insoluble.
Notable exceptions: Hydroxides of Group IA metals, NH 4+ and the larger
members of
Group IIA metals (starting with Ca2+) are partially soluble

2. All common carbonates ( CO32− ), phosphates ( PO43− ) and chromates ( CrO42− ) are
insoluble.
Notable exceptions: Group IA metals and NH 4+

3. All common sulphides ( S 2− ) are insoluble.


Notable exceptions: Group IA metals, Group IIA metals and NH 4+

These solubility rules must be memorized. They will be used to predict whether a precipitate will
form.

General Chemistry 202-NYA-05: Chapter 4 82


4.6 Describing reactions in Solution
If we take for example the previous Complete Molecular Equation:

NaCl (aq) + AgNO3 (aq) → AgCl(s) + NaNO3(aq)

The Complete Ionic Equation represents the actual forms of the reactants and products in
solution. In a Complete Ionic equation, all substances that are strong electrolytes are represented
as ions. Thus, the Complete Ionic Equation:

Na+ (aq) + Cl– (aq) + Ag+ (aq) + NO3− (aq) → Na+(aq) + NO3– (aq) + AgCl(s)

Note that in the complete ionic equation, Na+ (aq) and NO3– (aq) appear on both sides of the
equation. This means that these two ions merely serve to ensure that overall, all charges are
balanced. They serve no other function in the chemical reaction. Ions which do not participate in
the chemical equation are known as spectator ions. Spectator ions are ions that appear on both
sides of the complete ionic equation.

If the spectator ions are cancelled as if they were variables in an algebraic equation, then we are
left with the Net Ionic Equation. The net ionic equation describes only those solution
components directly involved in the reaction (the basic chemical process):

Ag+ (aq) + Cl– (aq) → AgCl (s)

Examples:

Write the balanced molecular equation (M.E.), complete ionic equation (C.I.E.), and net ionic
equation (N.I.E.) for the reaction that occurs when the following aqueous solutions are mixed:

1. Sodium carbonate and hydrochloric acid


(M.E.) Na2CO3 (aq) + 2 HCl (aq) → 2 NaCl (aq) + CO2 (g) + H2O (l)
(C.I.E.) 2 Na+ (aq) + CO32− (aq) + 2 H+ (aq) + 2 Cl– (aq)

→ 2 Na+ (aq) + 2 Cl– (aq) + CO2 (g) + H2O (l)


(N.I.E.) 2 H+ (aq) + CO32− (aq) → CO2 (g) + H2O (l)

2. Potassium bromide and copper (II) nitrate


(M.E.) 2 KBr (aq) + Cu(NO3)2 (aq) → 2 KNO3 (aq) + CuBr2 (aq)
(C.I.E.) 2 K+ (aq) + 2 Br– (aq) + Cu2+ (aq) + 2 NO3− (aq)

→ 2 K+ (aq) + 2 Br– (aq) + Cu2+ (aq) + 2 NO3− (aq)

(N.I.E.) None

General Chemistry 202-NYA-05: Chapter 4 83


3. Sodium acetate and calcium chloride
(M.E.) 2 NaC2H3O2 (aq) + CaCl2 (aq) → Ca(C2H3O2)2 (aq) + 2 NaCl (aq)
(C.I.E.) 2 Na+ (aq) + 2 C2 H 3O2− (aq) + Ca2+ (aq) + 2 Cl– (aq)
→ 2 Na+ (aq) + 2 C2 H 3O2− (aq) + Ca2+ (aq) + 2 Cl– (aq)

(N.I.E.) None
4. Copper (II) sulfate and sodium hydroxide
(M.E.) CuSO4 (aq) + 2 NaOH (aq) → Cu(OH)2 (s) + Na2SO4 (aq)
(C.I.E.) Cu2+ (aq) + SO42− (aq) + 2 Na+ (aq) + 2 OH– (aq)

→ Cu(OH)2 (s) + 2 Na+ (aq) + SO42− (aq)

(N.I.E.) Cu2+ (aq) + 2 OH– (aq) → Cu(OH)2 (s)

4.7 Stoichiometry of Precipitation Reactions


The same principles of chemical stoichiometry apply to reactions that take place in solutions.
Special emphasis on the fact that:
You will have to analyze the chemical equation and determine which reaction will occur
when 2 (or more) solutions are mixed.
You must use the volume of solution and its molarity to determine quantities.

Solving Stoichiometry Problems for Reactions in Solution


1. Identify the species present in the combined solution, and determine what reaction if any
occurs.
2. Write the balanced net ionic equation for the reaction.
3. Calculate the moles of reactants.
4. Determine which reactant is limiting.
5. Calculate the moles of product(s), as required.
6. Convert to grams or other units, as required.

Example:
10.0 mL of a 0.30 M sodium phosphate solution reacts with 20.0 mL of a 0.20 M lead(II) nitrate
solution (assume no volume change).
What precipitate will form?
What mass of precipitate will form?

General Chemistry 202-NYA-05: Chapter 4 84


Solution:
What are the ions present in the combined solution?
+ 3− 2+
Na3PO4 + Pb(NO3)2 → Na + PO4 + Pb + NO3−

What is the balanced net ionic equation for the reaction?


Na3PO4 + Pb(NO3)2 → Pb3(PO4)2 + NaNO3

(M.E.): 2 Na3PO4 (aq) + 3 Pb(NO3)2 (aq) → Pb3(PO4)2 (s) + 6 NaNO3 (aq)


+ 3− 2+
(C.I.E): 6 Na (aq) + 2 PO4 (aq) + 3 Pb (aq) + 6 NO3− (aq) →
+
Pb3(PO4)2 (s) + 6 Na (aq) + 6 NO3− (aq)
3− 2+
(N.I.E): 2 PO4 (aq) + 3 Pb (aq) → Pb3(PO4)2 (s)

Lead(II) phosphate (Pb3(PO4)2) is the precipitate that will be formed.

To find the mass of solid Pb3(PO4)2 formed, we need to know what are the moles of reactants
present in the solution? Which reactant is limiting? What moles of Pb3(PO4)2 will be formed?
3−
0.010 L x 0.30 mol L−1 = 3.00 x 10−3 mol of PO4 (which will give 1.50 x 10−3 mol of Pb3(PO4)2)
−1 −3 2+
0.020 L x 0.20 mol L = 4.00 x 10 mol of Pb (which will give 1.33 x 10−3 mol of Pb3(PO4)2)

Therefore, the limiting reagent is Pb(NO3)2


It will give 1.33 x 10−3 mol of Pb3(PO4)2
1.33 x 10 mol of Pb3(PO4)2 x 811.62 g mol−1 = 1.1 g of Pb3(PO4)2
−3

What is the concentration of nitrate ions left in solution after the reaction is
complete?

To find the concentration of nitrate ions left in solution after the reaction is complete, we have to
determine what are the moles of nitrate ions present in the combined solution? What is the total
volume of the combined solution?

0.020 L x 0.20 mol L−1 x 2 per molecule = 8.00 x 10−3 mol of NO3−
Total volume = 0.010 L + 0.020 L = 0.030 L

8.00 x 10−3 mol of NO3− / 0.030 L = 0.27 M

What is the concentration of phosphate ions left in solution after the reaction is
complete?

To find the concentration of phosphate ions left in solution after the reaction is complete, we have
to determine what are the moles of phosphate ions present in the solution at the start of the
reaction? How many moles of phosphate ions were used up in the reaction to make the solid
Pb3(PO4)2? How many moles of phosphate ions are left over after the reaction is complete?
What is the total volume of the combined solution?
3−
0.010 L x 0.30 mol L−1 = 3.00 x 10−3 mol of PO4 at the beginning
−3 3−
2 x 1.33 x 10 mol of PO4 were used in the reaction (according to the limiting reagent quantity)
3−
3.00 x 10−3 mol − 2.66 x 10−3 mol = 0.33 x 10−3 mol of PO4 left in the solution.

3−
0.33 x 10−3 mol of PO4 / 0.03 L = 0.011 M

General Chemistry 202-NYA-05: Chapter 4 85


4.8 Acid-Base Reactions
Arrhenius definitions:
• Acids: Substances which, when dissolved in water, produce H+ ions.
• Bases: Substances which, when dissolved in water produce OH– ions.

Strong acids and strong bases are acids that are excellent conductors of electricity.
The list of strong acids (to be memorized) is: HCl, HBr, HI, HNO3, H2SO4, and HClO4
The strong bases are: the alkali metal hydroxides and the alkaline earth hydroxides.

Bronsted-Lowry more general definitions:


+
• Acids: Proton (H ) donor.
+
• Bases: Proton (H ) acceptor.

Dissociation of acids
As with other ionic substances, acids dissociate into their component ions when dissolve in water.
However, the dissociation involves the removal of only one of the protons to produce H+ and the
conjugate acid anion.

Example:
H2SO4 (aq) → H+ (aq) + HSO4− (aq) NOT [ H2SO4 (aq) → 2 H+ (aq) + SO42− (aq) ]

H3PO4 (aq) → H+ (aq) + H 2 PO42− (aq)

H2S (aq) → H+ (aq) + HS − (aq)


Obviously this is not an issue for monoprotic acids (acids having only one ionizable proton, e.g.,
HCl, HNO3, etc.)

Acid-Base Reactions
Acid-Base reactions (reactions between acids and bases) are sometimes referred to as
neutralization reactions. When just enough base is added to react exactly with the acid in a
solution, we say the acid has been neutralized. The product of neutralization reactions are always
a salt (an ionic compound) and water. For example, when mixing HCl and NaOH solutions:

Complete Molecular Equation: HCl (aq) + NaOH (aq) → NaCl (aq) + H2O (l)
Complete Ionic Equation: [H+ (aq) + Cl– (aq)] + [Na+ (aq) + OH– (aq)]
→ Na+ (aq) Cl– (aq) + H2O (l)
Net Ionic Equation: H+ (aq) + OH– (aq) → H2O (l)

This is the net ionic equation for all acid-base reactions.

+
Because water is a nonelectrolyte, large quantities of H and OH− ions cannot coexist in
solutions. They react to form H2O molecules.
Example: H2SO4 (aq) + NaOH (aq) → ?
Ans: H2SO4 (aq) + 2 NaOH (aq) → 2 H2O (l) + Na2SO4 (aq)
Note that Acid-Base reactions are double displacement reactions.

General Chemistry 202-NYA-05: Chapter 4 86


The hydroxide ion is such a strong base that for purposes of stoichiometric calculations it can be
assumed to react completely with any weak acid that we will encounter. (OH− ions also react
+
completely with the H ions in solutions of strong acids).

Performing Calculations for Acid–Base Reactions


1. List the species present in the combined solution before any reaction occurs, and decide
what reaction will occur.
2. Write the balanced net ionic equation for this reaction.
3. Calculate moles of reactants.
4. Determine the limiting reactant, where appropriate.
5. Calculate the moles of the required reactant or product.
6. Convert to grams or volume (of solution), as required.

Example 1:
What volume of a 0.100 M HCl solution is needed to neutralize 25.0 mL of 0.350 M NaOH?

Solution 1:
+ –
The chemical reaction is : H (aq) + OH (aq) → H2O (l)

1) What are the ions present in the combined solution?


+ – + –
H Cl Na OH
+ –
Two chemical reactions are possible: Na (aq) + Cl (aq) → NaCl (s)
+ –
H (aq) + OH (aq) → H2O (l)
Since we know that NaCl is soluble, the first reaction does not take place (Na+ and Cl− are
+
spectator ions). However, as we have seen before, the reaction of the H and OH− ions to form
H2O occurs.

2) The balanced net ionic equation for the reaction is:


+ –
H (aq) + OH (aq) → H2O (l)

3) What are the moles of reactant present in the solution?


– –3 –
25.0 mL NaOH x 1L x 0.350 mol OH = 8.75 x 10 mol OH
1000 mL L NaOH

4) Which reactant is limiting?


+
This problem requires the addition of just enough H to react exactly with the OH− ions present.
We do not need to be concerned with limiting reactant here.
+
5) What moles of H are needed?
+ +
Since H and OH− ions react in a 1:1 ratio, 8.75 x 10−3 mol H is required to neutralize the OH−
ions present.

6) What Volume of HCl is required?


+ +
V x 0.100 mol H = 8.75 x 103 mol H
L
+
Solving for V gives: V = 8.75 x 10−3 mol H = 8.75 x 10−2 L
+
0.100 mol H
L

General Chemistry 202-NYA-05: Chapter 4 87


Example 2:

In a certain experiment, 28.0 mL of 0.250 M HNO3 and 53.0 mL of 0.320 M KOH are mixed.
+
Calculate the amount of water formed in the resulting reaction. What is the concentration of H or

OH ions in excess after the reaction goes to completion?

Solution 2:

1) What are the ions present in the combined solution?


+ – + –
H NO3 K OH

2) The balanced net ionic equation for the reaction is:


+ –
H (aq) + OH (aq) → H2O (l)

3) What are the moles of reactant present in the solution?


+ –3 +
28.0 mL HNO3 x 1L x 0.250 mol H = 7.00 x 10 mol H
1000 mL L HNO3
–2
53.0 mL KOH x 1L x 0.320 mol OH− = 1.70 x 10 mol OH−
1000 mL L HNO3

4) Which reactant is limiting?


+ +
Since H and OH− ions react in a 1:1 ratio, the limiting reactant is H .
–3
Therefore, 7.00 x 10 mol H2O will be formed.
–3
7.00 x 10 mol H2O x 18 g/mol = 0.126 g H2O = 0.126 mL H2O formed.

5) What amount of OH− will react?


–3 +
7.00 x 10 mol OH− is required to neutralize the H ions present.

What amount of OH− ions is in excess? The amount of OH− ions in excess is obtained from the
following difference:

Original amount − amount consumed = amount in excess


1.70 x 10−2 mol OH− − 7.00 x 10−3 mol OH− = 1.00 x 10−2 mol OH−

What is the volume of the combined solution? It is the sum of the individual volumes:
Original volume of HNO3 + original volume of KOH = total volume
28.0 mL + 53.0 mL = 81.0 mL = 8.10 x 10−2 L
The volume of water created by the reaction is negligible.

6) What is the molarity of the ions in excess?


Mol OH− = 1.00 x 10−2 mol OH− = 0.123 M OH−
L of solution 8.10 x 10−2 L

General Chemistry 202-NYA-05: Chapter 4 88


Acid-Base Titrations
Titration : Delivery of a measured volume of a solution of known concentration (the titrant) into a
solution containing the substance being analyzed (the analyte). In other words, titration is a
method for measuring the concentration of an unknown solution (analyte) using the known
concentration of another solution (the standard solution).
Indicator: A substance added at the beginning of the titration that changes color at the
equivalence point.
Equivalence point : Enough titrant added to react exactly with the analyte.
Endpoint : The indicator changes color so you can tell the equivalence point has been reached.
Neutralization reactions: Acid-base reactions in which the number of moles of removable
protons in an acid is balanced by an equivalent number of moles of OH–. The actual calculation
for neutralization reactions is identical as for other solution stoichiometric calculations.
In the case of an acid-base reactions, we:
1. Carry out an experiment in which an analyte is
neutralized by a standard solution.
• Measure a precise volume of analyte (using a Burette: contains
pipette). the titrant
• Add enough standard solution so that the analyte is
just neutralized (an indicator such as
phenolphthalein is often used to signal that the
reaction is complete; phenolphthalein is colourless
in acid solution and pink in basic solution. Therefore
a few drops of phenolphthalein is added to the Erlenmeyer
analyte and the standard solution is added until the Flask:
colour of the analyte just changes upon the addition contains the
of one drop. The error is then the volume of one analyte
drop).
2. Using the given concentration of the standard solution
and the volume of titrant added, the number of moles of
analyte can be obtained.
3. The concentration of the analyte may then be calculated by dividing this number of moles
by the volume of analyte (that was pipetted).

Example 1: What volume of 0.0962 M HCl solution would be required to neutralize 10.00 mL of
0.0352 M NaOH solution?
Solution 1:
Calculate the number of moles given: nNaOH = (0.0352 mol ⋅ L−1 )(0.01000 L) = 3.52 × 10−4 mol

For neutralization reactions, there is no limiting reactant


 1 mol HCl 
nNaOH = 3.52 × 10−4 mol NaOH  −4
 = 3.52 × 10 mol HCl
 1 mol NaOH 
–1
Calculate the volume of the original HCl solution that would deliver 3.52 × 10 mol.
3.52 × 10−4 mol
VHCl = = 0.00366 L = 3.66 mL
0.0962 mol ⋅ L−1

General Chemistry 202-NYA-05: Chapter 4 89


Example 2: What volume of 0.4056 M NaOH solution would be required to neutralize 25.00 mL of
0.1455 M H3PO4 solution?
NB: There are three removable H+ in phosphoric acid.
Solution 2: Chemical Equation:
3 NaOH (aq) + H3PO4 (aq) → Na3PO4 (aq) + 3 H2O (l)
Calculate the number of moles given:
nH PO = (0.1455 mol ⋅ L−1 )(0.02500 L) = 3.638 × 10−3 mol
3 4

For neutralization reactions, there is no limiting reactant

 3 mol NaOH 
nNaOH = 3.638 × 10−3 mol H 3 PO4  −2
 = 1.091×10 mol NaOH
 1 mol H 3 PO4 
Calculate the volume of the original NaOH solution that would deliver 1.092 × 10–2 mol
1.091× 10−2 mol
VNaOH = = 0.02690 L = 26.90 mL
0.4056 mol ⋅ L−1

Example 3: If 27.23 mL of 0.0492 M NaOH is needed to neutralize 25.00 mL of an unknown acid,


calculate the concentration of the unknown acid?

Solution 3: NB: Since we do not know the number of ionisable protons in the acid, we will simply
use the net ionic equation of acid-base reactions
Chemical Equation: H+ (aq) + OH– (aq) → H2O (l)

Calculate the number of moles given:

 1 mol OH − 
nOH = (0.0492 mol NaOH ⋅ L−1 )(0.02723 L) 

−3
 = 0.0013397 mol = 1.34 × 10 mol
 1 mol NaOH 

For neutralization reactions, there is no limiting reactant


nH + = nOH − = 1.34 ×10−3 mol
–3
Calculate the volume of the original acid solution that would deliver 1.34 × 10 mol,
based on one ionisable proton.
1.34 ×10−3 mol
[ HA] = = 0.0536 M
0.02500 L
NOTE: This is a standard type of calculation in solution chemistry: from the
known concentration of one substance, determine the concentration
of another.

General Chemistry 202-NYA-05: Chapter 4 90


Extra Problems:

1. A 17.0-g sample of HF is dissolved in water to give 2.0 × 102 mL of solution. The concentration of
the solution is:
a) 0.85 M
b) 0.17 M
c) 0.09 M
d) 4.2 M
e) 8.5 M

2. 1.00 mL of a 3.95 × 10–4 M solution of oleic acid is diluted with 9.00 mL of petroleum ether, forming
solution A. Then 2.00 mL of solution A is diluted with 8.00 mL of petroleum ether, forming solution
B. What is the concentration of solution B?
a) 3.95 × 10–6 M
b) 1.10 × 10–5 M
c) 7.90 × 10–5 M
d) 8.78 × 10–5 M
e) 7.90 × 10–6 M

3. How many grams of NaCl are contained in 350. mL of a 0.334 M solution of sodium chloride?
a) 19.5 g
b) 6.83 g
c) 13.66 g
d) 116.9 g
e) none of these

4 What mass of calcium chloride, CaCl2, is needed to prepare 3.650 L of a 1.75 M solution?
a) 231 g
b) 6.39 g
c) 53.2 g
d) 111 g
e) 709 g

5. For the reaction 4FeCl2(aq) + 3O2(g) → 2Fe2O3(s) + 4Cl2(g), what volume of a 0.890 M solution of
FeCl2 is required to react completely with 8.71 × 1021 molecules of O2?
a) 4.49 × 103 mL
b) 17.2 mL
c) 12.2 mL
d) 21.7 mL
e) 9.65 mL

6. Phosphoric acid, H3PO4, is a triprotic acid. What is the total number of moles of H+ available for
reaction in 3.50 L of 0.400 M H3PO4?
a) 0.133 mole
b) 1.20 mole
c) 0.467 mole
d) 3.00 moles
e) 4.20 moles

General Chemistry 202-NYA-05: Chapter 4 91


7. Which of the following compounds is soluble in water?
a) Ni(OH)2
b) K3PO4
c) BaSO4
d) CoCO3
e) PbCl2

8. A 1.57-g sample of a metal chloride, MCl2, is dissolved in water and treated with excess aqueous
silver nitrate. The silver chloride that formed weighed 3.47 g. Calculate the molar mass of M.
a) 70.9 g/mol
b) 29 g/mol
c) 58.9 g/mol
d) 65 g/mol
e) 72.4 g/mol

9. You have 135.8 mL of a 2.50 M solution of Na2CrO4(aq). You also have 125 mL of a 2.50 M
solution of AgNO3(aq). Calculate the concentration of Na+ after the two solutions are mixed together.
a) 0.00 M
b) 1.30 M
c) 2.60 M
d) 5.00 M
e) 0.679 M

10. Sulfamic acid, HSO3NH2 (molar mass = 97.1 g/mol), is a strong monoprotic acid that can be used to
standardize a strong base:
HSO3 NH 2 (aq ) + KOH(aq ) → KSO 2 NH 2 (aq ) + H 2 O(l )

A 0.177-g sample of HSO3NH2 required 19.4 mL of an aqueous solution of KOH for a complete
reaction. What is the molarity of the KOH solution?
a) 0.00182 M
b) 9.40 M
c) 0.0940 M
d) 0.0354 M
e) none of these

11. A student weighs out 0.681 g of KHP (molar mass = 204.22 g/mol) and titrates to the equivalence
point with 36.78 mL of a stock NaOH solution. What is the concentration of the stock NaOH
solution? KHP is an acid with one acidic proton.
a) 0.00333 M
b) 0.123 M
c) 0.0185 M
d) 0.0907 M
e) none of these

12. True or False? A chemical that changes color at the endpoint of a reaction is called a colorimeter.
13. True or False? An acid is a substance that produces OH– ions in water.
14. True or False? The concentration of a salt water solution that sits in an open beaker decreases over
time.
15. True or False? The filtrate is the solid formed when two solutions are mixed.

Ans: 1) d 2) e 3) b 4) e 5) d 6) e 7) b 8) c 9) c 10) c 11) d 12) F 13) F 14) F 15) F

General Chemistry 202-NYA-05: Chapter 4 92


4.9 Oxidation-Reduction Reactions
We have seen many reactions forming ionic products. However, we have to carefully look at the
phase in which the products are formed. For example:

2Na(s) + Cl2(g) → 2NaCl(s)

It is NOT a precipitation reaction, even if a solid product is formed, because it is simply not carried
out in solution (nothing precipitates). In this reaction, solid sodium (which contains neutral sodium
atoms) reacts with chlorine gas (which contains diatomic Cl2 molecules) to form the ionic solid
+
NaCl (which contains Na and Cl− ions).

Reactions where one or more electrons are transferred are called oxidation-reduction reaction
(or redox reactions). Common examples of redox reactions include photosynthesis, oxidation of
sugars, fats, proteins and combustion reactions.

If we consider another example:


CH4(g) + 2O2(g) → CO2 + 2H2O(g) + energy

Even though none of the reactants or products in this reaction is ionic, the reaction is still
assumed to involve a transfer of electrons from carbon to oxygen.

Oxidation States (Numbers)


The oxidation state (or oxidation number) is the charges an atom has within a substance. They
are used in two ways:
To determine the charge of a cation in an ionic compound so that the compound can be
named (as seen earlier).
To determine how many electrons have been transferred from one substance to another
during a chemical reaction (to keep track of electrons in redox reactions).

Recall that electrons are shared by atoms in covalent bonds. The oxidation states of atoms in
covalent compounds are obtained by arbitrarily assigning the electrons (which are actually
shared) to particular atoms. We consider that for a covalent bond between two identical atoms,
the electrons are split equally between the two. In cases where two different atoms are involved
(and the electrons are thus shared unequally), the shared electrons are assigned completely to
the atom that has the stronger attraction for electrons. For ionic compounds containing
monoatomic ions, the oxidation states of the ions are equal to the ion charges.

Oxidation numbers must usually be calculated because most atoms are capable of having
several oxidation states. The sum of the oxidation states must be zero for an electrically neutral
compound. The following rules for determining the oxidation state of an atom must be memorized
in the sequence that they are given below.

RULES FOR DETERMINING THE OXIDATION NUMBER


1 The sum of the oxidation number of each atom in a substance is equal to the overall charge
of the substance.
1.1 The oxidation number of a monatomic ion is the charge of the ion.
1.2 The oxidation number of an atom in its elemental form (e.g., Na, O2 , etc.) is zero.

General Chemistry 202-NYA-05: Chapter 4 93


2 Atoms of Group IA and Group IIA in a compound always have an O.N. = its Group number.
3 Fluorine in compounds always has O.N. = –1. 3+
Al
4 In compounds: Ag always has (+1)
2+ 3+
Zn always has (+2) Zn Ga
+ 2+ 3+
Cd always has (+2) Ag Cd In

5 Hydrogen in compounds has O.N. = + 1 except for hydrides (hydrogen bonded to metals)
where O.N. = –1.
6 Oxygen in compounds have O.N. = –2 except for peroxides (O.N. = –1) or superoxides (O.N.
= –½).
7 For Group VIIA (the halogens) in compounds, O.N. = –1 when combined with metals, non-
metals (except O) and other halogens lower in the Group VIIA.
8 When necessary, use the group number of the main group nonmetals (Groups V – VII) to
assign the oxidation state to the atom in a compound as follows:
O.N. = Group number – 8
Thus, for example, if it is necessary to assign the O.N. for P, a Group V atom, in a
compound (and it cannot be calculated) then it is assigned by:
O.N. of P = 5 – 8 = –3 so that it is P3–.
The oxidation number of all other atoms must be calculated.

Exercises: Determine the oxidation numbers for each atom in the following substances:
1. HNO2 H O 6. SO32− H O
2. CH3COOH H C 7. CH3CHO H C
3. PCl5 H O H 8. CO2
H H
4. SF6
5. NH 4+

General Chemistry 202-NYA-05: Chapter 4 94


The Characteristics of Oxidation-Reduction Reactions

Oxidation-reduction reactions are characterized by a transfer of electrons. In some cases, the


transfer occurs in a literal sense to form ions, such as in the reaction:

2 Na(s) + Cl2(g) → 2NaCl(s)

However, sometimes the transfer is less obvious. For example:

CH4(g) + 2O2(g) CO2(g) + 2H2O(g)

oxidation state: -4 +1 0 +4 -2 +1 -2
(each H) (each O)(each H)

In this reaction there are no ionic compounds, but we can still describe the process in terms of a
transfer of electrons. Note that carbon undergoes a change in oxidation state from −4 in CH4 to +
4 in CO2. Such a change can be accounted for by a loss of eight electrons (the symbol e− stands
for an electron):

CH4 CO2 + 8e

oxidation state: -4 +4

On the other hand, each oxygen changes from an oxidation state of 0 in O2 to −2 in H2O and
CO2, signifying a gain of two electrons per atom. Since four oxygen atoms are involved, this is a
gain of eight electrons:

2O2 + 8e CO2(g) + 2H2O(g)

oxidation state: 0 2( 2) 2( 2) = 8

Oxidation is an increase in oxidation state (a loss of electrons)


Reduction is a decrease in oxidation state (a gain of electrons)

Thus, in the reaction :

2Na(s) + Cl2(g) 2NaCl(s)

oxidation state: 0 0 +1 1

Sodium is oxidized and chlorine is reduced. In addition, Cl2 is called the oxidizing agent (an
electron acceptor), and Na is called the reducing agent (an electron donor).

General Chemistry 202-NYA-05: Chapter 4 95


So in the previous reaction:
CH4(g) + 2O2(g) CO2(g) + 2H2O(g)

oxidation state: -4 +1 0 +4 -2 +1 -2
(each H) (each O)(each H)

• Methane (CH4) is oxidized because there has


been an increase in carbon’s oxidation state (the
carbon atom has formally lost electrons).
• Oxygen is reduced because there has been a
decrease in its oxidation state (oxygen has
formally gained electrons).
• Methane (CH4) is the reducing agent.
• O2 is the oxidizing agent.
• When the oxidizing or reducing agent is named,
the whole compound is specified, not just the
element that undergoes the change in oxidation
state.

4.10 Balancing Oxidation-Reduction Equations

Balancing Oxidation–Reduction Reactions by Oxidation States


1. Write the unbalanced equation.
2. Determine the oxidation states of all atoms in the reactants and products.
3. Show electrons gained and lost using “tie lines.”
4. Use coefficients to equalize the electrons gained and lost.
5. Balance the rest of the equation by inspection.
6. Add appropriate states.

Example:
Balancing the reaction between solid zinc and aqueous hydrochloric acid that produces aqueous
zinc(II) chloride and hydrogen gas.

Solution:
1. What is the unbalanced equation?

2+ –
Zn(s) + HCl(aq) → Zn (aq) + Cl (aq) + H2(g)

General Chemistry 202-NYA-05: Chapter 4 96


2. What are the oxidation states for each atom?

2+ –
Zn(s) + HCl(aq) → Zn (aq) + Cl (aq) + H2(g)
0 +1 –1 +2 –1 0

3. How are electrons gained and lost?


1 e gained (each atom)

2+ –
Zn(s) + HCl(aq) → Zn (aq) + Cl (aq) + H2(g)
0 +1 –1 +2 –1 0


2 e lost

The oxidation state of chlorine remains unchanged.

4. What coefficients are needed to equalize the electrons gained and lost?

2+ –
Zn(s) + 2HCl(aq) → Zn (aq) + Cl (aq) + H2(g)

5. What coefficients are needed to balance the remaining elements? (All ionic or all molecular,
not ½ & ½ )
2+ –
Zn(s) + 2HCl(aq) → Zn (aq) + 2Cl (aq) + H2(g)

18.1 Balancing Oxidation-Reduction Equations

Review of Terms
• Oxidation–reduction (redox) reaction: Transfer of electrons from the reducing agent to the
oxidizing agent
• Oxidation: Loss of electrons
• Reduction: Gain of electrons
• Reducing agent: Electron donor
• Oxidizing agent: Electron acceptor

Half Reactions
The overall reaction is split into two half–reactions, one involving oxidation and one reduction.
For example, in the following reaction:
+ 2+ 2+ 3+
8 H + MnO4− + 5 Fe → Mn + 5 Fe + 4 H2O
The two half-reactions can be represented by
+ 2+
The Reduction: 8 H + MnO4− + 5e− → Mn + 4 H2O
2+ 3+ −
The Oxidation: 5 Fe → 5 Fe + 5e

General Chemistry 202-NYA-05: Chapter 4 97


Aqueous Redox reactions can occur in acidic solutions or in basic solutions. When this is the
+
case, attention must be given to H2O, H and OH−.

The Half–Reaction Method for Balancing Equations for Oxidation–Reduction Reactions


Occurring in Acidic Solution

1. Write separate equations for the oxidation and reduction half–reactions.


2. For each half–reaction:
A. Balance all the elements except H and O.
B. Balance O using H2O.
+
C. Balance H using H .
D. Balance the charge using electrons.
3. If necessary, multiply one or both balanced half–reactions by an integer to equalize the
number of electrons transferred in the two half–reactions.
4. Add the two half–reactions together, and cancel identical species.
5. Check that the elements and charges are balanced.

General Chemistry 202-NYA-05: Chapter 4 98


Example:
Balance the following equation.

2− 2− 3+ 2−
Cr2O7 (aq) + SO3 (aq) → Cr (aq) + SO4 (aq)

Solution:
Separate into half-reactions.
Balance elements except H and O.
2− 3+
Reduction: Cr2O7 (aq) → 2Cr (aq)
2− 2−
Oxidation: SO3 (aq) → SO4 (aq)

Determine the amount of electrons being exchange for each reaction.

2− 3+
6e− + Cr2O7 (aq) → 2Cr (aq)
2− 2−
SO3 (aq) → SO4 (aq) + 2e−

Balance the oxygen atoms by adding H2O on the other side of the reaction.
2− 3+
6e− + Cr2O7 (aq) → Cr (aq) + 7 H2O
2− 2−
H2O + SO3 (aq) → SO4 (aq) + 2e−

+
Balance the hydrogen atoms by adding H on the other side of the reaction.
+ 2− 3+
14 H + 6e− + Cr2O7 → 2 Cr + 7 H2O
2− 2− − +
H2O + SO3 → SO4 + 2e + 2H

Balance the electrons by multiplying the coefficients of the equation by the appropriate proportion.

+ 2− 3+
1x [14H + 6e− + Cr2O7 → 2 Cr + 7 H2O]
2− 2− − +
3x [H2O + SO3 → SO4 + 2e + 2H ]

Add both equations.

+ 2− 2− 3+ 2− +
14H + 6e− + Cr2O7 + 3 H2O + 3 SO3 → 2Cr + 7 H2O + 3 SO4 + 6e− + 6H

Cancel the recurring terms.

+ 2− 2− 3+ 2−
8H + Cr2O7 + 3 SO3 → 2Cr + 4 H2O + 3 SO4

+
This reaction occurs in an acidic solution, since there are H in the final balanced equation.

Chapter 18.1 Balancing Oxidation-Reduction Equations

Make sure to do the suggested Problems in the textbook.


Page 861-862 no: 15, 17, 29, 30, 31.

General Chemistry 202-NYA-05: Chapter 4 99


The Half–Reaction Method for Balancing Equations for Oxidation–Reduction Reactions
Occurring in Basic Solution

1. Use the half–reaction method as specified for acidic solutions to obtain the final balanced
equation as if H+ ions were present.
– +
2. To both sides of the equation, add a number of OH ions that is equal to the number of H
+
ions. (We want to eliminate H by forming H2O.)
+ –
3. Form H2O on the side containing both H and OH ions, and eliminate the number of H2O
molecules that appear on both sides of the equation.
4. Check that elements and charges are balanced.

Example:
What is the balanced redox reaction in basic solution of the previous example?
+ 2− 2− 3+ 2−
8 H + Cr2O7 + 3 SO3 → 2Cr + 4 H2O + 3 SO4

Solution:
+ 2− 2− 3+ 2−
8OH− + 8 H + Cr2O7 + 3 SO3 → 2Cr + 4 H2O + 3 SO4 + 8OH−
2− 2− 3+ 2−
8H2O + Cr2O7 +3 SO3 → 2Cr + 4 H2O + 3 SO4 + 8OH−

The same reaction described in basic media:


2− 2− 3+ 2−
4H2O + Cr2O7 + 3 SO3 → 2Cr + 3 SO4 + 8OH−

Extra Problems
1. In the following reaction, which species is oxidized?
8NaI + 5H2SO4 → 4I2 + H2S + 4Na2SO4 + 4H2O
a) sodium
b) iodine
c) sulfur
d) hydrogen
e) oxygen
2. How many of the following are oxidation-reduction reactions?
NaOH + HCl → NaCl + H2O
Cu + 2AgNO3 → 2Ag + Cu(NO3)2
Mg(OH)2 → MgO + H2O
N2 + 3H2 → 2NH3
a) 0
b) 1
c) 2
d) 3
e) 4
3. True or False? Oxidation is the gain of electrons.
4. True or False? A reducing agent is an electron donor.

General Chemistry 202-NYA-05: Chapter 4 100


5. In the reaction shown below, what species is oxidized?
2NaI + Br2 → 2NaBr + I2
a) Na+
b) I–
c) Br2
d) Br–
e) I2
6. Balance the following oxidation-reduction reaction using the oxidation number method:
Fe3+ + I– → Fe2+ + I2
In the balanced equation, the coefficient of Fe2+ is
a) 1
b) 2
c) 3
d) 4
e) none of these

7. In which of the following does nitrogen have an oxidation state of +4?


a) HNO3
b) NO2
c) N2O
d) NH4Cl
e) NaNO2
8. The oxidation state of iodine in IO3– is:
a) 0
b) +3
c) –3
d) +5
e) –5
9. Balance the following oxidation-reduction reaction using the oxidation number method:
Al + Br2 → Al3+ + Br–
In the balanced equation, the coefficient of Br– is
a) 2
b) 3
c) 4
d) 6
e) none of these

10. The MnO4– is often used to analyze for the Fe2+ content of an aqueous solution via the reaction
MnO4–(aq) + Fe2+(aq) + H+(aq) → Fe3+(aq) + Mn2+(aq) + H2O(l)
What is the ratio of Fe2+ : MnO4– in the balanced equation?
a) 1:1
b) 2:1
c) 3:1
d) 4:1
e) 5:1

General Chemistry 202-NYA-05: Chapter 4 101


11. Given the following reaction in acidic media:
Fe2+ + Cr2O72– → Fe3+ + Cr3+
The coefficient for water in the balanced reaction is:
a) 1
b) 3
c) 5
d) 7
e) none of these
12. For the reaction of sodium bromide with chlorine gas to form sodium chloride and bromine, the
appropriate half-reactions are (ox = oxidation and re = reduction):
a) ox: Cl2 + 2e– → 2Cl–; re: 2Br– → Br2 + 2e–
– –
b) ox: 2Br → Br2 + 2e ; re: Cl2 + 2e– → 2Cl–
– –
c) ox: Cl + e → Cl ; re: Br → Br– + e–
– 2–
d) ox: Br + 2e → Br ; re: 2Cl– → Cl2 + 2e–
e) ox: 2Na + 2e → 2Na; re: 2Cl– → Cl2 + 2e–
+ –

13. How many electrons are transferred in the following reaction when it is balanced in acidic solution?
SO32–(aq) + MnO4–(aq) → SO42–(aq) + Mn2+(aq)
a) 6
b) 2
c) 10
d) 5
e) 3
14. Ammonium metavandate reacts with sulfur dioxide in acidic solution as follows (hydrogen ions and
H2O omitted): The ratio x : y is
xVO3– + ySO2 → xVO2+ + ySO42–
a) 1:1
b) 1:2
c) 2:1
d) 1:3
e) 3:1

Use the following to answer questions 15-16. The following reaction occurs in aqueous acid solution:
– – –
NO3 + I → IO3 + NO2

15. In the balanced equation the coefficient of NO3– is:


a) 2
b) 3
c) 4
d) 5
e) 6

16. In the balanced equation the coefficient of water is:


a) 1
b) 2
c) 3
d) 4
e) 5

Ans: 1)b 2)c 3)F 4)T 5)b 6)b 7)b 8)d 9)d 10)e 11)d 12)b 13)c 14)c 15)e 16)c

General Chemistry 202-NYA-05: Chapter 4 102


Chapter 7:

Atomic Structure and Periodicity

th
Suggested problems in chapter 7 of Zumdahl 8 Edition:

37, 39, 41, 43, 45, 47, 49, 51, 53, 55, 57, 59, 63, 65, 67, 69, 71, 73, 75, 79, 81, 87, 91, 93, 99,
103, 109, 111, 115.

General Chemistry 202-NYA-05: Chapter 7 103


In this chapter, we will see that the modern theory of atomic structure accounts for periodicity in
terms of the electron arrangements in atoms.

In the first 30 years of the twentieth century, a radical new theory called “quantum mechanics”
was developed to account for the behaviour of light and atoms. This “new physics” provides many
surprises for humans who are used to the macroscopic world, but it seems to account flawlessly
for the behaviour of matter.

7.1Electromagnetic Radiation
Electromagnetic radiation is the properties of light. It is one of the ways that energy travels
through space.

Example of electromagnetic radiations: sunlight, microwave oven energy, dentist X-ray, heat from
a fireplace, coloured fireworks, etc.
Although these forms of radiant energy seem quite different, they all exhibit the same type of
wavelike behaviour, and travel at the speed of light in a vacuum.

Primary characteristics of waves:

• Wavelength (λ) – The distance between two consecutive peaks in a wave. (The distance
the wave must travel to repeat itself). In the SI system of units, it is measured in meters,
m (or in our context, nanometers, nm = 10–9 m).
• Frequency (ν) – The number of waves (cycles) per second that pass a given point in
space. (The number of wavelengths traveled by the wave in one second). In the SI
system of units, it is measured in s–1, which is equal to Hertz (Hz).
8
• Speed (c) – speed of light (2.9979×10 m/s)

c = λν

Note that the wave with the


shortest wavelength (λ3) has the
highest frequency and the wave
with the longest wavelenght (λ1)
has the lowest frequency. This
implies an inverse relationship
between wavelength and
frequency.

General Chemistry 202-NYA-05: Chapter 7 104


Classification of Electromagnetic Radiation
For example:
The energy from the sun reaches the earth mainly in the form of visible and ultraviolet radiations
The glowing coals of a fireplace transmit heat energy by infrared radiation
A microwave emits microwave radiations.

Example:
2
Calculate the frequency of a green light of wavelength 5.10 x 10 nm.

Solution:
1) Convert wavelength to frequency:
λν = c therefore ν = c/λ
8 14 14
2) ν = 2.9979 x 10 m⋅s−1 = 5.88 x 10 s−1 = 5.88 x 10 Hz
5.10 x 10−7 m

7.2 The Nature of Matter


th
By the mid 19 century, the idea prevailed that matter and energy were distinct. Matter was
thought to consist of particles, whereas energy in the form of light (electromagnetic radiation) was
described as a wave.

Matter Electromagnetic Radiation


• objects which have both mass and volume • have no mass
• have a definite position in space and time • are delocalized, propagated as waves
• governed by Newton’s Laws of motion • governed by Maxwell’s Equations

General Chemistry 202-NYA-05: Chapter 7 105


Max Planck (1858-1947) postulated that energy can be gained or lost only in whole-number
multiples of the quantity hν, where h is a constant called Planck’s constant, determined by
experiment to have the value 6.626 x 10−34 J⋅s.

That is, the change in energy for a system ∆E can be represented by the equation

∆E = nhν

Where n is an integer (1,2,3,…), h is Planck’s constant, and ν is the frequency of the


electromagnetic radiation absorbed or emitted.

It is noteworthy to say that Planck’s result was a real surprise. It had always been assumed that
the energy of matter was continuous, which meant that the transfer of any quantity of energy was
possible. Now it seemed clear that energy is in fact quantized and can occur only in discrete units
of size hν. Each of these small “packets” of energy is called a quantum. A system can transfer
energy only in whole quanta. Thus energy seems to have particulate properties.

Example:
The yellow color in fireworks is often achieved by heating sodium (Na) to about 1300°C. Then the
compound emits yellow light having a wavelength of 589 nm. What is the increment of energy
2
(the quantum) that is emitted at 5.89 x 10 nm by Na?

Solution:
The quantum of energy can be calculated from the equation
∆E = hν

The frequency ν for this case can be calculated as follows:


8 14
ν = c = 2.9979 x 10 m s−1 = 5.09 x 10 s−1
λ 5.89 x 10−7 m

So
14
∆E = hν = (6.626 x 10−34 J⋅s)(5.09 x 10 s−1) = 3.37 x 10−19 J

A sample of Na emitting light at 589 nm can lose energy only in increments of 3.37 x 10−19 J, the
size of the quantum in this case.

The Photoelectric Effect


Albert Einstein (1879-1955) suggested that electromagnetic radiation can be viewed as a
stream of “particles” called photons. The energy of each photon is given by the expression

Ephoton = hν = hc / λ

Where h is Planck’s constant, ν is the frequency of the radiation, and λ is the wavelength of the
radiation.

The photoelectric effect (for which Einsten was later awarded the 1921 Nobel Prize) refers to the
phenomenon in which electrons are emitted from the surface of a metal when light strikes it.

Electromagnetic radiation is quantized (consists of photons), and the threshold frequency


represents the minimum energy required to remove the electron from the metal’s surface (E0 =
hν0).

General Chemistry 202-NYA-05: Chapter 7 106


Because a photon with energy less than E0 (ν < ν0) cannot remove an electron, light with a
frequency less than the threshold frequency produces no electrons. On the other hand, for light
where ν > ν0, the energy in excess of that required to remove the electron is given to the electron
as kinetic energy.

In a related development, Einstein derived the famous equation


2
E = mc

So, a photon as a mass only in a relativistic sense - it has no rest mass (it only has a mass when
it is moving). Electromagnetic radiation, which was previously thought to exhibit only wave
properties, seems to show certain characteristics of particulate matter as well. This phenomenon
is sometimes referred to as the dual nature of light. That is, all matter exhibits both particulate and
wave properties.

Large objects: predominantly particulate


Small objects (for example photons): predominantly wave properties
Intermediate objects (for example electrons): show both wave and particulate

The de Broglie's Wavelength


French physicist Louis de Broglie (1892-1987) showed that matter also exhibits wave properties.
Rearranging the previous equations describing energy to isolate the wavelength and the mass,
we obtain the de Broglie's equation:

Since
2
E = hc / λ and E = mc

Therefore:
2
hc / λ = mc

Rearranging the last equation to isolate λ:


2
λ = hc / mc = h /mc

de Broglie wavelength:
λ = h /mc

General Chemistry 202-NYA-05: Chapter 7 107


7.3 The atomic Spectrum of Hydrogen
When a sample of hydrogen gas receives a high-energy spark, the H2 molecules absorb energy,
and some of the H−H bonds are broken. The resulting hydrogen atoms are excited; that is, they
contain excess energy, which they release by emitting light of various wavelengths to produce
what is called the emission spectrum of the hydrogen atom.

A continuous spectrum contains all wavelength of visible light (like the rainbow produced when
sunlight is dispersed by raindrops).

In contrast, a hydrogen line spectrum contains only a few discrete wavelengths (410, 434, 486
and 656 nm).

It indicates that only certain energies are allowed for the electron in the hydrogen atom. (In other
words, the energy of the electron in the hydrogen atom is quantized). Changes of energy
between discrete energy levels in hydrogen will produce only certain wavelengths of emitted light.

General Chemistry 202-NYA-05: Chapter 7 108


The discrete line spectrum of hydrogen shows that only certain energies are possible; that is, the
electron energy levels are quantized. In contrast, if any energy level were allowed, the emission
spectrum would be continuous.

Extra Problems:

1. When ignited, a uranium compound burns with a green flame. The wavelength of the light given off
by this flame is greater than that of
a) red light
b) infrared light
c) radio waves
d) ultraviolet light
e) none of these
2. Which form of electromagnetic radiation has the longest wavelengths?
a) gamma rays
b) microwaves
c) radio waves
d) infrared radiation
e) x-rays
3. Which of the following frequencies corresponds to light with the longest wavelength?
a) 3.00 × 1013 s–1
b) 4.12 × 105 s–1
c) 8.50 × 1020 s–1
d) 9.12 × 1012 s–1
e) 3.20 × 109 s–1

General Chemistry 202-NYA-05: Chapter 7 109


4. A line in the spectrum of atomic mercury has a wavelength of 258 nm. When mercury emits a photon
of light at this wavelength, the frequency of this light is
a) 8.61 × 10–16 s–1
b) 7.70 × 10–19 s–1
c) 1.16 × 1015 s–1
d) 77.3 s–1
e) none of these
5. What is the wavelength of a photon of red light (in nm) whose frequency is 4.64 × 1014 Hz?
a) 646 nm
b) 1.55 × 106 nm
c) 155 nm
d) 464 nm
e) none of these

6. Green light can have a wavelength of 512 nm. The energy of a photon of this light is
a) 1.02 × 10–31 J
b) 5.12 × 10–7 J
c) 3.88 × 10–19 J
d) 5.86 × 1014 J
e) 2.58 × 1018 J
7. Which one of the following types of radiation has the shortest wavelength, the greatest energy, and
the highest frequency?
a) Ultraviolet radiation.
b) Infrared radiation.
c) Visible red light.
d) Visible blue light.
e) None, because short wavelength is associated with low energy and low frequency, not high
energy and high frequency.

8. What is the energy of a photon of violet light that has a wavelength of 425 nm?
a) 4.25 × 10–7 J
b) 4.67 × 10–19 J
c) 7.05 × 1014 J
d) 8.44 × 10–32 J
e) 2.14 × 1018 J

9. The four lines observed in the visible emission spectrum of hydrogen tell us that:
a) The hydrogen molecules they came from have the formula H4.
b) We could observe more lines if we had a stronger prism.
c) There are four electrons in an excited hydrogen atom.
d) Only certain energies are allowed for the electron in a hydrogen atom.
e) The spectrum is continuous.

Ans: 1)d 2)c 3)b 4)c 5)a 6)c 7)a 8)b 9)d

General Chemistry 202-NYA-05: Chapter 7 110


7.4 The Bohr Model
Niels Bohr (1885-1962) developed a quantum model for the hydrogen atom. He proposed that the
electron in a hydrogen atom moves around the nucleus only in certain allowed circular orbits.

Even though his model later proved to be incorrect, Bohr remained a central figure in the drive to
understand the atom. He was awarded the Nobel Prize in physics in 1922.

1. The model correctly fits the quantized energy levels of the hydrogen atom and postulates
only certain allowed circular orbits for the electron.

2. As the electron becomes more tightly bound, its energy becomes more negative relative to
the zero-energy reference state (corresponding to the electron being at infinite distance from
the nucleus). As the electron is brought closer to the nucleus, energy is released from the
system.

The most important equation to come from Bohr’s model is the expression for the energy levels
available to the electron in the hydrogen atom:
2 2
E = − 2.178 x 10−18 J ( Z / n )

In which n is an integer (the larger value of n, the larger is the orbit radius) and Z is the nuclear
charge.

General Chemistry 202-NYA-05: Chapter 7 111


This equation can be used to calculate the change in energy of an electron when the electron
changes orbits. The ground state energy is the lowest possible energy state (n = 1).
For a single electron transition from one energy level to another (to calculate the energy change
between any two energy levels in a hydrogen atom):

 1 1 
∆E = − 2.178 × 10−18 J  2 − 2 
 nfinal ninitial 

∆E = change in energy of the atom (energy of the emitted photon)


nfinal = integer; final distance from the nucleus
ninitial = integer; initial distance from the nucleus

Example:
What color of light is emitted when an excited electron in the hydrogen atom falls from:
a) n = 5 to n = 2
b) n = 4 to n = 2
c) n = 3 to n = 2

Solution:
2 2
Since E = − 2.178 x 10−18 J ( Z / n ), so
2 2
E2 = − 2.178 x 10−18 J ( 1 /2 ) = − 5.445 x 10−19J
2 2
E3 = − 2.178 x 10−18 J ( 1 /3 ) = − 2.420 x 10−19J
−18 2 2
E4 = − 2.178 x 10 J ( 1 /4 ) = − 1.361 x 10−19J
2 2
E5 = − 2.178 x 10−18 J ( 1 /5 ) = − 0.871 x 10−19J
8
And that λ = hc / ∆E = (6.626 x 10−34 J⋅s)(2.9979 x 10 m⋅s−1) / ∆E

a) ∆E = E5 – E2 = 4.574 x 10−19J so λ = 4.34 x 10−7 m = 434 nm so blue light


b) ∆E = E4 – E2 = 4.084 x 10−19J so λ = 4.86 x 10−7 m = 486 nm so green light
c) ∆E = E3 – E2 = 3.025 x 10−19J so λ = 6.57 x 10−7 m = 657 nm so orange/red light

However, it is important to understand that the Bohr’s model is incorrect. This model only works
for hydrogen. Electrons do not move around the nucleus in circular orbits. We will discuss this in
the next session.

General Chemistry 202-NYA-05: Chapter 7 112


The Balmer-Rydberg Equation:

where λ is the wavelength of the absorbed/emitted light and RH is the Rydberg constant for
hydrogen (and hydrogen only!). n is the integral constant for the transition of hydrogen. The
7 −1
Rydberg constant value for an infinitely heavy nucleus = 1.0974 x 10 meter .

n=4

n=3

n=2

n=1
e
Z

∆E =

1. Electrons can only be in one of the orbits. They cannot occupy the space between orbits.
2. Each orbit has an energy associated with it (that is, an electron in a particular orbit has
that energy). For an orbit designated by the label n, the corresponding energy of the
electron in this orbit is En:
Z2
En = − B
n2
Bohr Orbit Constant B = 2.178 × 10−18 J
Here, Z is the number of protons in the nucleus (i.e., it is the atomic number of the atom).
3. An electron can gain or lose energy by jumping from one orbit to another. In doing so it
must change its energy by a fixed amount equal to the difference in energy between what
it had in its initial orbit and what it has in its final orbit. Thus the energy gained or lost as it
makes the jump comes in packets (i.e., it is quantized).
If the electron jumps from an outer orbit to an inner orbit it releases a photon to carry
away the excess energy. This quantum of energy, given by Planck’s formula (E = hv
=hc/λ), is observed as a line in the spectrum of the atom.

General Chemistry 202-NYA-05: Chapter 7 113


Energy-Level Diagram for the Hydrogen Atom
2
Z2 (1) −2.178 × 10−18 J
En = − B 2
for the Hydrogen atom, Z=1, En = − B 2 =
n n n2

Energy (J)

n = ∞, E∞ = 0 J

n = 6, E6 = –0.06050×10–18 J

n = 5, E5 = –0.08712×10–18 J

n = 4, E4 = –0.1361×10–18 J

n = 3, E3 = –0.2420×10–18 J

656.3 nm

486.1 nm

434.1 nm

410.2 nm
n = 2, E2 = –0.5445×10–18 J
Balmer Series
121.6 nm

102.6 nm

97.25 nm

94.98 nm

93.78 nm

91.21 nm

n = 1, E1 = –2.178×10–18 J
Lyman Series

Example: Suppose the electron in the hydrogen atom makes a transition from the n = 6 orbit to
the n = 3 orbit. Calculate the wavelength of the emitted light as a result of this
transition.
Method 1
Calculate the energy the electron in the n = 6 and in the n = 3 states. The formula En = -B(1)2/n2
can be used and setting the values of n to 6 and to 3. (this is what was done in preparing the
energy level diagram above).
E6 = –0.06050×10–18 J E3 = –0.2420×10–18 J
Thus the electron must change its energy from (–0.06050×10–18 J) to (–0.2420×10–18 J)
∆E = (–0.2420×10–18 J) – (–0.06050×10–18 J) =–0.1815×10–18 J
The energy of the emitted photon is of the same magnitude but opposite in sign to the energy lost
by the electron.
hc
E photon = + 0.1815 ×10−18 J =
λ

λ=
hc
=
( 6.626 ×10−34 J ⋅ s )( 2.998 ×108 m ⋅ s −1 ) = 1.094 × 10−6 m = 1094 nm
E photon 0.1815 ×10−18 J

General Chemistry 202-NYA-05: Chapter 7 114


Method 2: This is in fact identical to method one. The (Bohr) Rydberg formula used is simply
obtained by using variables instead of numbers as in Method 1.

1  1 1  1 1 
= RH  2 − 2  = (1.097 × 107 m −1 )  2 − 2 −5
 = 9.141× 10 m
−1

λ   3 6 
 n f ni 
−6
λ = 1.094 × 10 m = 1094 nm

Example: Calculate the wavelength of the photon emitted by an electron as it makes a 3→2
transition in:
a) the hydrogen atom
b) the Na10+ ion
a)

2
1 BZ 2  1 1  B (1)  1 1
2
 ( 2.178 ×10−18 J ) (1) 1 1 
 2 − 2  =
6 −1
=  −  = −1  2
− 2  = 1.523 × 10 m
hc  22 32  ( 6.626 ×10 J ⋅ s )( 2.998 × 10 m ⋅ s )  2 3 
−34
λ hc  n f ni 
8

λ = 6.563 ×10−7 m = 656.3 nm

b)

2
1 BZ 2  1 1  B (11)  1 1 
2
( 2.178 ×10−18 J ) (11) 1 1
=  2 − 2  =  2 − 2 = −1  2
− 2  = 1.843 × 109 m −1
λ hc  n f
ni  hc  2 3  ( 6.626 × 10 −34
J ⋅ s )( 2.998 × 10 8
m ⋅ s )  2 3 
−9
λ = 5.422 × 10 m = 5.422 nm

Example: The 4Be3+ ion has a ground state energy of 3.486×10–17 J. Calculate the energy and
wavelength of the radiation needed to raise the electron from the ground state to the
n = 5 state.
Ground State – the state with the lowest energy
In the case of the 4Be3+ ion, since there is only one electron, the ground state will correspond to
the electron occupying the n = 1 orbit. Thus, this question corresponds to asking what the energy
change for the 1→5 transition for the 4Be3+ ion.
The ground state energy for the 4Be3+ ion that is given (3.486×10–17 J) corresponds to the Energy of
st
the 1 Bohr orbit
22
Z2 ((4)
11)
E1 = B 2
= −B 2
= −3.486 × 10−17 J
n ()
1
1 1
∆E = −3.486 × 10−17 J  2 − 2 
5 1 
24
∆E =
25
( 3.486 ×10−17 J ) = 3.347 ×10−17 J
hc ( 6.626 ×10 J ⋅ s )( 2.998 × 10 m ⋅ s )
−34 8 −1

λ= = = 5.935 ×10−9 m = 5.935 nm


∆E 3.347 × 10−17 J

General Chemistry 202-NYA-05: Chapter 7 115


Example: One of the lines in the spectrum of the hydrogen atom occurs at 410.2 nm. To what
electronic transition does this wavelength correspond?
The 410.2 nm line falls in the visible portion of the spectrum. For hydrogen, the visible lines
(Balmer Series) occurs with a terminal state of nf = 2. The problem now reduces to solving for ni .
λ=410.2 nm = 410.2×10– 9 m = 4.102×10– 7 m

B (1)  1 1  B  1 1 
2
1
=  − =  − 
λ hc  n 2f ni2  hc  22 n 2 
hc  1 1 
= − 
Bλ  2 2 n 2 
1 1 hc
2
= 2−
n 2 Bλ
1 1
n= = = 6.01
1 hc 1 ( 6.626 ×10 J ⋅ s )( 2.998 × 10 m ⋅ s )
−34 8 −1
− −
2 2 Bλ 22 ( 2.178 ×10−18 J )( 4.102 ×10−7 m )
The electron has made a 6→2 transition.

Example. The electron in a one electron system is initially in the n=4 state.
a) How many different lines will appear in the line spectrum as the electron moves
from the n=4 state to the ground state?
b) Which of these lines is (are) most intense.

n=4
The lines are:
n=3 4→1 1 time
4→2 1 time
n=2 2→1 2 times
4→3 2 times
3→2 1 time
3→1 1 time
n=1

There are 6 lines. The lines corresponding to the 2→1 transition and the 4→3 transition are two-
times more intense than the other lines.

General Chemistry 202-NYA-05: Chapter 7 116


The Bohr Model: Simplified in other words

nucleus orbitals

...

8
Energy: -2.178 -0.545 -0.242 -0.1361 0

The zero (0) Energy is fixed at an infinite distance (∞) from the nucleus.

The electron naturally wants to get closer to the nucleus.

When the electron gets closer, its energy decreases (or increases with a negative sign).

The closer the electron gets to the nucleus, more energy is released in its surrounding.

That energy is released in the form of a photon.

A photon has a positive sign, with the same magnitude.

A wavelength can never have a negative sign.

General Chemistry 202-NYA-05: Chapter 7 117


Extra Easy Problems:

1. The wavelength of light associated with the n = 2 to n = 1 electron transition in the hydrogen
spectrum is 1.216 × 10–7 m. By what coefficient should this wavelength be multiplied to obtain the
wavelength associated with the same electron transition in the Li2+ ion?
a) 1/9
b) 1/7
c) 1/4
d) 1/3
e) 1

2. When a hydrogen electron makes a transition from n = 3 to n = 1, which of the statements is true?
I. Energy is emitted.
II. Energy is absorbed.
III. The electron loses energy.
IV. The electron gains energy.
V. The electron cannot make this transition.

a) I, IV
b) I, III
c) II, III
d) II, IV
e) V

3. What is the wavelength of light that is emitted when an excited electron in the hydrogen atom falls
from n = 5 to n = 1?
a) 1.05 × 107 m
b) 9.50 × 10–8 m
c) 2.09 × 10–18 m
d) 9.12 × 10–8 m
e) none of these

4. When an electron in a 2p orbital of a particular atom makes a transition to the 2s orbital, a photon of
approximate wavelength 669.5 nm is emitted. The energy difference between these 2p and 2s orbitals
is
a) 2.97 × 10–28 J
b) 2.97 × 10–19 J
c) 2.97 × 10–17 J
d) 1.33 × 10–31 J
e) none of these

Ans: 1)a 2)b 3)b 4)b

General Chemistry 202-NYA-05: Chapter 7 118


7.5 The quantum Mechanical Model of the Atom
Werner Heisenberg (1901-1976), Louis de Broglie (1892-1987) and Erwin Schrödinger (1887-
1961) developed what became known as the wave mechanics or, more commonly, quantum
mechanics.

De Broglie originated the idea that the electron, previously considered to be a particle, also shows
wave properties. (The waves are described as “standing” because they are stationary; the waves
do not travel along the length of a string).

A specific wave function is often called an orbital.

An orbital is not a Bohr orbit. The electron in the hydrogen 1s orbital is not moving around the
nucleus in a circular orbit. How is the electron moving? We don’t know.

There is a fundamental limitation to just how precisely we can know both the position and
momentum of a particle at a given time. This is a statement of the Heisenberg uncertainty
principle.
The more accurately we know a particle’s position, the less accurately we can know its
momentum, and vice versa.

The wave function is most conveniently represented as a probability distribution.

General Chemistry 202-NYA-05: Chapter 7 119


Illustration of integer number of wavelengths:

General Chemistry 202-NYA-05: Chapter 7 120


7.6 Quantum numbers
Each orbital (wave function) is characterized by a series of number called quantum numbers,
which describe various properties of the orbital.

• Principal quantum number (n) is related to the size and energy of the orbital. As n
increases, the orbital becomes larger and the electron spends more time farther from the
nucleus. An increase in n also means higher energy, because the electron is less tightly
bound to the nucleus. n as integral values (1, 2, 3, 4, 5, 6, 7).

• Angular momentum quantum number (l) is related to the shape of atomic orbitals
(sometimes called a subshell). It has integral values from 0 to n−1 for each value of n.
The value of l for a particular orbital is commonly assigned a letter:
l = 0 is called s
l = 1 is called p
l = 2 is called d
l = 3 is called f
l = 4 is called g

• Magnetic quantum number (ml) is related to the orientation of the orbital in space
relative to the other orbitals in the atom. It has integral values between l and −l,
including the zero.

Example
For principal quantum level n = 3, determine the number of allowed subshells (different
values of l), and give the designation of each.

Solution:
Number of allowed subshells = 3
l = 0, 3s
l = 1, 3p
l = 2, 3d

General Chemistry 202-NYA-05: Chapter 7 121


Example:
For l = 2, determine the magnetic quantum numbers (ml) and the number of orbitals.

Solution:
magnetic quantum numbers = –2, – 1, 0, 1, 2
number of orbitals = 5

7.7 Orbitals Shapes and Energies


The meaning of an orbital is represented most clearly by a probability distribution.

s Orbitals:
Note that the 2s and 3s orbitals contain areas of high probability separated by areas of zero
probability. These latter areas are called nodal surfaces, or simply nodes. The number of nodes
increases as n increases. For s orbitals, the number of nodes is given by n – 1.

General Chemistry 202-NYA-05: Chapter 7 122


p Orbitals:
There are two types of representations for the 2p orbitals (there are no 1p orbitals).

The p orbitals are not spherical like s orbitals but have two lobes separated by a node at the
nucleus. The p orbitals are labeled according to the axis of the xyz coordinate system along
which the lobes lie. For example, the 2p orbital with lobes centered along the x axis is called the
2px orbital. Note that the signs inside the surface indicate the phases (signs) of the orbital in that
region of space (not the charge).

Here is a view of a cross section of the electron probability distribution for a 3p orbital.

Hydrogen’s single electron can occupy any of its atomic orbitals. However, in the lowest energy
state, the ground state, the electron resides in the 1s orbital. If energy is put into the atom, the
electron can be transferred to a higher-energy orbital, producing an excited state.

For the hydrogen atom, the energy of a particular


orbital is determined by its value of n. Thus all
orbitals with the same value of n have the same
energy – they are said to be degenerate.

General Chemistry 202-NYA-05: Chapter 7 123


d Orbitals:
There are five shapes of d orbitals. There are no d orbitals that correspond to principal quantum
level n = 1 and n = 2. The d orbitals (l = 2) first occur in level n = 3, (3d).

f Orbitals:
There are seven shapes of f orbitals. There are no f orbitals that correspond to principal quantum
level n = 1, 2, 3. The f orbitals (l = 3) first occur in level n = 4, (4f). These orbitals are not involved
in the bonding in any of the compounds we will consider in this class. Their shapes and labels are
simply included for completeness.

General Chemistry 202-NYA-05: Chapter 7 124


7.8 Electron Spin and the Pauli Principle
The electron spin quantum number (ms), can have only one of two values, +½ and −½. We can
interpret this to mean that the electron can spin in one of two opposite directions, although other
interpretations also have been suggested.

The Pauli exclusion principle: In a given atom, no two electrons can have the same set of four
quantum numbers (n, l, ml and ms).

7.9 Polyelectronic Atoms


Polyelectronic Atoms are atoms with more than one electron.

More commonly, the approximation used is to treat each electron as if it were moving in a field of
charge that is the net result of the nuclear attraction and the average repulsions of all the other
electrons.

We will consider that polyelectronic atoms have hydrogenlike orbitals. They have the same
general shapes as the orbitals for hydrogen, but their sizes and energies are different. (The
differences occur because of the interplay between nuclear attraction and the electron
repulsions).

For polyelectronic atoms, we find that for a given principal quantum level the orbitals vary in
energy as follows:
Ens < Enp < End < Enf

When electrons are placed in a particular quantum level, they “prefer” the orbitals in the order s,
p, d and then f.

For example:
E4s < E4p < E4d < E4f

General Chemistry 202-NYA-05: Chapter 7 125


Extra Problems:
1. How many f orbitals have the value n = 2?
a) 0
b) 3
c) 5
d) 7
e) 1
2. How many d orbitals have n = 4?
a) 2
b) 5
c) 10
d) 7
e) 18
3. If n = 2, how many orbitals are possible?
a) 3
b) 4
c) 2
d) 8
e) 6
4. A given set of p orbitals consists of ______ orbitals.
a) 1
b) 2
c) 3
d) 4
e) 5
5. Which of the following is an incorrect designation for an atomic orbital?
a) 1s
b) 3d
c) 1p
d) 4f
e) 6s
6. The number of orbitals having a given value of l is equal to
a) 2l + 1
b) 2n + 2
c) 3l
d) l + ml
e) the number of lobes in each orbital
7. How many electrons in an atom can have the quantum numbers n = 3, l = 2?
a) 2
b) 5
c) 10
d) 18
e) 6
8. How many electrons can be described by the quantum numbers n = 2, l = 2, ml = 1?
a) 0
b) 2
c) 6
d) 10
e) 14

General Chemistry 202-NYA-05: Chapter 7 126


9. How many electrons can be contained in all of the orbitals with n = 4?
a) 2
b) 8
c) 10
d) 18
e) 32
10. What is the l quantum number for a 4p orbital?
a) 2
b) 1
c) 0
d) 3
e) more than one of the above
11. Which of the following could not be a valid ml quantum number for a 4f orbital?
a) 3
b) 1
c) –1
d) 2
e) 4
12. How many electrons in an atom can have the quantum numbers n = 4, l = 2?
a) 14
b) 12
c) 5
d) 10
e) 6
13. Which of the following combinations of quantum numbers (n, l, ml, ms) do not represent permissible
solutions of the Schrödinger equation for the electron in the hydrogen atom (i.e., which combination
of quantum numbers is not allowed)?
a) 9, 8, -4, 1/2
b) 8, 2, 2, 1/2
c) 6, -5, -1, 1/2
d) 6, 5, -5, 1/2
e) All are allowed.
14. If l = 3, how many electrons can be contained in all the possible orbitals?
a) 7
b) 6
c) 14
d) 10
e) 5
15. Which of the following combinations of quantum numbers is not allowed?
a) n = 1, l = 1, ml = 0, ms = 1 2
b) n = 3, l = 0, ml = 0, ms = - 1 2
c) n = 2, l = 1, ml = -1, ms = 1 2
d) n = 4, l = 3, ml = -2, ms = - 1 2
e) n = 4, l = 2, ml = 0, ms = 1 2

Ans: 1)a 2)b 3)b 4)c 5)c 6)a 7)c 8)a 9)e 10)b 11)e 12)d 13)c 14)c 15)a

General Chemistry 202-NYA-05: Chapter 7 127


7.11 The Aufbau Principle and the Periodic Table
The class notes regarding this session is exactly what you will find in your textbook. To make
sure you have assimilated the theory, please do all the suggested problems in your textbook.

We can use the quantum mechanical model of the atom to show how the electron arrangements
in the hydrogenlike atomic orbitals of the various atoms account for the organization of the
periodic table. Our main assumption here is that all atoms have the same type of orbitals as have
been described for the hydrogen atom.

The Aufbau Principle: As protons are added one by one to the nucleus to build up the elements,
electrons are similarly added to these hydrogenlike orbitals. (Aufbau is German for “building up”).

Hydrogen has one electron, which occupies the 1s orbital in its ground state. The electronic
1
configuration for hydrogen is written as 1s , which can be represented by the following orbital
diagram, where the arrow represents an electron spinning in a particular direction:
1s 2s 2p
1
H: 1s

The next element, helium, has two electrons. Since two electrons with opposite spins can occupy
an orbital, according to the Pauli exclusion principle, the electrons for helium are in the 1s orbital
2
with opposite spins, producing a 1s configuration:
1s 2s 2p
2
He: 1s

Lithium has three electrons, two of which can go into the 1s orbital before the orbital is filled.
Since the 1s orbital is the only orbital for n = 1, the third electron will occupy the lowest-energy
2 1
orbital with n = 2, or the 2s orbital, giving a 1s 2s configuration:
1s 2s 2p
2 1
Li: 1s 2s

The next element, beryllium, has four electrons, which occupy the 1s and 2s orbitals:
1s 2s 2p
2 2
Be: 1s 2s

Boron has five electrons, four of which occupy the 1s and 2s orbitals. The fifth electron goes into
the second type of orbital with n = 2, the 2p orbitals:
1s 2s 2p
2 2 1
B: 1s 2s 2p

Since all the 2p orbitals have the same energy (are degenerate), it does not matter which 2p
orbital the electron occupies.

Carbon is the next element and has six electrons. Two electrons occupy the 1s orbital, two
occupy the 2s orbital, and two occupy 2p orbitals. Since there are three 2p orbitals with the same
energy, the mutually repulsive electrons will occupy separate 2p orbitals. This behavior is
summarized by Hund’s rule:

Hund’s rule: The lowest energy configuration for an atom is the one having the maximum
number of unpaired electrons allowed by the Pauli principle in a particular set of degenerate
orbitals. (By convention, the unpaired electrons are represented as having parallel spins, with
spin “up”).

General Chemistry 202-NYA-05: Chapter 7 128


2 2 2
The configuration of carbon is given as 1s 2s 2p , and it is understood that the electrons are in
different 2p orbitals. Note that the unpaired electrons in the 2p orbitals are shown with parallel
spins. The orbital diagram for carbon is:
1s 2s 2p
2 2 2
C: 1s 2s 2p
2 2 3
The configuration for nitrogen, which has seven electrons, is 1s 2s 2p . The three electrons in the
2p orbitals occupy separate orbitals with parallel spins:
1s 2s 2p
2 2 3
N: 1s 2s 2p
2 2 4
The electronic configuration for oxygen, which has eight electrons, is 1s 2s 2p . One of the 2p
orbitals is now occupied by a pair of electrons with opposite spins, as required by the Pauli
exclusion principle:
1s 2s 2p
2 2 4
O: 1s 2s 2p
1s 2s 2p
2 2 5
F: 1s 2s 2p

1s 2s 2p
2 2 6
Ne: 1s 2s 2p

For sodium, the first ten electrons occupy the 1s, 2s, and 2p orbitals, and the eleventh electron
must occupy the first orbital with n = 3, the 3s orbital. The electron configuration for sodium is
2 2 6 1
1s 2s 2p 3s .
1
To avoid writing the inner-level electrons, this configuration is often abbreviated as [Ne]3s , where
2 2 6
[Ne] represents the electron configuration of neon, 1s 2s 2p . The next element follow the same
rules.

Valence electrons are the electrons in the outermost principal quantum level of an atom.
Valence electrons are the most important electrons because they are the ones involved in
bonding. (The inner electrons are known as core electrons).

The elements in the same group (vertical column of the periodic table) have the same valence
electron configuration.

Mendeleev originally placed the elements in groups based on similarities in chemical properties.
Now we understand the reason behind these groupings. Elements with the same valence
electron configuration show similar chemical behavior.

The element after argon is potassium. Since the 3p orbitals are fully occupied in argon, we might
expect the next electron to go into a 3d orbital (recall that for n = 3 the orbitals are 3s, 3p, and
3d). However, the chemistry of potassium is clearly very similar to that of lithium and sodium,
indicating that the last electron in potassium occupies the 4s orbital instead of one of the 3d
orbitals, a conclusion confirmed by many types of experiments. The electronic configurations of
potassium and calcium are:
2 2 6 2 6 1 1
K: 1s 2s 2p 3s 3p 4s or [Ar]4s
2 2 6 2 6 2 2
Ca: 1s 2s 2p 3s 3p 4s or [Ar]4s

General Chemistry 202-NYA-05: Chapter 7 129


The next element, scandium, begins a series of 10 elements (scandium through zinc) called the
transition metals, whose configurations are obtained by adding electrons to the five 3d orbitals.
The configuration of scandium, titanium and vanadium are:
2 1 2 2 2 3
Sc: [Ar]4s 3d Ti: [Ar]4s 3d V: [Ar]4s 3d
2 4
Chromium is the next element. The expected configuration is Cr: [Ar]4s 3d . However, the
observed configuration is :
1 5
Cr: [Ar]4s 3d
1s 2s 2p 3s 3p 4s 3d

The explanation for this configuration of chromium is beyond the scope of this class. In fact,
chemists are still disagreeing over the exact cause of this anomaly. The observed configuration
has both the 4s and 3d orbitals half-filled.

The next four elements have the expected configurations:


2 5 2 6 2 7 2 8
Mn: [Ar]4s 3d Fe: [Ar]4s 3d Co: [Ar]4s 3d Ni: [Ar]4s 3d
2 9
The configuration for copper is expected to be Cu: [Ar]4s 3d . However, the observed
configuration is:
1 10
Cu: [Ar]4s 3d
1s 2s 2p 3s 3p 4s 3d

In this case, a half-filled 4s orbital and a filled set of 3d orbitals characterize the actual
configuration.

Additional Important Points:

1. The (n + 1)s orbitals always fill before the nd orbitals. For example, the 5s orbitlas fill in
rubidium and strontium before the 4d orbitals fill in the second row of transition metals
(yttrium through cadmium).

2. After lanthanum a group of 14 elements called the lanthanide series (or the lanthanides)
occurs. This series of elements corresponds to the filling of the seven 4f orbitals. (Note that
sometimes an electron occupies a 5d orbital instead of a 4f orbital; Ce, Gd, Lu. This occurs
because the energies of the 4f and 5d orbitals are very similar).

3. After actinium a group of 14 elements called the actinide series (or the actinides) occurs. This
series of elements corresponds to the filling of the seven 5f orbitals. (Note that sometimes an
electron or two occupy a 6d orbital instead of a 5f orbital; this occurs because the energies
are very similar).

4. The group labels for Groups 1A, 2A, 3A, 4A, 5A, 6A, 7A, and 8A indicate the total number of
valence electrons for the atoms in these groups.

5. When a metal gets ionised (loses electrons, gets a positive charge) it is the "s" electrons that
2+ 10
are lost first. For example, Cd has the electronic configuration [Kr]4d

General Chemistry 202-NYA-05: Chapter 7 130


A typical exercice would ask you to give the electronic configuration of an atom, using a periodic
table (on which the electronic configuration is obviously not written). The electronic configuration
of Ytterbium?
6s 4f
14 2
Yb : [Xe]4f 6s and [Xe]

General Chemistry 202-NYA-05: Chapter 7 131


Extra Problems
1. Which of the following atoms or ions has three unpaired electrons?
a) N
b) O
c) Al
d) S2–
e) Ti2+

2. The electron configuration for the barium atom is:


a) 1s22s22p63s23p64s23d10
b) [Xe]6s2
c) 1s22s22p63s23p64s1
d) 1s22s22p63s23p64s2
e) none of these
3. The electron configuration for the carbon atom is:
a) 1s22s22p2
b) [He]2s4
c) [Ne]2s22p2
d) 1s22p4
e) none of these
4. The complete electron configuration of antimony is
a) 1s22s22p63s23p64s23d104p65s24d105d105p3
b) 1s22s22p63s23p64s23d104d104p3
c) 1s22s22p63s23p64s24p65s24d105d105p3
d) 1s22s22p63s23p64s23d104p65s24d105p3
e) none of these
5. Which of the following statements about quantum theory is incorrect?
a) The energy and position of an electron cannot be determined simultaneously.
b) Lower energy orbitals are filled with electrons before higher energy orbitals.
c) When filling orbitals of equal energy, two electrons will occupy the same orbital before filling
a new orbital.
d) No two electrons can have the same four quantum numbers.
e) All of these are correct.
6. Which of the following statements is true?
a) The exact location of an electron can be determined if we know its energy.
b) An electron in a 2s orbital can have the same n, l, and ml quantum numbers as an electron in a
3s orbital.
c) Ni has two unpaired electrons in its 3d orbitals.
d) In the buildup of atoms, electrons occupy the 4f orbitals before the 6s orbitals.
e) Only three quantum numbers are needed to uniquely describe an electron.
7. An element has the electron configuration [Kr] 5s24d105p2. The element is a(n)
a) nonmetal
b) transition element
c) metal
d) lanthanide
e) actinide
8. Element E has the electron configuration [Kr] 5s24d105p2. Formula for the fluoride of E is most likely
a) EF14
b) EF4
c) EF
d) EF6
e) EF8

General Chemistry 202-NYA-05: Chapter 7 132


9. An element with the electron configuration [Xe] 6s24f145d7 would belong to which class on the
periodic table?
a) transition elements
b) alkaline earth elements
c) halogens
d) rare earth elements
e) none of the above
10. All alkali metals have the following number of valence electrons:
a) 2
b) 3
c) 5
d) 1
e) none of these
11. Ti has __________ in its d orbitals.
a) one electron
b) two electrons
c) three electrons
d) four electrons
e) none of these
12. Germanium has __________ in its 4p orbitals.
a) one electron
b) two electrons
c) three electrons
d) four electrons
e) none of these
13. Fe has __________ that is (are) unpaired in its d orbitals.
a) one electron
b) two electrons
c) three electrons
d) four electrons
e) none of these

Use the following to answer questions 14-15:

Nitrogen has five valence electrons. Consider the following electron arrangements.
2s 2p
a) ↑↓ ↑ ↑ ↑

b) ↑ ↑↓ ↑ ↓

c) ↑ ↑↑ ↑ ↑

d) ↑↓ ↑ ↑

e) ↑↓ ↑↓ ↑ ↑

General Chemistry 202-NYA-05: Chapter 7 133


14. Which represents the ground state for N?
a) option a
b) option b
c) option c
d) option d
e) option e
15. Which represents the ground state for the N– ion?
a) option a
b) option b
c) option c
d) option d
e) option e
16. An atom of fluorine contains nine electrons. How many of these electrons are in s orbitals?
a) 2
b) 4
c) 6
d) 8
e) none
17. Of the following elements, which has occupied d orbitals in its ground-state neutral atoms?
a) Ba
b) Ca
c) Si
d) P
e) Cl
18. Of the following elements, which needs three electrons to complete its valence shell?
a) Ba
b) Ca
c) Si
d) P
e) Cl
19. Which of the following electron configurations is correct?
a) Ga: [Kr]4s23d104p1
b) Mo: [Kr]5s24d5
c) Ca: [Ar]4s13d10
d) Br: [Kr]4s23d104p7
e) Bi: [Xe]6s24f145d106p3
20. 1s22s22p63s23p64s23d3 is the correct electron configuration for which of the following atoms?
a) Ga
b) V
c) As
d) Nb
e) none of these

Ans: 1)a 2)b 3)a 4)d 5)c 6)c 7)c 8)b 9)a 10)d 11)b 12)b 13)d 14)a 15)e 16)b 17)a 18)d 19)e
20)b

General Chemistry 202-NYA-05: Chapter 7 134


7.12 Periodic Trends in Atomic Properties
We will use the model of polyelectronic atoms to account for the observed trends in several
important atomic properties:

• Ionization Energy
• Electron Affinity
• Atomic Size

Ionization Energy
Ionization energy is the energy required to remove an electron from a gaseous atom or ion:
+
X(g) → X (g) + e−
Where the atom or ion is assumed to be in its ground state.

To introduce some of the characteristics of ionization energy, we will take for example the case of
Aluminium. We will consider the energy required to remove several electrons in succession from
aluminium. The ionization energies are:
+
Al(g) → Al (g) + e− First ionization energy I1 = 580 kJ/mol
+ 2+
Al (g) → Al (g) + e− Second ionization energy I2 = 1815 kJ/mol
2+ 3+
Al (g) → Al (g) + e− Third ionization energy I3 = 2740 kJ/mol
3+ 4+
Al (g) → Al (g) + e− Fourth ionization energy I4 = 11 600 kJ/mol

In a stepwise ionization process, it is always the highest-energy electron (the one bound least
tightly) that is removed first. The first ionization energy is the energy required to remove the
highest-energy electron of an atom.

The first electron removed from the aluminium atom comes from the 3p orbital (Al has the
2 1
electron configuration [Ne]3s 3p ). The second electron comes from the 3s orbital (since Al+ has
2
the configuration [Ne]3s ). Note that the value of I1 is considerably smaller than the value of I2,
the second ionization energy.

The increase in positive charge binds the electrons more firmly, and the ionization energy
increases.

General Chemistry 202-NYA-05: Chapter 7 135


The largest jump in ionization energy by far occurs in going from the third ionization energy (I3) to
3+
the fourth (I4). This is so because I4 corresponds to removing a core electron (Al has the
2 2 6
configuration 1s 2s 2p ), and core electrons are bound much more tightly than valence electrons.

In general, as we go across a period from left to right, the first ionization energy increases

Thus, electrons in the same principal quantum level are generally more strongly bound as we
move to the right on the periodic table.

On the other hand, first ionization energy decreases in going down a group. The main reason is
that the electrons being removed are, on average, farther from the nucleus. As n increases, the
size of the orbital increases, and the electron is easier to remove.

Exceptions:

As always, there are some exceptions to these general rules.

Ordinarily, the first ionization energy increases as we go across a period. However, the decrease
in ionization energy in going from beryllium to boron reflects the fact that the electrons in the filled
2s orbital provide some shielding for electrons in the 2p orbital from the nuclear charge.

In going from nitrogen to oxygen (or from phosphorus to sulphur), the fourth p electron in oxygen
must be placed in an already occupied orbital. The electron-electron repulsions that result cause
this electron to be more easily removed than might be expected.

General Chemistry 202-NYA-05: Chapter 7 136


Exercise:
Consider the atoms in the following electron configurations:
2 6 2 6 1 2 6 2
[Ne]3s 3p [Ne]3s 3p 4s [Ne]3s 3p 4s
Which atom has the largest first ionization energy, and which one has the smallest second
ionization energy?
Solution:
2 6
The atom with the largest value of I1 is the one with the configuration [Ne]3s 3p (Argon),
because the 3p electrons do not shield each other very effectively, I1 will be relatively large. The
electrons in the 4s are effectively shielded by the core electrons and are farther from the nucleus
than the 3p electrons in Ar. Thus I1 for these atoms will be smaller than for Ar.
2 6 2
The atom with the smallest value of I2 is the one with the configuration [Ne]3s 3p 4s
(Ca). For Calcium, both I1 and I2 involve valence electrons. For the atom with the configuration
2 6 1
[Ne]3s 3p 4s (Potassium), the second electron lost (corresponding to I2) is a core electron (from
a 2p orbital).

Electron Affinity
Electron affinity is the energy change associated with the addition of an electron to a gaseous
atom:
X(g) + e− → X− (g)
In this course, we define electron affinity as a change in energy, which means that if the addition
of the electron is exothermic, the corresponding value for electron affinity will carry a negative
sign. (For your knowledge, electron affinity has been defined in many other textbooks as the
energy released when an electron is added to a gaseous atom.)

When we go down a group, electron affinity should become more positive (less energy released),
since the electron is added at increasing distances from the nucleus. Although this is generally
the case, the changes in electron affinity in going down most groups are relatively small, and
numerous exceptions occur.

In the case of the halogens, the energy released when an electron is added to fluorine is smaller
than expected because of the small size of the 2p orbitals. Because the electrons are very close
together, there are unusually large electron-electron repulsions.

Although electron affinities generally become more negative from left to right across a period,
there are several exceptions to this rule in each period.

General Chemistry 202-NYA-05: Chapter 7 137


Exceptions (in periods)
2 2 3 2 2 4
For example, an electron added to nitrogen (1s 2s 2p ) to form the N−(g) ion (1s 2s 2p )
would have to occupy a 2p orbital that already contains one electron. The extra repulsion
between the electrons in this doubly occupied orbital causes N−(g) to be unstable.
2 2 2
However, when an electron is added to carbon (1s 2s 2p ), no such extra repulsions
occur.

In contrast, oxygen’s greater nuclear charge compared with that of nitrogen is sufficient to
overcome the repulsion associated with putting a second electron into an already
occupied 2p orbital. However, a second electron cannot be added to an oxygen atom to
2−
form an isolated oxide ion (O ).

When we go down a group, electron affinity should become more positive (less energy released),
since the electron is added at increasing distances from the nucleus. Although this generally the
case, the changes in electron affinity in going down most groups are relatively small, and
numerous exceptions occur.

Exception (in groups)

For Fluorine (F), because the electron must be very close together in the 2p orbitals,
there are unusually large electron-electron repulsions.

General Chemistry 202-NYA-05: Chapter 7 138


Atomic Radius
The atomic radii decrease in going from left to right across a period. This decrease can be
explained in terms of the increasing effective nuclear charge (decreasing shielding) in going from
left to right. This means that the valence electrons are drawn closer to the nucleus, decreasing
the size of the atom.

Atomic radius increases down a group, because of the increases in the orbital sizes in successive
principal quantum levels.

Example:
+ + + +
Predict the trend in radius for the following ions: Li , Na , K , and Rb .

Solution:
All these ions are formed by removing one electron from an atom of a Group 1A element.
+ + + +
Li < Na < K < Rb

General Chemistry 202-NYA-05: Chapter 7 139


Extra Problems:
1. Which of the following processes represents the ionization energy of bromine?
a) → Br+(g) + e–
Br(s) 
b) → Br+(g) + e–
Br(l) 
c) → Br+(g) + e–
Br(g) 
d) → Br+(s) + e–
Br(s) 
e) → Br2+(g) + e–
Br2(g) 
2. Order the elements S, Cl, and F in terms of increasing ionization energy.
a) S, Cl, F
b) Cl, F, S
c) F, S, Cl
d) F, Cl, S
e) S, F, Cl
3. Order the elements S, Cl, and F in terms of increasing atomic radii.
a) S, Cl, F
b) Cl, F, S
c) F, S, Cl
d) F, Cl, S
e) S, F, Cl
4. Which of the following atoms would have the largest second ionization energy?
a) Mg
b) Cl
c) S
d) Ca
e) Na
5. The first ionization energy of Mg is 735 kJ/mol. The second ionization energy is
a) 735 kJ/mol
b) less than 735 kJ/mol
c) greater than 735 kJ/mol
d) More information is needed to answer this question.
e) None of these.

General Chemistry 202-NYA-05: Chapter 7 140


6. Which of the following exhibits the correct orders for both atomic radius and ionization energy,
respectively? (smallest to largest)
a) S, O, F, and F, O, S
b) F, S, O, and O, S, F
c) S, F, O, and S, F, O
d) F, O, S, and S, O, F
e) none of these
7. Choose the element with the highest ionization energy.
a) Na
b) Mg
c) Al
d) P
e) S
8. Which of the following concerning second ionization energies is true?
a) That of Al is higher than that of Mg because Mg wants to lose the second electron, so it is
easier to take the second electron away.
b) That of Al is higher than that of Mg because the electrons are taken from the same energy
level, but the Al atom has one more proton.
c) That of Al is lower than that of Mg because Mg wants to lose the second electron, thus the
energy change is greater.
d) That of Al is lower than that of Mg because the second electron taken from Al is in a p orbital,
thus it is easier to take.
e) The second ionization energies are equal for Al and Mg.

9. Consider the following orderings.


+ 2+ 3+ 4+
I. Na < Mg < Al < Si
II. Be < Mg < Ca < Sr
III. I < Br < Cl < F
IV. Al < Si < P < Cl
Which of these give(s) a correct trend in ionization energy?
a) III
b) II, IV
c) I, IV
d) I, III, IV
e) none of them
10. List the following atoms in order of increasing ionization energy: Li, Na, C, O, F.
a) Li < Na < C < O < F
b) Na < Li < C < O < F
c) F < O < C < Li < Na
d) Na < Li < F < O < C
e) Na < Li < C < F < O
11. Consider the ionization energy (IE) of the magnesium atom. Which of the following is not true?
a) The IE of Mg is lower than that of sodium.
b) The IE of Mg is lower than that of neon.
c) The IE of Mg is lower than that of beryllium.
d) The IE of Mg is higher than that of calcium.
e) The IE of Mg is lower than that of Mg+.

Ans: 1)c 2)a 3)d 4)e 5)c 6)d 7)d 8)b 9)d 10)b 11)a

General Chemistry 202-NYA-05: Chapter 7 141


General Chemistry 202-NYA-05: Chapter 7 142
Chapter 8

Bonding: General Concepts

th
Suggested problems in chapter 8 of Zumdahl 8 Edition:

25, 27, 29, 33, 35, 37, 41, 43, 45, 47, 49, 51, 79, 81, 93, 97, 109, 115, 117, 118, 119.

General Chemistry 202-NYA-05: Chapter 8 143


The world around us is composed almost entirely of compounds and mixtures of compounds.

For example, graphite is a soft, slippery material used as a lubricant in locks, and diamond is one
of the hardest materials known. These materials are both composed solely of carbons. However,
the different bonding of the atoms in these substances gives them different properties.

In this chapter, we will have a look at various classes of compounds that illustrate the different
types of bonds and then develop models to describe the structure and bonding that characterize
materials found in nature.

8.1 Types of Chemical Bonds


Chemical Bond: A force that holds a group of atoms together and make them function as a unit.

We can obtain information about the strength of a bonding interaction by measuring the bond
energy, which is the energy required to break the bond.

Coulomb’s Law
The energy of interaction between a pair of ions

E = (2.31 x 10−19 J⋅nm) Q1Q2


r

Where E has the units of joules, r is the distance between the ion centers in nanometers, and Q1
and Q2 are the numerical ion charges.

Example:
+
In solid sodium chloride, the distance between the centers of the Na and Cl− ions is 2.76Å (0.276
nm), and the ionic energy per pair of ions is

E = (2.31 x 10−19 J⋅nm) (+1)(−1) = −8.37 x 10−19 J


0.276 nm

The negative sign indicates an attractive force (the ion pair has lower energy than the separated
ions).

Coulomb’s law can also be used to calculate the repulsive energy when two like-charged ions are
brought together. In that case, the calculated value of the energy will have a positive sign.

A bond will form if the system can lower its total energy in the process.

General Chemistry 202-NYA-05: Chapter 8 144


Energy profile of two hydrogen atoms

The energy terms involved are the net potential energy that results from the attractions and
repulsions among the charged particles and the kinetic energy due to the motions of the
electrons.

The zero point of energy is defined with atoms at infinite separation.

At very short distances the energy rises steeply because of the importance of the repulsive forces
when the atoms are very close together.

The bond length is the distance at which the system has minimum energy.

When we say that a bond is formed between the hydrogen atoms, we mean that the H2 molecule
is more stable than two separated hydrogen atoms by a certain quantity of energy (the bond
energy).

The type of bounding we encounter in the hydrogen molecule and in many other molecules in
which electrons are shared by nuclei is called covalent bonding.

Key Ideas in Bonding


In ionic bonding, the participating atoms are so different that one or more electrons are
transferred to form oppositely charged ions, which then attract each other.

In covalent bonding, two identical atoms share electrons equally. The bonding results from the
mutual attraction of the two nuclei for the shared electrons.

In polar covalent bonding, the atoms are not so different that electrons are completely transferred
but are different enough that unequal sharing results. The partial positive (δ+) and negative (δ−)
charges on the atoms (bond polarity) in such molecules as HF and H2O is that the electrons in
the bonds are not shared equally. For example, we can account for the polarity of the HF
molecule by assuming that the fluorine atom has a stronger attraction for the shared electrons
than the hydrogen atom.

General Chemistry 202-NYA-05: Chapter 8 145


8.2 Electronegativity
The different affinities of atoms for the electrons in a bond are described by a property called
electronegativity: The ability of an atom in a molecule to attract shared electrons to itself.

The most commonly used method of calculation of electronegativity values was originally
proposed by Linus Pauling (1901-1994). It gives a dimensionless quantity, commonly referred to
as the Pauling scale, on a relative scale running from 0.7 (for cesium) to 4.0 (for fluorine).

H−−F
δ+ δ−

Electronegativity generally increases across a period and decreases down a group.

For identical atoms (an electronegativity of zero), the electrons in the bond are shared equally,
and no polarity develops (covalent bond).
When two atoms with very different electronegativies interact, electron transfer can occur to form
the ions that make up an ionic substance.
Intermediate cases (an electronegativity not too large) give polar covalent bonds with unequal
electron sharing.

Example:
Order the following bonds according to polarity : H−H, H−F, H−Cl, H−Br, H−I, NaF.

Solution:
The polarity of the bond increases as the difference in electronegativity increases. From the
electronegativity values in the previous table, the following variation in bond polarity is expected
(the electronegativity value appears in parentheses below each element):

H−H < H−I < H−Br < H−Cl < H−F < NaF
(2.1)(2.1) (2.1)(2.5) (2.1)(2.8) (2.1)(3.0) (2.1)(4.0) (0.9)(4.0)

Electronegativity 0 0.4 0.7 0.9 1.9 3.1


difference:
Covalent bond Polar covalent bond Ionic bond
→ Polarity increases →

General Chemistry 202-NYA-05: Chapter 8 146


8.3 Bond Polarity and Dipole Moments
A molecule such as HF that has a center of positive charge and a center of negative charge is
said to be dipolar, or to have a dipolar moment. The dipolar moment of a molecule is often
represented by an arrow pointing to the negative charge center with the tail of the arrow indicating
the positive center of charge:

Polyatomic molecules also can exhibit dipolar behaviour.

For example, the oxygen atom in the water molecule has a greater electronegativity than the
hydrogen atoms. Because of this charge distribution, the water molecule behaves in an electric
field as if it had two centers of charge (one positive and one negative). The water molecule has a
dipole moment (so does NH3).

Charge distribution Behaviour in an electric field Electrostatic potential diagram

Some molecules have polar bounds but do not have a dipole moment. This occurs when the
individual bond polarities are arranged in such a way that they cancel each other out (ex: CO2).

General Chemistry 202-NYA-05: Chapter 8 147


Example:

HCl: The electronegativity of chlorine (3.0) is greater than that of hydrogen (2.1). Thus the
chlorine will be partially negative, and the hydrogen will be partially positive. The HCl
molecule has a dipole moment:

Cl2: The two chlorine atoms share the electrons equally. No bond polarity occurs, and the Cl2
molecule has no dipole moment.

SO3: The electronegativity of oxygen (3.5) is greater than of sulphur (2.5). Each oxygen will have
a partial negative charge, and the sulphur will have a partial positive charge. The bond
polarities arranged symmetrically as shown cancel, and the molecule has no dipole
moment:

General Chemistry 202-NYA-05: Chapter 8 148


CH4: Carbon has a slightly higher electronegativity (2.5) than does hydrogen (2.1). This leads to
small partial positive charges on the hydrogen atoms and a small partial negative charge
on the carbon. The molecule has no dipole moment:

H2S: The electronegativity of sulphur (2.5) is greater than that of hydrogen (2.1), the sulphur will
have a partial negative charge, and the hydrogen atoms will have a partial positive charge.
The polar bonds result in a dipole moment:

Bond Length

Single bond > double bond > triple bond

Bigger the difference in electronegativity between 2 atoms forming a bond, shorter the bond.

Bigger the difference in electronegativity between 2 atoms forming a bond, stronger the bond.

So overall, shorter bonds are stronger bonds (when comparison is done for elements in the same
period).

General Chemistry 202-NYA-05: Chapter 8 149


8.4 Ions: Electron Configurations and Sizes
In virtually every case, the atoms in a stable compound have a noble gas arrangement of
electrons. Nonmetallic elements achieve a noble gas electron configuration either by sharing
electrons with other nonmentals to form covalent bonds or by taking electrons from metals to form
ions.

When two nonmetals react to form a covalent bond, they share electrons in a way that completes
the valence electron configurations of both atoms. That is, both nonmetals attain noble gas
electron configurations.

When a nonmetal and a representative-group metal react to form a binary ionic compound, the
ions form so that the valence electron configuration of the nonmetal achieves the electron
configuration of the next noble gas atom. The valence orbitals of the metal are emptied. In this
way both ions achieve noble gas electron configurations.

Predicting Formulas of Ionic Compounds


In the solid state the ions are close together. That is, solid ionic compounds contain a large
collection of positive and negative ions packed together in a way that minimizes the repulsions
and maximizes the attractions. This situation stands in contrast to the gas phase of an ionic
substance, where the ions are quite far apart on average. In the gas phase, a pair of ions may get
close enough to interact, but large collections of ions do not exist.

In this section, the discussion describing the nature of ionic compounds usually refers to the solid
state, where many ions are simultaneously interacting.

Solid state of an ionic compound Gas phase of an ionic substance

To illustrate the principles of electron configurations in stable, solid ionic compounds, we will
consider the formation of an ionic compound from calcium and oxygen. We can predict what
compound will form by considering the valence electron configurations of the two atoms:
2
Ca [Ar] 4s
2 4
O [He] 2s 2p

The electronegativity of oxygen (3.5) is much greater than that of calcium (1.0). Because of this
large difference, electrons will be transferred from calcium to oxygen to form oxygen anions and
calcium cations in the compound. Noble gas configurations are generally the most stable.

Oxygen needs two electrons to fill its 2s and 2p valence orbitals and to achieve the configuration
2 2 6
of neon (1s 2s 2p ). By losing two electrons, calcium can achieve the configuration of argon. Two
electrons are therefore transferred:
2+ 2–
Ca + O —→ Ca + O

General Chemistry 202-NYA-05: Chapter 8 150


To predict the formula of the ionic compound, we simply recognize that chemical compounds are
always electrically neutral, they have the same quantities of positive and negative charges. In this
2+ 2–
case we have equal numbers of Ca and O ions, and the empirical formula of the compound is
CaO.

For example, let’s take a look at a compound formed between aluminium and oxygen. Because
2 1 3+
aluminium has the configuration [Ne] 3s 3p , it loses three electrons to form the Al ion and thus
3+ 2–
achieves the neon configuration. Therefore, the Al and O ions form in this case. Since the
compound must be electrically neutral, the compound has the empirical formula Al2O3.

In losing electrons to form cations, metals in Group 1A lose one electron, those in Group 2A lose
two electrons, and those in Group 3A lose three electrons. In gaining electrons to form anions,
nonmetals in Group 7A (the halogens) gain one electron, and those in Group 6A gain two
electrons. Hydrogen typically behaves as a non-metal and can gain one electron to form the

hydride ion (H ), which has the electronic configuration of helium. It can also lose its only electron
+
to become a proton (H ).

Exceptions:
2+ 4+
Tin can form both Sn and Sn
2+ 4+
Lead: Pb , Pb
3+ 5+
Bismuth: Bi , Bi
+ 3+
Thallium: Tl , Tl

The transition metals exhibit more complicated behaviour, forming a variety of ions that will not be
considered in this course.

General Chemistry 202-NYA-05: Chapter 8 151


Size of ions
Various factors influence ionic size. Since a positive ion is formed by removing one or more
electrons from a neutral atom, the resulting cation is smaller than its parent atom. The opposite is
true for negative ions; the addition of electrons to a neutral atom produces an anion significantly
larger than its parent atom.

The sizes of ions vary depending on the positions of the parent elements in the periodic table.
The following figure shows the sizes of the most important ions (each with a noble gas
configuration). Note that the size generally increases down a group.

In a series of isoelectronic ions (ions containing the same number of electrons), size decreases
2– – + 2+ 3+
with increasing atomic number. O , F , Na , Mg , and Al all have the neon electron
configuration. Since these ions are isoelectronic, the number of electrons is 10 in each case.
(Electron repulsions therefore should be about the same in all cases). However, the number of
2– 3+
protons increases from 8 to 13 as we go from the O ion to the Al ion. The 10 electrons
experience a greater attraction as the positive charge on the nucleus increases. This causes the
ions to become smaller.

General Chemistry 202-NYA-05: Chapter 8 152


Extra Problems:
1. The force between two bodies having identical electric charges
a) is a force of repulsion
b) is a force of repulsion if the charges are negative, and one of attraction if they are positive
c) increases as the bodies are moved further apart
d) is independent of the distance between them
e) is directly proportional to the distance between them
2. Which of the following groups contains no ionic compounds?
a) HCN, NO2, Ca(NO3)2
b) PCl5, LiBr, Zn(OH)2
c) KOH, CCl4, SF4
d) NaH, CaF2, NaNH2
e) CH2O, H2S, NH3
3. Which of the following bonds is least polar?
a) C—O
b) H—C
c) S—Cl
d) Br—Br
e) They are all nonpolar.
4. For the elements Cs, F, and P, the order of increasing electronegativity is:
a) Cs < F < P
b) Cs < P < F
c) P < F < Cs
d) F < Cs < P
e) none of these
5. For the elements Cs, F, and Cl, the order of increasing electronegativity is:
a) F < Cl < Cs
b) Cs < Cl < F
c) Cl < Cs < F
d) F < Cs < Cl
e) none of these
6. Based on electronegativities, which of the following would you expect to be most ionic?
a) N2
b) CaF2
c) CO2
d) CH4
e) CF4
7. In which case is the bond polarity incorrect?
δ+
a) H–Fδ–
δ+
b) K–Oδ–
δ+
c) Mg–Hδ–
δ+
d) Cl–Iδ–
δ+
e) Si–Sδ–
8. What is the correct order of the following bonds in terms of decreasing polarity?
a) N-Cl, P-Cl, As-Cl
b) P-Cl, N-Cl, As-Cl
c) As-Cl, N-Cl, P-Cl
d) P-Cl, As-Cl, N-Cl
e) As-Cl, P-Cl, N-Cl

General Chemistry 202-NYA-05: Chapter 8 153


9. Which of the following bonds would be the most polar without being considered ionic?
a) Mg-O
b) C-O
c) O-O
d) Si-O
e) N-O
10. Which of the following bonds would be the least polar, yet still be considered polar covalent?
a) Mg-O
b) C-O
c) O-O
d) Si-O
e) N-O
11. In which of the following compounds does the bond between the central atom and bromine have the
greatest ionic character?
a) LiBr
b) KBr
c) SeBr2
d) AsBr3
e) CaBr2
12. Which of the following molecules has no dipole moment?
a) CO2
b) NH3
c) H2O
d) all
e) none
13. Which of the following has the smallest radius?
a) Br–
b) S2–
c) Xe
d) Ca2+
e) Kr
14. Which of these is an isoelectronic series?
a) Na+, K+, Rb+, Cs+
b) K+, Ca2+, Ar, S2–
c) Na+, Mg2+, S2–, Cl–
d) Li, Be, B, C
e) none of these (A-D)
15. Which of the following pairs is isoelectronic?
a) Li+ and K+
b) Na+ and Ne
c) I– and Cl–
d) S2– and Ne
e) Al3+ and B3+
16. Which of the following arrangements is in order of decreasing size?
a) Ga3+ > Ca2+ > K+ > Cl– > S2–
b) S2– > Cl– > K+ > Ca2+ > Ga3+
c) Ga3+ > S2– > Ca2+ > Cl– > K+
d) Ga3+ > Ca2+ > S2– > Cl– > K+
e) Ga3+ > Ca2+ > S2– > K+ > Cl–

Ans: 1)a 2)e 3)d 4)b 5)b 6)b 7)d 8)e 9)d 10)e 11)b 12)a 13)d 14)b 15)b 16)b

General Chemistry 202-NYA-05: Chapter 8 154


8.6 Partial Ionic Character of Covalent Bonds
How well can we tell the difference between an ionic bond and a polar bond? The only honest
answer to this question is that there are probably no totally ionic bonds between discrete pairs of
atoms. The evidence of this statement comes from calculations of the percent ionic character for
the bonds of various binary compounds in the gas phase.

Percent ionic character of a bond = measured dipole moment of X—Y x 100%


+ –
calculated dipole moment of X Y

The relationship between the ionic character of a covalent bond and the electronegativity
difference of the bonded atoms

None of the bonds reaches 100% ionic character even with compounds that have a large
electronegativity difference (when tested in the gas phase).

These results are for the gas phase, where individual XY molecules exist. These results can not
necessarily be assumed to apply to the solid state, where the existence of ions is favoured by the
multiple ion interactions.

Note that the compounds with ionic character greater than 50% are normally considered to be
ionic compounds. Any compound that conducts an electric current when melted will be classified
as ionic.

8.7 The covalent Chemical Bond: A Model


Chemical bonds can be viewed as forces that cause a group of atoms to behave as a unit.

Bonds are neither inherently "good" not inherently "bad" as far as nature is concerned; bonds
result from the tendency of a system to seek its lowest possible energy. From a simplistic point of
view, bonds occur when collections of atoms are more stable together (lower in energy) than the
separate atoms.

General Chemistry 202-NYA-05: Chapter 8 155


For example:

Approximately 1652 kJ of energy is required to break a mole of methane (CH4) molecules into
separate C and H atoms. Or, taking the opposite view, 1652 kJ of energy is released when 1
mole of methane is formed from 1 mole of gaseous C atoms and 4 moles of gaseous H atoms.
Thus, we can say that 1 mole of CH4 molecules in the gas phase is 1652 kJ lower in energy than
1 mole of carbon atoms plus 4 moles of hydrogen atoms. Methane is a more stable molecule than
its separated atoms.

It the case of CH4, we envision four individual C—H interactions (that we call bond). The energy
of stabilization of CH4 is divided equally among the four bonds to give an average CH bond
energy per mole of CH bonds:
1652 kJ/mol = 413 kJ/mol
4

It is useful to interpret molecular stability in terms of model called a chemical bond.

The concept of bonds arose from the observations that most chemical processes involve
collections of atoms and that chemical reactions involve rearrangements of the ways the atoms
are grouped.

Models are attempts to explain how nature operates on the microscopic level based on
experiences in the macroscopic world. It is important to note that the bond concept is a human
invention. Bonds provide a method for dividing up the energy evolved when a stable molecule is
formed.

Although our concept of discrete bonds in molecules agrees with many of our observations, some
molecular properties require that we think of a molecule as a whole, with the electrons free to
move through the entire molecule. This is called delocalization of the electrons.

8.9 The localized Electron Bonding Model


A simple model that can be applied easily even to very complicated molecules and that can be
used routinely by chemists to interpret and organize the wide variety of chemical phenomena is a
localized electron (LE) model.

It assumes that a molecule is composed of atoms that are bound together by sharing pairs of
electrons using the atomic orbitals of the bound atoms. Electron pairs in the molecule are
assumed to be localized on a particular atom or in the space between two atoms. Those pairs of
electrons localized on an atom are called lone pairs, and those found in the space between the
atoms are called bonding pairs.

The Model has 3 parts:

1. Description of valence electron arrangement in the molecule using Lewis structures.


2. Prediction of the geometry of the molecule using the valence shell electron-pair repulsion
(VSEPR) model.
3. Description of the type of atomic orbitals used by the atoms to share electrons or hold
lone pairs.

General Chemistry 202-NYA-05: Chapter 8 156


8.10 Lewis Structures
The Lewis structure of a molecule shows how the valence electrons are arranged among the
atoms in the molecule. These representations are named after the American physical chemist G.
N. Lewis (1875-1946).

The most important requirement for the formation of a stable compound is that the atoms achieve
noble gas electron configurations.

We have already seen that when metals and nonmetals react to form binary ionic compounds,
electrons are transferred and the resulting ions typically have noble gas electron configurations.
+ –
An example is the formation of KBr, where K ion has the [Ar] electron configuration and the Br
ion has the [Kr] electron configuration. In writing Lewis structures, the rule is that only the valence
electrons are included. Using dots to represent electrons, the Lewis structure for KBr is

K Br
+
1 charge 1− charge
+ –
No dots are shown on the K ion because it has no valence electrons. The Br ion is shown with
eight electrons because it has a filled valence shell.

Individual atoms represented by the Lewis structures (also called Lewis-dot diagrams):

We will now consider Lewis structures for molecules with covalent bonds, involving elements in
the first and second periods.

General Chemistry 202-NYA-05: Chapter 8 157


The Duet Rule:

Hydrogen forms stable molecules where it shares two electrons. For example, when two
hydrogens atoms, each with one electron, combine to form the H2 molecule, we have:

By sharing electrons, each hydrogen in H2, in effect, has two electrons; that is, each hydrogen
has a filled valence shell.

Helium does not form bonds because its valence orbital is already filled; it is a noble gas. Helium
2
has the electron configuration 1s and can be represented by the Lewis structure :He

The Octet Rule:

The second-row nonmetals carbon through fluorine form stable molecules when they are
surrounded by enough electrons to fill the valence orbitals, that is, the 2s and the three 2p
orbitals. Since eight electrons are required to fill these orbitals, these elements typically obey the
octet rule (when bonding together, they end up being surrounded by eight electrons).

F F F F

Each fluorine atom in F2 is surrounded by 8 electrons, two of which are shared with the other
atom. This is a bonding pair of electrons, as discussed earlier. Each fluorine atom also has
three pairs of electrons not involved in the bonding. Those are called lone pairs of electrons.

Neon does not form bonds because it already has an octet of valence electrons (noble gas).

Steps for Writing Lewis Structures

1. Sum the valence electrons from all the atoms. (Do not worry about keeping track of which
electrons come from which atoms. It is the total number of electrons that is important).
2. Use a pair of electrons to form a bond between each pair of bound atoms.
3. Atoms usually have noble gas configurations. Arrange the remaining electrons to satisfy
the octet rule (or duet rule for hydrogen).

Example: Water (H2O)

1. We sum the valence electrons for H2O:


1 + 1 + 6 = 8 valence electrons

2. Using a pair of electrons per bond, we draw in the two O−H single bonds:
HOH

A line is used instead of a pair of dots to indicate each pair of bonding electrons. This is the
standard notation.

General Chemistry 202-NYA-05: Chapter 8 158


3. We distribute the remaining electrons to achieve a noble gas electron configuration for
each atom. Since four electrons have been used in forming the two bonds, four electrons
(8 − 4) remain to be distributed. Hydrogen is satisfied with two electrons (duet rule), but
oxygen needs eight electrons to have a noble gas configuration. Thus, the remaining four
electrons are added to oxygen as two lone pairs. (Dots are used to represent the lone
pairs.
H O H

Example: Carbon dioxide (CO2)

1. We sum the valence electrons for CO2:


4 + 6 + 6 = 16 valence electrons

2. Some electrons are used to form a bond between the carbon and each oxygen:
OCO

3. The remaining electrons are distributed to achieve noble gas configuration on each atom.
In this case we have 12 electrons (16 − 4) remaining after the bonds are drawn. The
distribution of these electrons is determined by a trial-and-error process. We have 6 pairs
of electrons to distribute. To make sure all atoms are surrounded by 8 electrons, we have
to introduce two double bonds and four lone pairs.

O C O

Examples: Cyanide ion (CN−)


C N

Tribromophosphine (PBr3)
Br
Br P Br

Nitrite ion (NO2−)

N
O O

Chloroethyne (C2HCl)
H C C Cl

Propionic acid (CH3CH2CO2H)


H H O
H C C C O H
H H

General Chemistry 202-NYA-05: Chapter 8 159


8.11 Exceptions to the Octet Rule
Boron tends to form compounds in which the boron atom has fewer than eight electrons around
it; it does not have a complete octet.

For example, boron trifluoride (BF3) reacts very energetically with molecules that have available
electron pairs (lone pairs). The violent reactivity of BF3 with electron-rich molecules arises
because the boron atom is electron deficient. Boron trifluoride has 24 valence electrons. The
Lewis structure for BF3 is:

F
B
F F

In a reaction with NH3, it will form H3NBF3

H F H F
H N + B F H N B F
H F H F
In this stable compound, boron has an octet of electrons.

Some atoms exceed the octet rule. This behaviour is observed only for those elements in Period
3 of the periodic table and beyond.

For example, sulfur hexafluoride (SF6) which has 48 valence electrons.

FFF
S
FFF
Sulfur has 12 electrons around it; that is, sulphur exceeds the octet rule.

The second-row elements have 2s and 2p orbitals fill with electrons in going from sodium to
argon, but the 3d orbitals remain empty. The valence orbital diagram for a sulphur atom is

3s 3p 3d

The localized electron model assumes that the empty 3d orbitals can be used to accommodate
extra electrons. Thus the sulphur atom in SF6 can have 12 electrons around it by using the 3s
and 3p orbitals to hold 8 electrons, with the extra 4 electrons placed in the formerly empty 3d
orbitals.

For example, phosphorus pentachloride (PCl5) which has 40 valence electrons:

Cl Cl Cl
P
Cl Cl

General Chemistry 202-NYA-05: Chapter 8 160


In sum
• C, N, O, and F should always be assumed to obey the octet rule.
• B and Be often have fewer than 8 electrons around them in their compounds.
• Second-row elements never exceed the octet rule.
• Third-row and heavier elements often satisfy the octet rule but can exceed the octet rule
by using their empty valence d orbitals.
• When writing the Lewis structure for a molecule, satisfy the octet rule for the atoms first. If
electrons remain after the octet rule has been satisfied, then place them on the elements
having available d orbitals (elements in Period 3 or beyond).

Molecules containing multiple atoms exceeding the octet rule

In molecules having more than one atom that can exceed the octet rule, it is not always clear
which atom should have the extra electrons.

For example, the triiodide ion (I3−) has 22 valence electrons.

When it is necessary to exceed the octet rule for one several third-row (or higher) elements,
assume that the extra electrons should be placed on the central atom.

Thus, for I3− the Lewis structure is

I I I

Example of different valid Lewis structures:

H
B
BH3 (6e) : H H

Br BrBr
As
Br Br
AsBr5 (40e) :
F
F S F
F
SF4 (34e) :
F
Cl F
F
ClF3 (28e) :
O
Xe O
O
XeO3 (26e):
Br Rn Br
RnBr2 (22e):

General Chemistry 202-NYA-05: Chapter 8 161


Extra Problems:

1. As indicated by Lewis structures, which of the following would probably not exist as a stable
molecule?
a) CH3OH
b) CH2O
c) CH3O
d) C2H2
e) C3H4
2. Complete the Lewis structure for the molecule:
CH3 O
CH3 CH C C N
This molecule has __________ single bonds and __________ multiple bonds.
a) 4, 2
b) 6, 3
c) 11, 5
d) 11, 2
e) 13, 0

Use the following to answer questions 3-5:

Consider the compound crotonaldehyde, whose skeleton is:


H H H H
H C1 C2 C3 C4 O
H
3. How many electrons must be shown (as bonding or nonbonding electrons) in the Lewis structure of
this molecule?
a) 12
b) 18
c) 24
d) 28
e) 32
4. How many nonbonding electrons appear in the Lewis structure of this molecule?
a) 2
b) 4
c) 6
d) 8
e) 10
5. Which carbon in this molecule has tetrahedral bonding?
a) 1
b) 2
c) 3
d) 4
e) all

General Chemistry 202-NYA-05: Chapter 8 162


6. Which of the following molecules contains a double bond?
a) CO2
b) NH3
c) H2O
d) all
e) none
7. When molten sulfur reacts with chlorine gas, a vile-smelling orange liquid forms that is found to have
the empirical formula SCl. Which of the following could be the correct Lewis structure for this
compound?
a)
S Cl
b)
S Cl
c)
S Cl Cl S
d)
Cl S S Cl

e)
Cl S S Cl:

8. Which of the following has a Lewis structure most like that of CO32–?
a) CO2
b) SO32–
c) NO3–
d) O3
e) NO2
9. Choose the electron dot formula that most accurately describes the bonding in CS2. (Hint: Consider
formal charges.)
a) S C S
b) C S S
c) S C S
d) S C S
e) S C S
10. Which of the following has an incomplete octet in its Lewis structure?
a) SO2
b) ICl
c) CO2
d) F2
e) NO
11. In the Lewis structure for ICl2–, how many lone pairs of electrons are around the central iodine atom?
a) 0
b) 1
c) 2
d) 3
e) 4
12. As indicated by Lewis structures, which of the following species could probably not exist as a stable
molecule?
a) NH3
b) N2H2
c) N2H4
d) N2H6
e) N2O4
Ans: 1)c 2)d 3)d 4)b 5)a 6)a 7)e 8)c 9)a 10)e 11)d 12)d

General Chemistry 202-NYA-05: Chapter 8 163


8.12 Resonance
Sometimes more than one valid Lewis structure is possible for a given molecule. For example,
the nitrate ion (NO3−):

N
O O

Although the structure above is a valid Lewis structure, it does not correctly represent the bonding
in NO3−. There is no reason for choosing a particular oxygen atom to have the double bond. It has
three equivalent bonds. The correct description of NO3− is not given by any one of the three Lewis
structures but is given only by the superposition of all three:

O O O

N N N
O O O O O O

Resonance is invoked when more than one valid Lewis structure can be written for a particular
molecule. The resulting electron structure of the molecule is given by the average of these
resonance structures.

Note that in all these resonance structures the arrangement of the nuclei is the same. Only the
placement of the electrons differs. The arrows do not mean that the molecule “flips” from one
resonance to another. They simply show that the actual structure is an average of the three
resonance structures.

Resonance is necessary to compensate for the defective assumption of the localized electron
model. However, this model is so useful that we retain the concept of localized electrons and add
resonance to allow the model to treat species such as NO3−.

Example:
Give the Lewis structures for nitrite anion (NO2−).
Solution:
In NO2− there is 18 valence electrons:

N N
O O O O

Odd-Electron Molecules
Since the localized electron model is based on pairs of electrons, it does not handle odd-electron
cases, such as nitric oxide (NO). A more sophisticated model is needed.

General Chemistry 202-NYA-05: Chapter 8 164


Formal Charge
Although oxidation states are useful for bookkeeping electrons in redox reactions, they are not
realistic estimates of the actual charges on individual atoms in a molecule, so they are not
suitable for judging the appropriateness of Lewis structures. Another definition of the charge on
an atom in a molecule, the formal charge, can, however, be used to evaluate Lewis structures.

The formal charge of an atom in a molecule is the difference between the number of valence
electrons on the free atom and the number of valence electrons assigned to the atom in the
molecule.

To determine the formal charge of an atom in a molecule:

1. The number of valence electrons on the free neutral atom (which has a zero net charge
because the number of electrons equals the number of protons)
2. The number of valence electrons “belonging” to the atom in a molecule

We then compare these numbers. If in the molecule the atom has the same number of valence
electrons as it does in the free state, the positive and negative charges just balance, and it has a
formal charge of zero. If the atom has one more valence electron in a molecule than it has as a
free atom, it has a formal charge of −1, and so on. Thus, the formal charge on an atom in a
molecule is defined as

Formal charge = (# valence é on free atom) – (# valence é assigned to the atom in the molecule).

To compute the formal charge of an atom in a molecule, we assign the valence electrons in the
molecule to the various atoms, making the following assumptions:

1. Lone pair electrons belong entirely to the atom in question


2. Shared electrons are divided equally between the two sharing atoms.

Thus the number of valence electrons assigned to a given atom is calculated as follows:

(Valence electrons)assigned = (number of lone pair electrons) + ½ (number of shared electrons)


2−
Example: The sulphate ion (SO4 ) which has 32 valence electrons.

2 2 2
O O O
O S O O S O O S O
O O O

Formal charge on oxygen = −1 Formal charge on oxygen = 2 of 0 and 2 of −1


Formal charge on sulphur = +2 Formal charge on sulphur = 0

To evaluate Lewis structures, we will use the two following assumptions:

1. Atoms in molecules try to achieve formal charges as close to zero as possible


2. Any negative formal charges are expected to reside on the most electronegative atoms

General Chemistry 202-NYA-05: Chapter 8 165


Rules governing Formal charge

To calculate the formal charge on an atom:


1. Take the sum of the lone pair electrons and one-half the shared electrons. (This is the
number of valence electrons assigned to the atom in the molecule)
2. Subtract the number of assigned electrons from the number of valence electrons on the
free, neutral atom to obtain the formal charge.
• The sum of the formal charges of all atoms in a given molecule or ion must equal the
overall charge on that species.
• If nonequivalent Lewis structures exist for a species, those with formal charges closest to
zero and with any negative formal charges on the most electronegative atoms are
considered to best describe the bonding in the molecule or ion.

Example:
Give the possible Lewis structures for XeO3 (26 valence electrons). Which structure are the most
appropriate according to the formal charges?

Solution:
(+3) (+2) (+2) (+2)
Xe Xe Xe Xe
O O O O O O O O
(-1) O (-1) (0) O (-1) (-1) O (-1) (-1) O (0)
(-1) (-1) (0) (-1)

(+1) (+1) (+1) (0)


Xe Xe Xe Xe
O O O O O O O O
(-1) O (0) (0) O (0) (0) O (-1) (0) O (0)
(0) (-1) (0) (0)

Although formal charges are closer to actual atomic charges in molecules than are oxidation
states, formal charges still provide only estimates of charges – they should not be taken as actual
atomic charges. The evaluation of Lewis structures using formal charge ideas can lead to
erroneous predictions.

General Chemistry 202-NYA-05: Chapter 8 166


8.13 Molecular Structure: The VSEPR Model
Molecular structure: The three dimensional arrangement of the atoms in a molecule.

The valence shell electron-pair repulsion (VSEPR) model is useful in predicting the geometries of
molecules formed from nonmetals. The main postulate of this model is that structure around a
given atom is determined principally by minimizing electron-pair repulsions. The idea here is that
the bonding and nonbonding pairs around a given atom will be positioned as far apart as
possible.

Geometrical Bond Molecular Polar


Example
Structure Angle Scheme (yes or no)
X A X Cl Be Cl
Linear 180° No
180o 180o
V-shaped A O
(angular planar X X H H Yes
<180°
o o
or bent) <180 105
X F
Trigonal planar 120° 120o 120o No
A B
X X F F

X 109.5o
Tetrahedral H 109.5o No
109.5° A
X X
X C H
H H

Trigonal pyramidal <120° Yes

Steps to Apply the VSEPR Model

1. Draw the Lewis structure for the molecule.


2. Count the electron pairs and arrange them in the way that minimizes repulsion (put the
pairs as far apart as possible.
3. Determine the positions of the atoms from the way electron pairs are shared (how
electrons are shared between the central atom and surrounding atoms).
4. Determine the name of the molecular structure from positions of the atoms.

General Chemistry 202-NYA-05: Chapter 8 167


The XAX bond angle in CH4, NH3 and H2O should be tetrahedral angle of 109.5 degrees.
However, experimental studies show that the bond angle between bonding pairs decreases as
the number of lone pairs increases.

Lone pairs require more space than bonding pairs. Since lone pairs require more room than
bonding pairs they tend to compress the angles between the bonding pairs.

A 120° angle provides lone pairs with enough space so that distortions do not occur. Angles less
than 120° are distorted when lone pairs are present.

General Chemistry 202-NYA-05: Chapter 8 168


Molecules with four pairs of electrons around the central atom can be tetrahedral (AB4), trigonal
pyramid (AB3), and V-shaped (AB2).

Molecules with five pairs of electrons around the central atom can be trigonal bipyrimidal (AB5),
see-saw (AB4), T-shaped (AB3), and linear (AB2).

General Chemistry 202-NYA-05: Chapter 8 169


Molecules with six pairs of electrons can best be arranged around a given atom with 90-degree
angles to form an octahedral structure. Its related structures also include square pyramidal (AB5)
and square planar (AB4).

Please do the suggested problems as well as example 8.12 and 8.13.

The VSEPR Model and Multiple bonds

Multiple bonds count as one effective electron pair. When a molecule exhibits resonance, any
one of the resonance structures can be used to predict the molecular structure using the VSEPR
model. For example, the NO3− ion:

O O O

N N N
O O O O O O

Molecules Containing No Single Central Atom

The molecular structure sums the arrangements around every atom present in the molecule. For
example, methanol:

H
H C O H
H

C has O has overall:


é pair arr.: Tetrahedral é pair arr.: Tetrahedral Doesn’t exist.
Mol. Geo.: Tetrahedral Mol. Geo.: Bent It is per atom.

General Chemistry 202-NYA-05: Chapter 8 170


Extra Problems:

1. Which of these molecules show resonance: (I) BH3, (II) NO2−, (III) SF6, (IV) O3, (V) PCl5 ?
a) I, II
b) II, IV
c) II, V
d) III, IV
e) III, V
2. This molecule contains a carbon atom with trigonal planar geometry.
a) CH3CHO
b) CO2
c) CH3Cl
d) C2H6
e) none of these
3. Which carbon in this molecule has tetrahedral bonding? H H H H
a) 1
b) 2 H C1 C 2 C 3 C 4 O
c) 3
d) 4 H
e) all
4. Which of the following molecules contains a double bond?
a) CO2
b) NH3
c) H2O
d) all
e) none

Use the following to answer questions 5-7:


Consider the following molecules :
I. BF3
II. CHBr3 (C is the central atom)
III. Br2
IV. XeCl2
V. CO
VI. SF4
Select the molecule(s) that fit the given statement.
5. These molecules violate the octet rule.
a) I, II, IV
b) I, III, IV, VI
c) III, V, VI
d) I, IV, VI
e) I, II, IV, VI
6. These molecules have a zero net dipole moment.
a) III, V
b) I, III, IV
c) III, IV, V
d) I, III, IV, VI
e) none of them

General Chemistry 202-NYA-05: Chapter 8 171


7. These molecules have a trigonal bipyramidal electron pair arrangement.
a) II, IV, VI
b) I, IV
c) IV, VI
d) VI only
e) none of them
8. Which of the following exhibits resonance?
a) NH3
b) SCl6
c) H2O
d) SO2
e) At least two of the molecules (A-D) exhibit resonance.
9. Which of the following species is best described by drawing resonance structures?
a) PH3
b) NH4+
c) O3
d) SO3
e) HCN
10. In the cyanide ion (CN–), the nitrogen has a formal charge of
a) -2
b) -1
c) 0
d) 2
e) More information is needed.
11. How many resonance structures can be drawn for the molecule O3?
a) 1
b) 2
c) 3
d) 4
e) 5
12. Which of the following is not a valid resonance structure for N3–?
a) c)
- -
N N N N N N
b) d)
- -
N N N N N N

13. Choose the statement that best describes the PbCl4 molecule in the gas phase.
a) The bond angles are all about 109°.
b) The molecule is polar.
c) The molecule has a dipole moment.
d) The bonds are nonpolar.
e) More than one of the above.
14. Which of the following types of molecules always has a dipole moment?
a) Linear molecules with two identical bonds.
b) Tetrahedral molecules (four identical bonds equally spaced).
c) Trigonal pyramid molecules (three identical bonds) (except for PH3).
d) Trigonal planar molecules (three identical bonds equally spaced).
e) None has a dipole moment.

General Chemistry 202-NYA-05: Chapter 8 172


15. Of the following, which molecule has the largest bond angle?
a) O3
b) OF2
c) HCN
d) H2O
e) More than one of the above have equally large bond angles.
16. According to the VSEPR model, the arrangement of electron pairs around NH3 and CH4 is
a) different, because in each case there are a different number of atoms around the central atom
b) different, because in each case there are a different number of electron pairs around the central
atom
c) the same, because both nitrogen and carbon are both in the second period
d) the same, because in each case there are the same number of electron pairs around the central
atom
e) different or the same, depending on the conditions leading to maximum repulsion
17. The Cl–Kr–Cl bond angle in KrCl4 is closest to
a) 90°
b) 109°
c) 120°
d) 150°
e) 360°
18. Which of the following molecules has a nonlinear structure?
a) XeF2
b) BeCl2
c) O3
d) CO2
e) N2O (central atom is N)
19. Which of the following molecules are nonlinear?
NO2–, C2H2, N3–, HCN, CO2, H2O2
a) C2H2, HCN
b) CO2, N3–
c) NO2–, H2O2
d) N3–, NO2–
e) all are linear
20. How many of the following molecules or ions are linear?
NH3 OF2 HCN CO2 NO2
a) 0
b) 1
c) 2
d) 3
e) 4
21. The molecular structure of SOCl2 is
a) a) pyramidal
b) b) none of these
c) c) octahedral
d) d) trigonal planar
e) e) bent

General Chemistry 202-NYA-05: Chapter 8 173


22. The molecular structure of OF2 is
a) a) pyramidal
b) b) none of these
c) c) octahedral
d) d) trigonal planar
e) e) bent
23. The molecular structure of NCl3 is
a) a) pyramidal
b) b) none of these
c) c) octahedral
d) d) trigonal planar
e) e) bent
24. The molecular structure of BeF3– is
a) a) pyramidal
b) b) none of these
c) c) octahedral
d) d) trigonal planar
e) e) bent
25. The molecular structure of BrF6+ is
a) a) pyramidal
b) b) none of these
c) c) octahedral
d) d) trigonal planar
e) e) bent
26. The molecular structure of AsCl5 is
a) trigonal bipyramidal
b) square pyramidal
c) distorted tetrahedral
d) octahedral
e) none of these
27. The molecular structure of XeF5+ is
a) trigonal bipyramidal
b) square pyramidal
c) distorted tetrahedral
d) octahedral
e) none of these
28. What type of structure does the XeOF2 molecule have?
a) pyramidal
b) tetrahedral
c) T-shaped
d) trigonal planar
e) octahedral
29. The bond angles about the carbon atom in the formaldehyde molecule, H2C=O, are about:
a) 120°
b) 60°
c) 109°
d) 180°
e) 90°

General Chemistry 202-NYA-05: Chapter 8 174


30. Which of the following species has a trigonal bipyramid structure?
a) NH3
b) IF5
c) I3–
d) PCl5
e) none of these
31. The bond angle in H2Se is about:
a) 120°
b) 60°
c) 180°
d) 109°
e) 90°
32. Which ion is planar?
a) NH4+
b) CO32–
c) SO32–
d) ClO3–
e) all are planar

Use the following to answer questions 33-36:


Select the correct molecular structure for the given species from the choices below:
33. PF6–
a) pyramidal
b) tetrahedral
c) square planar
d) octahedral
e) none of these
34. PCl4+
a) pyramidal
b) tetrahedral
c) square planar
d) octahedral
e) none of these
35. XeF6
a) pyramidal
b) tetrahedral
c) square planar
d) octahedral
e) none of these
36. NI3
a) pyramidal
b) tetrahedral
c) square planar
d) octahedral
e) none of these

Ans: 1)b 2)a 3) a 4)a 5)d 6)b 7)c 8) d 9)c 10)c 11)b 12)a 13)a 14)c 15)c 16)d 17)a 18)c 19)c 20)c
21)a 22)e 23)a 24)d 25)c 26)a 27)b 28)c 29)a 30)d 31)d 32)b 33)d 34)b 35)e 36)a

General Chemistry 202-NYA-05: Chapter 8 175


Summary:

General Chemistry 202-NYA-05: Chapter 8 176


Chapter 9

Covalent Bonding: Orbitals

th
Suggested problems in chapter 9 of Zumdahl 8 Edition:

17, 19, 21, 27, 29, 30, 31, 33, 35, 36, 37, 39, 49, 51, 55.

General Chemistry 202-NYA-05: Chapter 9 177


9.1 Hybridization and the localized Electron Model
The localized electron model views a molecule as a collection of atoms bound together by
sharing electrons between their atomic orbitals. The arrangement of valence electrons is
represented by the Lewis structure and the molecular geometry can be predicted from the
VSEPR model.

sp3 Hybridization
In general, we assume that bonding involves only the valence orbitals. This means that the
hydrogen atoms in methane (CH4) use 1s orbitals. The valence orbitals of a carbon atom are the
2s and 2p orbitals. The carbon atom has four equivalent atomic orbitals arranged tetrahedrally.

The “native” 2s and three 2p atomic orbitals characteristic of a free carbon atom are combined to
3
form a new set of four sp orbitals.

This mixing of the native atomic orbitals to form special orbitals for bonding is called
hybridization.
3
The four new orbitals are called sp orbitals because they are formed from one 2s orbital and
1 3 3 3
three 2p orbitals (s p ). We say that the carbon atom undergoes sp hybridazation or is sp
hybridized.
3
The four sp orbitals are identical in shape, each one having a large lobe and a small lobe. The
four orbitals are oriented in space so that the large lobes form a tetrahedral arrangement. The
hybridization of the carbon 2s and 2p orbitals also can be represented by an orbital energy-level
diagram.

General Chemistry 202-NYA-05: Chapter 9 178


sp2 Hybridization
A double bond acts as one effective pair, so in the ethylene molecule (H2C=CH2) each carbon is
surrounded by three effective pairs. This requires a trigonal planar arrangement with bond angles
of 120 degrees.
2
Since one 2s and two 2p orbitals are used to form three hybrids orbitals, this is called sp
hybridization.

2
In forming the sp orbitals, one 2p orbital on carbon has not been used. This remaining p orbital is
2
oriented perpendicular to the plane of the sp orbitals.

Two different types of bonds are present. The covalent bonds where the electron pair is shared in
an area centered on a line running between the atoms are called sigma (σ
σ) bond. In the ethylene
2
molecule, the s bonds are formed using sp orbitals on each carbon atom and the 1s orbital on
each hydrogen atom.

The second bond must therefore result from sharing an electron pair in the space above and
below the s bond. This type of bond can be formed using the 2p orbital perpendicular to the sp2
hybrid orbitals on each carbon atom. These parallel p orbitals can share an electron pair, which
occupies the space above and below a line joining the atoms, to form a pi (π
π) bond.

General Chemistry 202-NYA-05: Chapter 9 179


Note that σ bonds are formed from orbitals whose lobes point toward each other, but π bonds
result from parallel orbitals. A double bond always consists of one σ bond, where the electron pair
is located directly between the atoms, and one π bond, where the shared pair occupies the space
above and below the σ bond.
2
Whenever an atom is surrounded by three effective pairs, a set of sp hybrid orbitals is required.

sp Hybridization
sp Hybridization involve one s orbital and one p orbital.

In terms of this model, two effective pairs around an atom will always require sp hybridization of
that atom.

For example, in CO2, sp hybrid orbitals are used to form the s bonds between the carbon and the
oxygen atoms. Two 2p orbitals remain unchanged on the sp hybridized carbon. These are used
to form the p bonds with the oxygen atoms.

Another molecule whose bonding can be described by sp hybridization is acetylene, where each
carbon atom requires sp hybridization, leaving two unchanged p orbitals. One of the oppositely
oriented sp orbitals is used to form a bond to the hydrogen atom.

H C C H

General Chemistry 202-NYA-05: Chapter 9 180


dsp3 Hybridization
Trigonal bipyramidal arrangement formed by dsp3 hybridization of one d orbital, one s orbital, and
three p orbitals.

A set of five effective pairs around a given atom always requires a trigonal bipyramidal
3
arrangement, which in turn requires dsp hybridization of that atom.
3
For example, in PCl5, the five P−Cl σ bonds are formed by sharing electrons between a dsp
orbital on the phosphorous atom and an orbital on each chlorine.

d2sp3 Hybridization
2 3
Octahedral arrangement of six pairs of electrons around a central atom. In the d sp hybridization
orbitals, two d orbitals, one s orbital and three p orbitals are combined.

Using the Localized Electron Model: Summary

• Draw the Lewis structure(s).


• Determine the arrangement of electron pairs using the VSEPR model geometrical
structure.
• Specify the hybrid orbitals needed to accommodate the electron pairs

General Chemistry 202-NYA-05: Chapter 9 181


General Chemistry 202-NYA-05: Chapter 9 182
9.2 The molecular Orbital Model
Another model often used to describe bonding is the Molecular Orbital Model.

To introduce the assumptions, methods and results of this model, we will consider the simplest of
all molecules, H2, which consists of two protons and two electrons. A very stable molecule, H2 is
lower in energy than the separated hydrogen atoms by 432 KJ/mol.

When the quantum mechanical equations (complicated probably distribution theory) for the
hydrogen molecule are solved, two molecular orbitals result, which can be represented as:

MO1 = 1sA + 1sB = σ1s = bonding molecular orbital


MO2 = 1sA − 1sB = σ1s* = antibonding molecular orbital

The orbital properties of most interest are size, shape and energy.

• MO1 and MO2 are sigma (σ) molecular orbitals.


• In a molecule only the molecular orbitals are available for occupation by electrons.
• MO1 is lower in energy than the 1s orbitals of free hydrogen atoms, while MO2 is higher in
energy than the 1s orbitals.
• Nature tends to seek the lowest energy state.
• A bonding molecular orbital is lower in energy than the atomic orbitals of which it is
composed. Electrons in this type of orbital will favour the molecule (they will favour bonding).
• An antibonding molecular orbital is higher in energy than the atomic orbitals of which it is
composed. Electrons in this type of orbital will favour the separated atoms (they are
antibonding).

General Chemistry 202-NYA-05: Chapter 9 183


Bond Order
To indicate bond strength, we use the concept of bond order. Bond order is the difference
between the number of bonding electrons and the number of antibonding electrons divided by 2.

Bond order = number of bonding electrons − number of antibonding electrons


2

Larger bond order means greater bond strength.

H2 molecule H2− ion He2 molecule

Bond Order: 1 ½ 0

The bond in the H2 molecule is predicted to be about twice as strong as the bond in the H2− ion.

The He2 molecule is not stable (does not exist).

Extra Problems:

1. The hybridization of the nitrogen atom in the cation NH2+ is:


a) sp2
b) sp3
c) dsp
d) sp
e) none of these
2. In the molecule C2H4 the valence orbitals of the carbon atoms are assumed to be
a) not hybridized
b) sp hybridized
c) sp2 hybridized
d) sp3 hybridized
e) dsp hybridized

General Chemistry 202-NYA-05: Chapter 9 184


3. The hybridization of the central atom in XeF5+ is:
a) sp
b) sp2
c) sp3
d) dsp3
e) d2sp3
4. The hybridization of the central atom in ClF2+ is:
a) sp
b) sp2
c) sp3
d) dsp3
e) d2sp3
5. The hybridization of the central atom in I3– is:
a) sp
b) sp2
c) sp3
d) dsp3
e) d2sp3
6. The hybridization of the central atom in O3 is:
a) sp
b) sp2
c) sp3
d) dsp3
e) d2sp3
7. Which of the following molecules contains a central atom with sp2 hybridization?
a) b) c) d) e)

8. What hybridization is predicted for the nitrogen atom in the NO3– ion?
a) sp2
b) sp3
c) dsp3
d) d2sp3
e) none of these
9. Which of the following does not contain at least one pi bond?
a) HCN
b) O3
c) C2H4
d) C3H8
e) All of the above (A-D) contain at least one pi bond.
10. The hybridization of the central atom in NO3– is
a) p3
b) sp2
c) sp3
d) sp
e) dsp2

General Chemistry 202-NYA-05: Chapter 9 185


11. Consider the following Lewis structure:
H H H
H C O C C C C N H
1 2 3 4
H H
What is the hybridization of the atoms O, C-1, C-2, and C-4?
a) sp3 sp3 sp sp2
b) sp sp3 sp sp
c) sp sp2 sp sp2
d) sp2 sp3 sp2 sp3
e) sp3 sp sp sp2

Use the following to answer questions 12-15:

Tetracyanoethylene has the skeleton shown below:

From its Lewis structure determine the following:


12. How many sigma and pi bonds are in the molecule?
a) 4 sigma and 5 pi
b) 6 sigma and 8 pi
c) 9 sigma and 8 pi
d) 9 sigma and 9 pi
e) 5 sigma and 8 pi
13. How many nonbonded electron pairs are in the molecule?
a) 0
b) 2
c) 4
d) 5
e) 8
14. How many of the atoms are sp2 hybridized?
a) 2
b) 4
c) 6
d) 8
e) 10
15. How many of the atoms are sp hybridized?
a) 2
b) 4
c) 6
d) 8
e) 10

General Chemistry 202-NYA-05: Chapter 9 186


16. Which of the following substances contains two pi bonds?
a) C2H4
b) C3H8
c) C2H2
d) C2H6
e) CH4
17. Consider the molecule C2H4. The hybridization of each C atom is
a) sp
b) sp2
c) sp3
d) dsp3
e) d2sp3
18. T F Whenever a set of equivalent tetrahedral atomic orbitals is required, an atom will adopt a set
of sp3 orbitals.
19. T F The hybridization of the B in BH3 is sp3.
20. T F The hybridization of a molecule is measured to determine the shape of the molecule.
21. Which of the following statements is true?
a) Electrons are never found in an antibonding MO.
b) All antibonding MOs are higher in energy than the atomic orbitals of which they are composed.
c) Antibonding MOs have electron density mainly outside the space between the two nuclei.
d) None of the above is true.
e) Two of the above statements are true.
22. Which of the following statements is (are) correct?
2
I. The hybridization of boron in BF is sp .
3
II. The molecule XeF4 is nonpolar.
III. The bond order of N2 is three.
IV. The molecule HCN has two pi bonds and two sigma bonds.
23. Which of the following molecules contains the shortest C–C bond?
a) C2H2
b) C2H4
c) C2H6
d) C2Cl4
e) b and d
24. The electron configuration of a particular diatomic species is (σ2s)2(σ*2s)2(σ2p)2(π2p)4(π*2p)4. What is
the bond order for this species?
a) 2.5
b) 2
c) 1.5
d) 1
e) 0.5
25. What is the bond order of He2+?
a) 0
b) 1
2
c) 1
d) 1 12
e) 2
26. T F Larger bond order means greater bond strength.
Ans: 1)a 2)c 3)e 4)c 5)d 6)b 7)b 8)a 9)d 10)b 11)a 12)d 13)c 14)a 15)d 16)c 17)b 18) True
19)False 20)False 21)e 22)all 23)a 24)d 25)b 26)True

General Chemistry 202-NYA-05: Chapter 9 187


9.3 Bonding in Homonuclear Diatomic Molecules
Homonuclear diatomic molecules: Compounds composed of two identical atoms.

In this section, we will only discuss elements in the period 2 of the Periodic Table.

To participate in molecular orbitals, atomic orbitals must overlap in space. This means that only
the valence orbitals of the atoms contribute significantly to the molecular orbitals of a particular
molecule.

For example, only the electrons in the 2s orbital of lithium (which has an electronic configuration
2 1
of 1s 2s ) participate in bonding. The 1s orbitals are smaller and do not overlap in space. The
2
electron configuration for Li2 (valence electrons only) is σ2s , and the bond order is 1.

For the beryllium molecule (Be2) the bonding and antibonding orbitals both contain two
electrons. In this case the bond order is 0, and since Be2 is not more stable than two separated
Be atoms, no molecule forms.
2 2 1
Since boron atom has a 1s 2s 2p configuration, the B2 molecule is described by considering
how p atomic orbitals combine to form molecular orbitals.

The bonding orbital is formed by reversing the sign of the right orbital so the positive phases of
both orbitals match between the nuclei to produce constructive interference. This leads to
enhanced electron probability between the nuclei.

The antibonding orbital is formed by the direct combination of the orbitals, which gives destructive
interference of the positive phase of one orbital with the negative phase of the second orbital.
This produces a nodde between the nuclei, which gives decreased electron probability.

General Chemistry 202-NYA-05: Chapter 9 188


σ molecular orbitals

π molecular orbitals

Note that in certain molecular bonding, they’re 2 sets of π molecular orbitals.

The B2 molecule has 6 valence electrons, and its bond


order is 1. It is a stable molecule to which we could
assign the molecular orbital energy diagram shown here.
However, it is wrong because of the effect of
paramagnetism.

General Chemistry 202-NYA-05: Chapter 9 189


Paramagnetism
Most materials have no magnetism until they are placed in a magnetic field. However, in the
presence of such a field, magnetism of two types can be induced.

Paramagnetism causes the substance to be attracted into the inducing magnetic field.

Diamagnetism causes the substance to be repelled from the inducing magnetic field.

Paramagnetism is associated with unpaired electrons and diamagnetism is associated with


paired electrons. Any substance that has both paired and unpaired electrons will exhibit a net
paramagnetism, since the effect of paramagnetism is much stronger than that of diamagnetism.

The correct molecular orbital energy-level diagram for the B2 molecule is the one below. When p-
s mixing is allowed, the energies of the σ2p and π2p orbitals are reversed. The two electrons from
the B 2p orbitals now occupy separate, degenerate π2p molecular orbitals and thus have parallel
spins. This diagram explains the observed paramagnetism of B2.

General Chemistry 202-NYA-05: Chapter 9 190


There are definite correlations between bond order, bond energy, and bond length. As the bond
order predicted by the molecular orbital model increases, the bond energy increases and the
bond length decreases.

Please do Example 9.6 and 9.7 in your textbooks.

Those three different representations of B2 illustrate the same phenomenon:

B B2 B

is is
equivalent equivalent
to to

General Chemistry 202-NYA-05: Chapter 9 191


9.4 Bonding in Heteronuclear Diatomic Molecules
Heteronuclear diatomic molecules : Compounds composed of two different atoms.

We will consider here two different cases.

Since the atoms involved in molecules containing atoms adjacent to each other in the periodic
table are so similar, the molecular orbital diagram for homonuclear molecules can be used.

For example: Nitric oxide (NO).

The molecule is paramagnetic


And has a bond order of 2.5

In the other case, when the two atoms of a diatomic molecule are very different, the energy-level
diagram for homonuclear molecules can no longer be used.

To keep things as simple as possible, we will assume that fluorine uses only one of its 2p orbitals
to bond to hydrogen. Thus the molecular orbitals for HF will be composed of fluorine 2p and
hydrogen 1s orbitals.

The 2p orbital of fluorine is shown at a lower energy than the 1s orbital of hydrogen on the
diagram because fluorine binds its valence electrons more tightly. The diagram predicts that the
HF molecule should be stable because both electrons are lowered in energy relative to their
energy in the free hydrogen and fluorine atoms, which is the driving force for bond formation.

Because the fluorine 2p orbital is lower in energy than the hydrogen 1s orbital, the electrons
prefer to be closer to the fluorine atom.

Please do Example 9.8 and exercices 9.47 and 9.48 in your textbooks

General Chemistry 202-NYA-05: Chapter 9 192


Extra Problems:

1. When comparing Be2 and H2:


I. Be2 is more stable because it contains both bonding and antibonding valence electrons.
II. H2 has a higher bond order than Be2.
III. H2 is more stable because it only contains σ1s electrons.
IV. H2 is more stable because it is diamagnetic, whereas Be2 is paramagnetic.
a) I, II
b) III only
c) II, III
d) II, III, IV
e) III, IV
2. If a molecule demonstrates paramagnetism, then :
I. The substance can have both paired and unpaired electrons.
II. The bond order is not a whole number.
III. It can be determined by drawing a Lewis structure.
IV. It must be an ion.
a) I, II
b) I, II, IV
c) II, III
d) I only
e) All of the above are correct.
3. For which of the following diatomic molecules would the bond order become greater if an electron is
removed (i.e., if the molecule is converted to the positive ion in its ground state)?
a) B2
b) C2
c) P2
d) F2
e) Na2
4. The configuration (σ2s)2(σ2s*)2(π2py)1(π2px)1 is the molecular orbital description for the ground state of
a) Li2+
b) Be2
c) B2
d) B22–
e) C2

General Chemistry 202-NYA-05: Chapter 9 193


5. Which of the following species is paramagnetic?
a) C2
b) B2
c) F2
d) H2
e) none of these
6. The fact that O2 is paramagnetic can be explained by
a) the Lewis structure of O2
b) resonance
c) a violation of the octet rule
d) the molecular orbital diagram for O2
e) hybridization of atomic orbitals in O2
7. For how many of the following does the bond order decrease if you add one electron to the neutral
molecule?
B2, C2, P2, F2
a) 0
b) 1
c) 2
d) 3
e) 4
8. Which of the following diatomic molecules has a bond order of 3?
a) B2
b) N2
c) He2
d) F2
e) Li2
9. Which of the following has the largest bond order?
a) N2
b) N2–
c) N22–
d) N2+
e) N22+
10. How many of the following: F2, B2, O2, N2 , are paramagnetic?
a) 0
b) 1
c) 2
d) 3
e) 4

11. Order the following from shortest to longest bond:


C2, B2, H2, N2
a) H2, N2, C2, B2
b) N2, C2, B2, H2
c) C2, N2, H2, B2
d) C2, B2, H2, N2
e) none of these
12. Which charge(s) on an O2 ion would give a bond order of 2.5?
a) –2
b) –1
c) +1
d) two of these
e) none of these

General Chemistry 202-NYA-05: Chapter 9 194


13. Which of the following statements about the molecule BN is false?
a) It is paramagnetic.
b) Its bond order is 2.
c) The total number of electrons is 12.
d) It has two pi bonds.
e) All of these are true.
14. Which of the following statements about the species CN– is false?
a) It is paramagnetic.
b) The total number of electrons is 14.
c) Its bond order is 3.
d) It has two pi bonds.
e) All of these are true.
15. Which of the nitrogen-containing molecules below is paramagnetic in its lowest energy state?
a) N2
b) NO
c) NH3
d) N2H4
e) none of these
16. Which of the following molecules or ions is not paramagnetic in its ground state?
a) O2
b) O2+
c) B2
d) NO
e) F2
17. Which of the following has the greatest bond strength?
a) B2
b) O2–
c) CN–
d) O2+
e) NO–
18. The bond order in the NO molecule is
a) 1
b) 1 12
c) 2
d) 2 12
e) 3
19. The CO molecule has the bond order:
a) 0
b) 1
c) 2
d) 3
e) 4

20. T F The bond order for CN– is 2.

Ans: 1)c 2)d 3)d 4)c 5)b 6)d 7)c 8)b 9)a 10)c 11)a 12)c 13)a 14)a 15)b 16)e 17)c 18)d
19)d 20)False

General Chemistry 202-NYA-05: Chapter 9 195


Hybridization of Molecules: Enrichment

HYBRIDIZATION ELECTRON PAIRS


Sigma (σ)
Lone pairs (l.p.) NOT pi (π)
sp 2
sp2 3
sp3 4
dsp3 5
d2sp3 6

How to determine hybridization of atoms in molecule with orbital diagram box schemes.

Example: CH2O H
σ l.p.
σ
C O
π l.p.
σ
H
1. Draw all orbital boxes for each atom.
2. Hybridize possible atoms (distribute the paired electrons if possible).
3. Fill the boxes by making pairs with other atoms.
4. Determine the nature of those pairs (l.p., σ, π).
5. Determine the hybridization of the molecule from the σ and l.p.

1s 1s
σ σ
H H

1s 2s 2p

C hybridized

1s 2s 2p

O
σ π

H: s

C: σ + σ + σ + π
sp2
2 2
So H = s, C = sp , O = sp . O: l.p. + l.p. + σ + π
sp2
General Chemistry 202-NYA-05: Chapter 9 196
Chapter 10

Liquids and Solids

th
Suggested problems in chapter 10 of Zumdahl 8 Edition:

33, 35, 37, 38, 39, 40, 101, 103.

General Chemistry 202-NYA-05: Chapter 10 197


Arrangements of molecules are significantly different in their gas, liquid or solid forms.

10.1 Intermolecular Forces


In Chapters 8 and 9 we saw that atoms can form stable units called molecules by sharing
electrons. This is called intramolecular (within the molecule) bonding.

In this chapter we consider the properties of the condensed states of matter (liquids and solids)
and the forces that cause the aggregation of the components of a substance to form a liquid or a
solid. These forces may involve covalent or ionic bonding, or they may involve weaker
interactions usually called intermolecular forces (because they occur between molecules, rather
than within).

It is important to recognize that when a substance such as water changes from solid to liquid to
gas, the molecules remain intact. The changes in states are due to changes in the forces among
the molecules rather than in those within the molecules.

For example, in ice, the molecules are virtually locked in place, although they can vibrate about
their positions. If energy is added, the motions of the molecules increase, and they eventually
achieve the greater movement and disorder characteristic of liquid water. The ice has melted. As
more energy is added, the gaseous state is eventually reached, with the individual molecules far
apart and increasing relatively little. However, the gas still consists of water molecules. (40.7 kJ
are needed to vaporize 1 mole of liquid water and 934 kJ are needed to break the O−H bonds in
1 mole of water molecules).

General Chemistry 202-NYA-05: Chapter 10 198


Dipole-Dipole Forces
Molecules with polar bonds often behave in an electric field as if they had a center of positive
charge and a center of negative charge. That is, they exhibit a dipole moment. Molecules with
dipole moments can attract each other electrostatically by lining up so that the positive and
negative ends are close to each other. This is a dipole-dipole attraction.

In a condensed state such as a liquid, where many molecules are in close proximity, the dipoles
find the best compromise between attraction and repulsion. That is, the molecules orient
themselves to maximize the ⊕---Θ interactions and to minimize ⊕---⊕ and Θ---Θ interactions.

Dipole-dipole forces are typically only about 1% as strong as covalent or ionic bonds, and they
rapidly become weaker as the distance between the dipoles increases. At low pressures in the
gas phase, where the molecules are far apart, these forces are relatively unimportant.

Particularly strong dipole-dipole forces, however, are seen among molecules in which hydrogen is
bound to a highly electronegative atom, such as nitrogen, oxygen, or fluorine. Two factors
account for the strengths of these interactions: the great polarity of the bond and the close
approach of the dipoles, allowed by the very small size of the hydrogen atom. Because dipole-
dipole attractions of this type are so unusually strong, they are given the special name HYDROGEN
BONDING.

HYDROGEN BONDING has a very important effect on the boiling point compounds.

These unusually strong hydrogen bonding forces are due to primarily to two factors. One factor is
the relatively large electronegativity values of the highest elements in each group, which leads to
especially polar H−X bonds. The second factor is the small size of the first element of each group,
which allows for the close approach of the dipoles, further strengthening the intermolecular
forces.
Hydrogen bonding is mainly observed with N―H, O―H, F―H and NOT with C―H.

General Chemistry 202-NYA-05: Chapter 10 199


London Dispersion Forces
The forces that exist among noble gas atoms and nonpolar molecules are called London
dispersion forces.

As the electrons move about the nucleus, a momentary nonsymmetrical electron distribution can
develop that produces a temporary dipolar arrangement of charge. The formation of this
temporary dipole can, in turn, affect the electron distribution of a neighbouring atom. That is, this
instantaneous dipole that occurs accidentally in a given atom can then induce a similar dipole in a
neighbouring atom.

This phenomena leads to an interatomic attraction that is relatively weak and short-lived but that
can be very significant especially for large atoms. For these interactions to become strong
enough to produce a solid, the motions of the atoms must be greatly slowed down. This explains,
for instance, why the noble gas elements have such low freezing points.

Polarizability indicates the ease with which the electron “cloud” of an atom can be distorted to
give a dipolar charge distribution.

This means that the importance of London dispersion forces increases greatly as the size of the
atom increases.

General Chemistry 202-NYA-05: Chapter 10 200


Strength Comparison Chart : Note that this comparison is only approximate- the actual relative
strengths will vary depending on the molecules involved.

Bond type Dissociation energy (kJ)


Covalent 400
Hydrogen bonds 12-16
Dipole-dipole 2.0 - 0.5
London (Van der Waals) Forces <1

(Remember that covalent bonds are intramolecular forces, not intermolecular forces)

10.9 Phase Diagrams


The three phases we are discussing are the solid, liquid and gas phases. Changing from one to
another is summarized in the figure below.

A phase diagram (next page) is a way to represent the phases of a substance as a function of
temperature and pressure.

A phase diagram describes conditions and events in a closed system, where no material can
escape into the surroundings and no air is present.

Notice that the diagram is not drawn to scale. This is done to emphasize certain features of the
diagram.

General Chemistry 202-NYA-05: Chapter 10 201


• Tm represents the normal melting point • T3 and P3 denote the triple point
• Tb represents the normal boiling point • Tc represents the critical temperature
• Pc represents the critical pressure

The negative slope of the solid/liquid line reflects the fact that the density of ice is less than that of
liquid water. That means that at a certain pressure, for instance −50°C, as we increase the
pressure, water goes from solid to liquid.

The triple point is where all three states of matter (in this example water) are present. In fact, only
under these conditions can all three states of matter coexist in a closed system.

Critical temperature (Tc) is the temperature above which the vapour cannot be liquefied no matter
what pressure is applied.

Critical pressure (Pc) is the pressure required to produce liquefaction at the critical temperature.

Together, the critical temperature and critical pressure define the critical point.

A fluid is neither true liquid nor vapour.

Applications of the phase diagram to water


A particular behaviour of water, which is opposite to that observed for most substances, occurs
because the density of ice is less than that of liquid water at the melting point.

At the melting point, liquid and solid water coexist. They are in dynamic equilibrium, since the rate
at which ice is melting is just balanced by the rate at which the water is freezing.

If we apply pressure, matter reduces its volume.

In other words, freezing point of water is less than 0°C when the pressure is greater than 1 atm.
For example, water boils at a lower temperature in Leadville, Colorado (3000 m above sea level)
than in San Diego, California (sea level).

General Chemistry 202-NYA-05: Chapter 10 202


The Phase Diagram for Carbon Dioxide

At a pressure of 1 atm, solid carbon dioxide sublimes at −78°C, a property that leads to its
common name, dry ice.

Carbon dioxide is often used in fire extinguishers, where it exists as a liquid at 25°C.

The liquid state does not exist at a pressure of 1 atm. The solid/liquid line has a positive slope,
since the density of solid carbon dioxide is greater than that of liquid carbon dioxide.

Extra Problems:

1. Order the intermolecular forces (dipole-dipole, London dispersion, ionic, and hydrogen-bonding)
from weakest to strongest .
a) dipole-dipole, London dispersion, ionic, and hydrogen-bonding
b) London dispersion, dipole-dipole, hydrogen-bonding, and ionic
c) hydrogen-bonding, dipole-dipole, London dispersion, and ionic
d) dipole-dipole, ionic, London dispersion, and hydrogen-bonding
e) London dispersion, ionic, dipole-dipole, and hydrogen-bonding
2. Hydrogen bonds account for which of the following observation?
a) Hydrogen naturally exists as a diatomic molecule.
b) Hydrogen is easily combustible with oxygen.
c) Water molecules are bent or "V-shaped."
d) Air is more dense than hydrogen gas.
e) For its molar mass, water has a high boiling point.
3. Which of the following would you expect to have the highest boiling point?
a) F2
b) Cl2
c) Br2
d) I2
e) All of the above have the same boiling point.

General Chemistry 202-NYA-05: Chapter 10 203


4. Which of the following should have the lowest boiling point?
a) Na2S
b) HF
c) NH3
d) N2
e) H2O
5. Which of the species below would you expect to show the least hydrogen bonding?
a) NH3
b) H2O
c) HF
d) CH4
e) all the same
6. The molecules in a sample of solid SO2 are attracted to each other by a combination of
a) London forces and H-bonding
b) H-bonding and ionic bonding
c) covalent bonding and dipole-dipole interactions
d) London forces and dipole-dipole interactions
e) none of these
7. The elements of group 5A, the nitrogen family, form compounds with hydrogen having the boiling
points listed below:
SbH3 –17°C, AsH3 –55°C, PH3 –87°C, NH3 –33°C
The first three compounds illustrate a trend where the boiling point decreases as the mass decreases;
however, ammonia (NH3) does not follow the trend because of
a) dipole-dipole attraction
b) metallic bonding
c) hydrogen bonding
d) London dispersion forces
e) ionic bonding
8. Which substance involves no bonding forces except London dispersion forces?
a) NaCl(l)
b) HF(l)
c) N2(s)
d) H2O(l)
e) K(s)
9. The bonds between hydrogen and oxygen within a water molecule can be characterized as?
a) hydrogen bonds
b) London dispersion forces
c) intermolecular forces
d) intramolecular forces
e) dispersion forces
10. When a water molecule forms a hydrogen bond with another water molecule, which atoms are
involved in the interaction?
a) a hydrogen from one molecule and a hydrogen from the other molecule
b) a hydrogen from one molecule and an oxygen from the other molecule
c) an oxygen from one molecule and an oxygen from the other molecule
d) an oxygen and a hydrogen from the same molecule
e) two hydrogens from one molecule and one hydrogen from the other molecule

General Chemistry 202-NYA-05: Chapter 10 204


11. Which of the following is the correct order of boiling points for KNO3, CH3OH, C2H6, Ne?
a) Ne < CH3OH < C2H6 < KNO3
b) KNO3 < CH3OH < C2H6 < Ne
c) Ne < C2H6 < KNO3 < CH3OH
d) Ne < C2H6 < CH3OH < KNO3
e) C2H6 < Ne < CH3OH < KNO3

Use the following to answer questions 12-14:

Consider the representations below to answer the next three questions.


(I) (II) (III)

12. How many of the following statements are correct concerning drawing I?
I. Each molecule induces a dipole onto the next molecule in close proximity.
II. The phenomenon shown is relatively weak and short-lived.
III. C8H18 contains this type of interaction.
IV. The forces that exist in this example are London dispersion forces.
a) 0
b) 1
c) 2
d) 3
e) 4
13. Which drawing best represents the interactions in a sample of HF?
a) I
b) II
c) III
d) I, II
e) all of the above
14. Which of the following statements are incorrect concerning drawing III?
a) Electrostatic interactions exist between the molecules.
b) The molecules find the best compromise between attraction and repulsion.
c) These molecules exhibit ionic bonding.
d) OCS exhibits this type of interaction.
e) Two of the above statements are incorrect.
15. The freezing point of helium is –270°C. The freezing point of xenon is –112°C. Both of these are in
the noble gas family. Which of the following statements is supported by these data?
a) Helium and xenon form highly polar molecules.
b) As the molecular weight of the noble gas increases, the freezing point decreases.
c) The London dispersion forces between the helium molecules are greater than the London
dispersion between the xenon molecules.
d) The London dispersion forces between the helium molecules are less than the London
dispersion forces between the xenon molecules.
e) None of these.

General Chemistry 202-NYA-05: Chapter 10 205


16. Which best explains the following trend?
Element b.p. (K)
He 4
Ne 25
Ar 95
Kr 125
Xe 170
a) London dispersion forces
b) dipole-dipole interaction
c) hydrogen bonding
d) Le Chatelier's principle
e) none of these
17. T F Intermolecular forces are weaker than intramolecular bonds.
18. T F Hydrogen bonding is a type of London dispersion force.
19. T F Methane (CH4) exhibits stronger hydrogen bond interactions than ammonia (NH3).
20. A certain substance, X, has a triple-point temperature of 20°C at a pressure of 2.0 atm. Which one of
the statements (A-D) cannot possibly be true?
a) X can exist as a liquid above 20°C.
b) X can exist as a solid above 20°C.
c) Liquid X can exist as a stable phase at 25°C, 1 atm.
d) Both liquid and solid X have the same vapor pressure at 20°C.
e) All of the statements (A-D) could be true.
21. Which statement regarding water is true?
a) Energy must be given off in order to break down the crystal lattice of ice to a liquid.
b) Hydrogen bonds are stronger than covalent bonds.
c) Liquid water is less dense than solid water.
d) Only covalent bonds are broken when ice melts.
e) All of the statements (A–D) are false.
22. The triple point of iodine is at 90 torr and 115°C. This means that liquid I2
a) is more dense than I2(s)
b) cannot exist above 115°C
c) cannot exist at 1 atmosphere pressure
d) cannot have a vapor pressure less than 90 torr
e) can exist at pressure of 10 torr
23. The triple point of CO2 is at 5.2 atm and –57°C. Under atmospheric conditions present in a typical
Boulder, Colorado, laboratory (P = 630 torr, T = 23°C), solid CO2 will:
a) remain solid
b) boil
c) melt
d) sublime
e) none of these

General Chemistry 202-NYA-05: Chapter 10 206


24. Choose the correct statement about the diagram below.

a) The diagram is qualitatively correct for water.


b) The diagram shows that the melting point of the solid increases with increasing pressure.
c) The diagram shows the triple point above 1 atm pressure.
d) The diagram could represent the phase diagram of CO2.
e) None of the above statements is correct.
25. Based on the phase diagram shown below, which of the following statements are correct?

I. Sublimation occurs at a point in the transformation that occurs along a straight line from point A to
point F.
II. C and E represent points where the gas and liquid phases are in equilibrium.
III. ∆Hvap can be measured at point B.
IV. Molecules at point D have a greater average kinetic energy than those at point F.
V. The temperature at point E is called the critical temperature of the compound.
a) II, V
b) I, III, IV
c) I, II, III
d) II, IV, V
e) I, II, IV
26. The density of the solid phase of a substance is 0.90 g/cm3 and the density of the liquid phase is 1.0
g/cm3. A large increase in pressure will
a) lower the freezing point
b) raise the freezing point
c) lower the boiling point
d) raise the triple point
e) lower the triple point

Ans: 1)b 2)e 3)d 4)d 5)d 6)d 7)c 8)c 9)d 10)b 11)d 12)e 13)b 14)c 15)d 16)a 17)True
18)False 19)False 20)c 21)e 22)d 23)d 24)b 25)e 26)a

General Chemistry 202-NYA-05: Chapter 10 207


General Chemistry 202-NYA-05: Chapter 10 208
Chapter 6

Thermochemistry

th
Suggested problems in chapter 6 of Zumdahl 8 Edition:

55, 59, 61, 67, 69, 71, 73, 75, 77, 79, 81, 83.

th
Suggested problems in chapter 8 of Zumdahl 8 Edition:

65, 67, 69, 71, 72, 77.

General Chemistry 202-NYA-05: Chapter 6 209


6.2 Enthalpy and Calorimetry
Enthalpy is a thermodynamic property of a thermodynamic system. It can be used to calculate the heat
transfer during a process taking place in a closed thermodynamic system under constant pressure.

Enthalpy is a state function defined as

H = E + PV

Where E is the internal energy of the system, P is the pressure of the system, and V is the volume of
the system.

This means that for a reaction studied at constant pressure, the flow of heat is a measure of the change
in enthalpy for the system. For this reason, the terms heat of reaction and change in enthalpy are used
interchangeably for reactions studied at constant pressure.

For a chemical reaction, the enthalpy change is given by the equation

∆H = Hproducts − Hreactants

In a case in which the products of a reaction have a greater enthalpy than the reactants, ∆H will be
positive. Thus heat will be absorbed by the system, and the reaction is endothermic (absorbs energy in
the form of heat).

On the other hand, if the enthalpy of the products is less than that of the reactnts, ∆H will be negative.
In this case, the overall decrease in enthalpy is achieved by the generation of heat, and the reaction is
exothermic (release energy in the form of heat).

For example, consider the combustion of propane (C3H8):

C3H8(g) + 5O2(g) → 3CO2(g) + 4H2O(l)

∆H = –2221 kJ

Assume that all of the heat comes from the combustion of propane. Calculate ∆H in which 5.00 g of
propane is burned in excess oxygen at constant pressure.

Solution:
∆H = (5.00 g C3H8)(1 mol / 44.094 g C3H8)(-2221 kJ / mol C3H8)
∆H = -252 kJ

Also do Example 6.4 in your text book.

General Chemistry 202-NYA-05: Chapter 6 210


Calorimetry
Calorimtery: the measurement of heat
Calorimeter: a device for measuring heat

Generally there are two types of calorimeters:

Bomb Calorimeter: a calorimeter in which the volume is held constant.


(Since the energy changes at constant volume, it is useful to calculate ∆E)
Adiabatic Calorimeter: a calorimeter that does not allow the exchange of heat between the system
and its surroundings. Also referred to as a Coffee-cup calorimeter.
(Since the energy changes at constant pressure, it’s useful to calculate ∆H)

One substance might require a great deal of heat energy to raise its temperature by one degree,
whereas another will exhibit the same temperature change after absorbing relatively little heat. The
heat capacity C is the measure of that property, defined by:

C = heat absorbed
Increase in temperature

The amount of substance present is important. It takes twice as much energy to raise the temperature
of two grams of water by one degree than it takes to raise the temperature of one gram of water by one
degree.

The specific heat capacity is the energy required to raise the temperature of one gram of a substance
by one degree Celsius; it is the heat capacity given per gram of substance (units are J⋅°C−1⋅g−1)

The molar heat capacity is the energy required to raise the temperature of one mole of substance by
one degree Celsius; it is the heat capacity given per mole of substance (units are J⋅°C−1⋅mol−1)

It takes much less energy to change the temperature of a gram of a metal by 1°C than for a gram of
water.

When reactants are mixed, the temperature of the mixed solution changes according to:

Energy released (heat) = s × m × ∆T

s = specific heat capacity (J/°C·g)


m = mass (g)
∆T = change in temperature (°C)

For example, if you have a Styrofoam cup with 50.0 g of water at 10°C, and you add a 50.0 g iron ball
at 90°C to the water. (sH2O = 4.18 J/°C·g and sFe = 0.45 J/°C·g) What is the final temperature of the
water is?

Solution:
−(50.0 g)(0.45 J/°C·g)(Tf – 90) °C = (50.0 g)(4.18 J/°C·g)(Tf – 10) °C
–0.1077 (Tf – 90)°C = (Tf – 10)°C
–0.1077Tf °C + 9.688°C = Tf°C – 10°C
19.688°C = 1.1077Tf°C
Tf = 18°C

General Chemistry 202-NYA-05: Chapter 6 211


6.3 Hess’s Law
Hess Law is a technique used to calculate the enthalpy of reaction (or indeed any other state function)
regardless of the actual mechanism (path) the system follows as it evolves from reactants to products.
It makes use of the fact that thermodynamic state functions are path-independent, therefore as long as
the initial and final states are well defined thermodynamically the change in the state function for the
process will be independent of the actual reaction mechanism. Because a state function is independent
of path, it is possible to choose any convenient reaction sequence to calculate the change of the state
function for the process and be assured that this change in state function is invariant to the actual
mechanism.

So basically: In going from a particular set of reactants to a particular set of products, the change in
enthalpy is the same whether the reaction takes place in one step or in a series of steps.

For example: N2(g) + 2O2(g) → 2NO2(g) ∆H1 = 68 kJ

This reaction also can be carried out in two distinct steps, with enthalpy changes designated by ∆H2
and ∆H3:

N2(g) + O2(g) → 2NO(g) ∆H2 = 180 kJ


2NO(g) + O2(g) → 2NO2(g) ∆H3 = – 112 kJ
N2(g) + 2O2(g) → 2NO2(g) ∆H2 + ∆H3 = 68 kJ

∆H1 = ∆H2 + ∆H3 = 68 kJ

Illustration of the principle of Hess’s law: The same change in enthalpy occurs when nitrogen and
oxygen react to form nitrogen dioxide, regardless of whether the reaction occurs in one (right) or two
(left) steps.

General Chemistry 202-NYA-05: Chapter 6 212


Characteristics of Enthalpy Changes
• If a reaction is reversed, the sign of ∆H is also reversed.
• The magnitude of ∆H is directly proportional to the quantities of reactants and products in a
reaction. If the coefficients in a balanced reaction are multiplied by an integer, the value of ∆H
is multiplied by the same integer.

Problem-Solving Strategy

• Work backward from the required reaction, using the reactants and products to decide how to
manipulate the other given reactions at your disposal.
• Reverse any reactions as needed to give the required reactants and products.
• Multiply reactions to give the correct numbers of reactants and products.

Example: Given the following data,


1. S (s) + 3/2 O2 (g) → SO3 (g) ∆H1 = –395.2 kJ
2. 2 SO2 (g) + O2 (g) → 2 SO3 (g) ∆H2 = –198.2 kJ
Calculate ∆Hr for the following reaction:
S (s) + O2 (g) → SO2 (g)

Solution: Combine the given chemical equations so that they add up to the desired chemical reaction.
Equation (1) S (s) + 3/2 O2 (g) → SO3 (g) ∆H1 = –395.2 kJ
–½ x Equation (2) SO3 (g) → SO2 (g) + ½ O2 (g) (–½) ∆H2 = – ½ (–198.2 kJ) = + 99. 1 kJ
S (s) + O2 (g) → SO2 (g) ∆Hr
∆Hr = ∆H1 + (– ½∆H2) = –395.2 kJ + 99. 1 kJ = –296.1 kJ

Example: Calculate ∆H° for the following reaction:


N2H4 (l) + O2 (g) → N2 (g) + 2 H2O (l)
Given the following data.
1. 2 NH3 (g) + 3 N2O (g) → 4 N2 (g) + 3 H2O (l) ∆H1 = –1010 kJ
2. N2O (g) + 3 H2 (g) → N2H4 (l) + H2O (l) ∆H2 = –317 kJ
3. 2 NH3 (g) + ½ O2 (g) → N2H4 (l) + H2O (l) ∆H3 = –143 kJ
4. H2 (g) + ½ O2 (g) → H2O (l) ∆H4 = –286 kJ
Solution:
(–1) x Equation (3) N2H4 (l) + H2O (l) → 2 NH3 (g) + ½ O2 (g) (g) –∆H3 = +143 kJ
Equation (1) 2 NH3 (g) + 3 N2O (g) → 4 N2 (g) + 3 H2O (l) ∆H1 = –1010 kJ
(–3) x Equation (2) 3 N2H4 (l) + 3 H2O (l) → 3 N2O (g) + 9 H2 (g) –3∆H2 = +951 kJ
(9) x Equation (4) 9 H2 (g) + 9/2 O2 (g) → 9 H2O (l) 9∆H4 = –2574 kJ
4 N2H4 (l) + 4 O2 (g) → 4 N2 (g) + 8 H2O (l) rest next page!

General Chemistry 202-NYA-05: Chapter 6 213


To obtain the appropriate equation, multiply this result by ¼
N2H4 (l) + O2 (g) → N2 (g) + H2O (l) ∆Hr
∆Hr = ¼ [(+143 kJ) + (–1010 kJ) + (+951 kJ) + (–2574 kJ)] = –622.5 kJ

Extra Problems:
1. How much heat is required to raise the temperature of a 4.48-g sample of iron (specific heat = 0.450 J/g°C)
from 25.0°C to 79.8°C?
a) 1.98 J
b) 246 J
c) 546 J
d) 661 J
e) 110 J
2. A 18.3 g piece of aluminum (which has a molar heat capacity of 24.03 J/°C·mol) is heated to 82.4°C and
dropped into a calorimeter containing water (specific heat capacity of water is 4.18 J/g°C) initially at
22.3°C. The final temperature of the water is 25.8°C. Ignoring significant figures, calculate the mass of
water in the calorimeter.
a) 63.1 g
b) 1.70 kg
c) 1.91 g
d) 923 g
e) none of these
3. A 45.9 g sample of a metal is heated to 95.5°C and then placed in a calorimeter containing 120.0 g of water
(c = 4.18 J/g°C) at 21.6°C. The final temperature of the water is 24.5°C. Which metal was used?
a) Aluminum (c = 0.89 J/g°C)
b) Iron (c = 0.45 J/g°C)
c) Copper (c = 0.20 J/g°C)
d) Lead (c = 0.14 J/g°C)
e) none of these
4. You take 243.8 g of a solid at 30.0°C and let it melt in 425 g of water. The water temperature decreases
from 85.1°C to 30.0°C. Calculate the heat of fusion of this solid.
a) 132 J/g
b) 201 J/g
c) 401 J/g
d) 669 J/g
e) cannot solve without the heat capacity of the solid
5. The enthalpy of fusion of ice is 6.020 kJ/mol. The heat capacity of liquid water is 75.4 J/mol·°C. What is
the smallest number of ice cubes at 0°C, each containing one mole of water, necessary to cool 500 g of
liquid water initially at 20°C to 0°C?
a) 1
b) 7
c) 14
d) 15
e) 126

General Chemistry 202-NYA-05: Chapter 6 214


6. 30.0 mL of pure water at 282 K is mixed with 50.0 mL of pure water at 328 K. What is the final temperature
of the mixture?
a) 305 K
b) 311 K
c) 397 K
d) 610 K
e) 46 K
7. The ∆H value for the reaction 1 2 O 2 (g ) + Hg(l ) → HgO(s ) is -90.8 kJ. How much heat is released when
63.8 g Hg is reacted with oxygen?
a) 0.318 kJ
b) 5.79 × 103 kJ
c) 28.9 kJ
d) 90.8 kJ
e) none of these
8. The total volume of hydrogen gas needed to fill the Hindenburg was 2.01 × 108 L at 1.00 atm and 24.7°C.
How much energy was evolved when it burned?
H 2 (g ) + 1 2 O 2 (g ) → H 2 O(l ), ∆H = −286 kJ

a) 8.23 × 106 kJ
b) 2.84 × 1010 kJ
c) 2.88 × 104 kJ
d) 2.35 × 109 kJ
e) 4.71 × 109 kJ
9. Consider the following processes:
2A → (1/2)B + C ∆H1 = 5 kJ/mol
(3/2)B + 4C → 2A + C + 3D ∆H2 = –15 kJ/mol
E + 4A → C ∆H3 = 10 kJ/mol
Calculate ∆H for: C → E + 3D
a) 0 kJ/mol
b) 10 kJ/mol
c) –10 kJ/mol
d) –20 kJ/mol
e) 20 kJ/mol

10. Consider the following processes:

∆H (kJ/mol)
3B → 2C + D –125.
(1/2)A → B 150
E+A→D 350
Calculate ∆H for: B → E + 2C
a) 325 kJ/mol
b) 525 kJ/mol
c) –175 kJ/mol
d) –325 kJ/mol
e) none of these

General Chemistry 202-NYA-05: Chapter 6 215


11. Consider the following numbered processes:
1. A → 2B
2. B→C+D
3. E → 2D
∆H for the process A → 2C + E is
a) ∆H1 + ∆H2 + ∆H3
b) ∆H1 + ∆H2
c) ∆H1 + ∆H2 – ∆H3
d) ∆H1 + 2∆H2 – ∆H3
e) ∆H1 + 2∆H2 + ∆H3
12. At 25°C, the following heats of reaction are known:
∆H (kJ/mol)
2ClF + O2 → Cl2O + F2O 167.4
2ClF3 + 2O2 → Cl2O + 3F2O 341.4
2F2 + O2 → 2F2O –43.4
At the same temperature, calculate ∆H for the reaction: ClF + F2 → ClF3
a) –217.5 kJ/mol
b) –130.2 kJ/mol
c) +217.5 kJ/mol
d) –108.7 kJ/mol
e) none of these

13. Calculate ∆H° for the reaction C4H4(g) + 2H2(g) → C4H8(g), using the following data:
∆H°combustion for C4H4(g) = –2341 kJ/mol
∆H°combustion for H2(g) = –286 kJ/mol
∆H°combustion for C4H8(g) = –2755 kJ/mol
a) –128 kJ
b) –158 kJ
c) 128 kJ
d) 158 kJ
e) none of these
14. Using the following thermochemical data, calculate ∆Hf° of Fe2O3(s).
2FeCl3(s) + 3H2O(l) → Fe2O3(s) + 6HCl(g) ∆ H° = 278.4 kJ/mol
2Fe(s) + 3Cl2(g) → 2FeCl3(s) ∆ H° = –799.0 kJ/mol
4HCl(g) + O2(g) → 2Cl2(g) + 2H2O(l) ∆ H° = –202.4 kJ/mol

a) –824.2 kJ/mol
b) –723.0 kJ/mol
c) 1279.8 kJ/mol
d) –875.0 kJ/mol
e) 318.2 kJ/mol
15. T F The specific heat capacities of metals are relatively low.
16. T F The change in enthalpy can always be thought of as equal to energy flow as heat.

Ans: 1) e 2)a 3)b 4)c 5)b 6)b 7)c 8)d 9)c 10)c 11)d 12)d 13)b 14)a 15)T 16)F

General Chemistry 202-NYA-05: Chapter 6 216


6.4 Standard Enthalpy of Formation
The Standard Enthalpy of Formation (∆Hf°) is the change in enthalpy that accompanies the
formation of one mole of a compound from its elements with all substances in their standard
states.

A degree symbol on a thermodynamic function, for example ∆Hf°, indicates that the
corresponding process has been carried out under standard conditions.

Conventional Definitions of Standard States


• For a Compound
The standard state of a gaseous substance is a pressure of exactly 1 atm.
For a pure substance in a condensed state (liquid or solid), the standard state is
the pure liquid or solid.
For a substance present in a solution, the standard state is a concentration of
exactly 1 M.
• For an Element
The form under which the element exists at 1 atm and 25°C [N2(g), K(s)].
Heat of formation is zero.

Definition of the Standard Enthalpy of Formation of a Compound:


The standard enthalpy of formation of a compound is the enthalpy change when
elements in their standard states react to form exactly one mole of the compound.

In this pathway for the combustion of methane, the reactants are first taken apart in reactions (a)
and (b) to form the constituent elements in their standard states, which are then used to
assemble the products in reaction (c) and (d).

You can see that this is a very exothermic reaction because very little energy is required to
convert the reactants to the respective elements, but a great deal of energy is released when
these elements form the products.

General Chemistry 202-NYA-05: Chapter 6 217


The reactants were broken down into the elements in their standard states. This process involved
reversing the formation reactions and thus switching the signs of the enthalpies of formation.

Schematic Diagram of the Energy Changes for the Reaction:

CH4(g) + 2O2(g) → CO2(g) + 2H2O(l)


∆H°reaction = –(–75 kJ) + 0 + (–394 kJ) + (–572 kJ) = –891 kJ

The enthalpy change for a given reaction can be calculated by subtracting the enthalpies of
formation of the reactants from the enthalpies of formation of the products. Remember to multiply
the enthalpies of formation by integers as required by the balanced equation.

∆H°rxn = Σnp∆Hf°(products) - Σnr∆Hf°(reactants)

Where Σ (sigma) means “sum of”, np and nr represent the moles of each product and reactant,
respectively.

Elements are not included in the calculation because elements require no change in form.

Problem-Solving Strategy: Enthalpy Calculations


1. When a reaction is reversed, the magnitude of ∆H remains the same, but its sign
changes.
2. When the balanced equation for a reaction is multiplied by an integer, the value of ∆H for
that reaction must be multiplied by the same integer.
3. The change in enthalpy for a given reaction can be calculated from the enthalpies of
formation of the reactants and products:
4. Elements in their standard states are not included in the ∆Hreaction calculations because
∆Hf° for an element in its standard state is zero.

Please do Example 6.9, 6.10 and 6.11 in your text book.

General Chemistry 202-NYA-05: Chapter 6 218


Example 1:
Calculate the enthalpy change for the combustion of glucose, given the enthalpy of formation
data of the elements.
C6H12O6 (s) + 6 O2 (g) → 6 CO2 (g) + 6 H2O (l)
o −1
∆H f ( kJ ⋅ mol ) –1275 0 –393.5 –286

Solution 1:
By using the definition of the standard state of a compound and Hess’ Law
o
1. 6 C (graphite) + 6 H2 (g) + 3 O2 (g) → C6H12O6 (s) ∆H f [C6 H 12 O6 ( s )] = −1275 kJ

2. O2 (g) → O2 (g) o
∆H f [O2 ( g )] = 0 kJ
3. C (graphite) + O2 (g) → CO2 (g) o
∆H f [CO2 ( g )] = −393.5 kJ

4. H2 (g) + ½ O2 (g) → H2O (l) o


∆H f [ H 2 O (l )] = −286 kJ
Apply Hess’ Law
(-1) x Equation (1) C6H12O6 (s) → 6 C (graphite) + 6 H2 (g) + 3 O2 (g) o
−∆H f [C6 H 12 O6 ( s )]
(–6) x Equation (2) 6 O2 (g) → 6 O2 (g) o
−6∆H f [O2 ( g )]
6 x Equation (3) 6 C (graphite) + 6 O2 (g) → 6 CO2 (g) o
6 ∆H f [CO2 ( g )]
6 x Equation (4) 6 H2 (g) + 3 O2 (g) → 6 H2O (l) o
6 ∆H f [ H 2 O (l )]
C6H12O6 (s) + 6 O2 (g) → 6 CO2 (g) + 6 H2O (g)

∆H ro = 6∆H of [CO2 ( g )] + 6∆H of [ H 2 O (l )] − ∆H of [C6 H12 O6 ( s )] − 6∆H of [O2 ( g )]


∆H ro = 6(−393.5) + 6(−286) − (−1275) − 6(0)
∆H ro = −2802 kJ

Example 2:
Calculate the enthalpy change for the following reaction:
2 Na (s) + 2 H2O (l) → 2 NaOH (s) + H2 (g)
o
∆H (kJ ⋅ mol )
f
−1
0 –286 –470 0

Solution 2:
∆H ro = 2∆H of [ NaOH ( s )] + ∆H of [ H 2 ( g )] − 2∆H of [ Na ( s )] − 2∆H of [ H 2 O (l )]
∆H ro = 2(−470) + (0) − 2(0) − 2(−286) = −368 kJ

General Chemistry 202-NYA-05: Chapter 6 219


8.8 Covalent Bond Energies and Chemical Reactions
In this section we will consider the energies associated with various types of bonds and see how
the bonding concept is useful in dealing with the energies of chemical reactions.

Bond Energies
• To break bonds, energy must be added to the system (endothermic).
• To form bonds, energy is released (exothermic).

∆H = Σn×D(bonds broken) – Σn×D(bonds formed)

D represents the bond energy per mole of bonds (always has a positive sign).

You do not need to know them by heart.

Do Example 8.5 in your text book.

General Chemistry 202-NYA-05: Chapter 6 220


Extra Problems:
1. The heat of formation of Fe2O3(s) is -826.0 kJ/mol. Calculate the heat of the reaction
4Fe(s ) + 3O 2 (g ) → 2Fe 2 O3 (s ) when a 27.42-g sample of iron is reacted.
a) –101.4 kJ
b) –202.8 kJ
c) –405.5 kJ
d) –811 kJ
e) –1.132 × 104 kJ
2. Which of the following does not have a standard enthalpy of formation equal to zero at 25°C and 1.0
atm?
a) F2(g)
b) Al(s)
c) H2O(l)
d) H2(g)
e) They all have a standard enthalpy equal to zero.
3. Given the following two reactions at 298 K and 1 atm, which of the statements is true?
1. N2(g) + O2(g) → 2NO(g) ∆H1
2. NO(g) + 2 O2(g) → NO2(g)
1 ∆H2

a) ∆Hf° for NO2(g) = ∆H2


b) ∆Hf° for NO(g) = ∆H1
c) ∆Hf° = ∆H2
d) ∆Hf° for NO2(g) = ∆H2 + 1
2 ∆H1
e) none of these

4. Given:
Cu2O(s) + 1 2 O2(g) → 2CuO(s) ∆H° = –144 kJ
Cu2O(s) → Cu(s) + CuO(s) ∆H° = +11 kJ
Calculate the standard enthalpy of formation of CuO(s).
a) –166 kJ
b) –299 kJ
c) +299 kJ
d) +155 kJ
e) –155 kJ
5. Using the following data, calculate the standard heat of formation of the compound ICl in kJ/mol:
∆H° (kJ/mol)
Cl2(g) → 2Cl(g) 242.3
I2(g) → 2I(g) 151.0
ICl(g) → I(g) + Cl(g) 211.3
I2(s) → I2(g) 62.8
a) –211 kJ/mol
b) –14.6 kJ/mol
c) 16.8 kJ/mol
d) 245 kJ/mol
e) 439 kJ/mol

General Chemistry 202-NYA-05: Chapter 6 221


6. The heat combustion of acetylene, C2H2(g), at 25°C is –1299 kJ/mol. At this temperature, ∆Hf°
values for CO2(g) and H2O(l) are –393 and –286 kJ/mol, respectively. Calculate ∆Hf° for acetylene.
a) 2376 kJ/mol
b) 625 kJ/mol
c) 227 kJ/mol
d) –625 kJ/mol
e) –227 kJ/mol
7. Choose the correct equation for the standard enthalpy of formation of CO(g), where ∆Hf° for CO = –
110.5 kJ/mol (gr indicates graphite).
a) 2C(gr) + O2(g) → 2CO(g), ∆H° = –110.5 kJ
b) C(gr) + O(g) → CO(g), ∆H° = –110.5 kJ
c) C(gr) + 1 2 O2(g) → CO(g), ∆H° = –110.5 kJ
d) C(gr) + CO2(g) → 2CO(g), ∆H° = –110.5 kJ
e) CO(g) → C(gr) + O(g), ∆H° = –110.5 kJ
8. For the reaction:
AgI(s) + 1
2 Br2(g) → AgBr(s) + 1 I (s), ∆H° = –54.0 kJ
2 2

∆Hf° for AgBr(s) = –100.4 kJ/mol


∆Hf° for Br2(g) = +30.9 kJ/mol
The value of ∆Hf° for AgI(s) is:
a) –123.5 kJ/mol
b) +77.3 kJ/mol
c) +61.8 kJ/mol
d) –77.3 kJ/mol
e) –61.8 kJ/mol
9. Using the information below, calculate ∆Hf° for PbO(s)
PbO(s) + CO(g) → Pb(s) + CO2(g) ∆H° = –131.4 kJ
∆Hf° for CO2(g) = –393.5 kJ/mol
∆Hf° for CO(g) = –110.5 kJ/mol
a) –151.6 kJ/mol
b) –283.0 kJ/mol
c) +283.0 kJ/mol
d) –372.6 kJ/mol
e) +252.1 kJ/mol
10. Using the following bond energies:
Bond Bond Energy (kJ/mol)
C≡C 839
C–H 413
O=O 495
C=O 799
O–H 467
estimate the heat of combustion for one mole of acetylene:
C2H2(g) + 5 2 O2(g) → 2CO2(g) + H2O(g)
a) 1228 kJ
b) –1228 kJ
c) –447 kJ
d) +447 kJ
e) +365 kJ

General Chemistry 202-NYA-05: Chapter 6 222


11. Using the following data reactions:
∆H° (kJ)
H2(g) + Br2(g) → 2HBr(g) –103
H2(g) → 2H(g) 432
Br2(g) → 2Br(g) 193
calculate the energy of an H-Br bond.
a) 728 kJ
b) 261 kJ
c) 522 kJ
d) 52 kJ
e) 364 kJ
12. Given the following bond energies:
C–C 347 kJ/mol
C=C 614 kJ/mol
C–O 358 kJ/mol
C=O 799 kJ/mol
C–H 413 kJ/mol
O–H 463 kJ/mol
O–O 146 kJ/mol
estimate ∆H for the reaction H2O2 + CH3OH → H2CO + 2H2O.
a) –345 kJ b) –199 kJ c) –105 kJ d) +199 kJ e) +345 kJ

13. Given the following information:


Br2 bond energy = 193 kJ/mol
F2 bond energy = 154 kJ/mol
Br2(g) + 3 2 F2(g) → BrF3(g)
1
2 ∆H° = –384 kJ/mol
calculate the Br-F bond energy.
a) 244 kJ/mol b) 237 kJ/mol c) 712 kJ/mol d) 128 kJ/mol e) none of these

14. Consider the following reaction:


A2 + B2 → 2AB ∆H = –321 kJ
The bond energy for A2 is half the amount of AB. The bond energy of B2 = 393 kJ/mol. What is the
bond energy of A2?
a) 714 kJ/mol b) 554 kJ/mol c) 238 kJ/mol d) –161 kJ/mol e) none of these

Good job! Now review the whole semester ;) Keep up the good work!

Cheers

- Yann

Ans: 1)b 2)c 3)d 4)e 5)c 6)c 7)c 8)e 9)a 10)b 11)e 12)a 13)b 14)c

General Chemistry 202-NYA-05: Chapter 6 223


PERIODIC TABLE OF THE ELEMENTS
1 2
1.0079 4.003
H He
hydrogen helium
hydrogène hélium
3 4 atomic number 5 6 7 8 9 10
6.941 9.012 atomic mass 10.811 12.011 14.007 15.9994 18.998 20.18
Li Be Symbol B C N O F Ne
lithium beryllium English name boron carbon nitrogen oxygen fluorine neon
lithium béryllium French name* bore carbone azote oxygène fluor néon
11 12 *all are masculine 13 14 15 16 17 18
22.99 24.31 26.98 28.086 30.974 32.07 35.453 39.95
Na Mg Al Si P S Cl Ar
sodium magnesium aluminum silicon phosphorus sulfur chlorine argon
sodium magnésium aluminium silicium phosphore soufre chlore argon
19 20 21 22 23 24 25 26 27 28 29 30 31 32 33 34 35 36
39.098 40.08 44.96 47.87 50.94 51.996 54.94 55.85 58.93 58.69 63.546 65.39 69.72 72.61 74.922 78.96 79.904 83.80
K Ca Sc Ti V Cr Mn Fe Co Ni Cu Zn Ga Ge As Se Br Kr
potassium calcium scandium titanium vanadium chromium manganese iron cobalt nickel copper zinc gallium germanium arsenic selenium bromine krypton
potassium calcium scandium titane vanadium chrome manganèse fer cobalt nickel cuivre zinc gallium germanium arsenic sélénium brome krypton
37 38 39 40 41 42 43 44 45 46 47 48 49 50 51 52 53 54
85.468 87.62 88.91 91.22 92.91 95.94 98 101.07 102.91 106.42 107.87 112.411 114.82 118.71 121.76 127.60 126.904 131.29
Tc
Rb Sr Y Zr Nb Mo technetium Ru Rh Pd Ag Cd In Sn Sb Te I Xe
rubidium strontium yttrium zirconium niobium molybdenum technétium ruthenium rhodium palladium silver cadmium indium tin antimony tellurium iodine xenon
rubidium strontium yttrium zirconium niobium molybdène ruthénium rhodium palladium argent cadmium indium étain antimoine tellure iode xénon
55 56 57 72 73 74 75 76 77 78 79 80 81 82 83 84 85 86
132.91 137.33 138.91 178.49 180.95 183.84 186.21 190.21 192.22 195.08 196.97 200.59 204.38 207.2 208.98 209 210 222

Cs Ba La Hf Ta W Re Os Ir Pt Au Hg Tl Pb Bi Po At Rn
cesium barium lanthanum hafnium tantalum tungsten rhenium osmium iridium platinum gold mercury thallium lead bismuth polonium astatine radon
césium baryum lanthane hafnium tantale tungstène rhénium osmium iridium platine or mercure thallium plomb bismuth polonium astate radon
87 88 89 104 105 106 107 108 109 110 111 112
223 226.03 227.03 261 262 266 264 269 268 269 272 277

Fr Ra Ac Rf Unp Unh Uns Uno Une Uun Uuu Uub


francium radium actinium rutherfordium unnilpentium unnilhexium unnilseptium unnilloctium unnilennium ununnilium unununium ununbium
francium radium actinium rutherfordium unnilpentium unnilhexium unnilseptium unnilennium unnilennium ununnilium unununium ununbium

58 59 60 61 62 63 64 65 66 67 68 69 70 71
140.12 140.91 144.24 145 150.36 151.96 157.25 158.93 162.50 164.93 167.26 168.93 173.04 174.97

Ce Pr Nd Pm Sm Eu Gd Tb Dy Ho Er Tm Yb Lu
cerium praseodym- neodymium promethium samarium europium gadolinium terbium dysprosium holmium erbium thulium ytterbium lutetium
cérium ium néodyme prométhium samarium europium gadolinium terbium dysprosium holmium erbium thulium ytterbium lutécium
praséodyme
90 91 92 93 94 95 96 97 98 99 100 101 102 103
232.038 231.036 238.029 237.048 244 243 247 247 251 252 257 258 259 262

Th Pa U Np Pu Am Cm Bk Cf Es Fm Md No Lr
thorium protactinium uranium neptunium plutonium americium curium berkelium californium einsteinium fermium mendelevium nobelium lawrencium
thorium protactinium uranium neptunium plutonium américium curium berkélium californium einsteinium fermium mendélévium nobélium lawrencium

You might also like